Você está na página 1de 154

CASES:

REGALIAN DOCTRINE

SECOND DIVISION

[G.R. No. 50464. January 29, 1990.]

SUNBEAM CONVENIENCE FOODS INC., CORAL BEACH


DEVELOPMENT CORP., and the
REGISTER OF DEEDS OF BATAAN, petitioners, vs. HON. COU
RT OFAPPEALS and THE REPUBLIC OF THE
PHILIPPINES, respondents.

Filoteo T. Banzon for petitioners.

SYLLABUS

1. REMEDIAL LAW; PETITION FOR REVIEW ON CERTIORARI; GRANT


THEREOF; NOT A MATTER OF RIGHT. — A petition for review on certiorari is
not a matter of right but ofsound judicial discretion, and is granted only when
there are special and important reasons therefore. The following, while neither
controlling nor fully measuring the Court's discretion, enumerates the
premises for granting a review: (a) When the Court of Appeals has decided a
question of substance, not theretofore determined by the SupremeCourt or has
decided it in a way probably not in accord with law or the applicable
decisions of the Supreme Court; and (b) When the Court of Appeals has so far
departed from the accepted and usual course of judicial proceedings or so far
sanctioned such departure by a lower court as to call for supervision.

2. ID.; RULES OF PROCEDURE CONSTRUED; DESIGNED TO FACILITATE


ATTAINMENT OF JUSTICE. — Generally, the rules of procedure must be
observed so that the efficient administration of justice is ensured. However, the
rules of procedure should be viewed as mere tools designed to facilitate the
attainment of justice. They must lead to the proper and just
determination of litigation, without tying the hands of the law or making it
indifferent to realities.

3. ID.; SPECIAL CIVIL ACTION; CERTIORARI; GROUNDS FOR GRANT


THEREOF. — Considered extraordinary, certiorari is made available only when
there is no appeal, nor any plain, speedy or adequate remedy in the ordinary
course of the law. The long line of decisions denying the petition for certiorari,
either before appeal was availed of or specially in instances where the appeal
period has lapsed, far outnumbers the instances when certiorari was given due
course. The few significant exceptions were: when public welfare and the
advancement of public policy dictate; or when the broader interests of justice
so require, or when the writs issued are null, or when the questioned order
amounts to an oppressive exercise of judicial authority.

4. LAND TITLES; FOREST LANDS; TO BE ALIENABLE REQUIRE, AN OFFICIAL


PROCLAMATION TO THAT EFFECT. — Our adherence to the Regalian doctrine
subjects all agricultural, timber, and mineral lands to the dominion of the
State. Thus, before any land may be declassified from the forest group and
converted into alienable or disposable land for agricultural or other purposes,
there must be a positive act from the government. Even rules on the
confirmation of imperfect titles do not apply unless and until the land classified
as forest land is released in an official proclamation to that effect so that it may
form part of the disposable agricultural lands of the public domain. The mere
fact that a title was issued by the Director of Lands does not confer any validity
on such title if the property covered by the title or patent is part of the public
forest.

DECISION
SARMIENTO, J p:

In this petition for review on


certiorari, Sunbeam Convenience Foods Corporation (hereafter
simply SUNBEAM) and Coral Beach Development Corporation (hereafter simply
CORAL BEACH) bring to our attention the decision rendered by
the Court of Appeals in "Republic of the Philippines v. Hon. Pedro T. Santiago,
et al.," disposing as follows:

WHEREFORE, the writ prayed for is granted. The order of the


respondent judge dated October 7, 1977, dismissing Civil Case
No. 4062 is set aside, and respondent judge is ordered to require
private respondents to file their answer to the complaint in said
Civil Case No. 4062 and thereafter to proceed with the trial of the
case on the merits and to render judgment thereon. 1

The following facts stated by the respondent Court in its decision and restated
by the petitioners in their petition are accurate:

a) On April 29, 1963, the Director of Lands caused the


issuance of a Sales Patent in
favor of defendant Sunbeam Convenience Foods, Inc., over the
parcels of land both situated in Mariveles, Bataan and more
particularly described and bounded as follows:

Lot 1-Sgs-2409 (area 3,113,695 sq. m.)


Lot 2-Sgs-2409 (area 1,401,855 sq. m.)

(b) On May 3, 1963, the aforesaid Sales Patent was registered


with the defendant Register of Deeds of Bataan who in turn
issued Original Certificate of Title No. Sp-24 in
favorof defendant Sunbeam Convenience Foods, Inc., for the two
parcels of land above-described;
(c) Subsequently, Original Certificate of Title No. Sp-24 was
cancelled and in lieu thereof, Transfer Certificate of Title No. T-
12421 was issued over Lot 1, Sgs-2409, while Transfer
Certificate of Title No. 12422 was issued over Lot 2, Sgs-2409,
both in favor of defendant Coral Beach Development Corporation;

(d) On May 11, 1976, the Solicitor General in the name of the
Republic of the Philippines instituted before the Court of First
Instance of Bataan, an action for reversion docketed as Civil Case
No. 4062. 2

SUNBEAM and CORAL BEACH filed a Motion to Dismiss on the following


grounds:

1. The Republic of the Philippines should have exhausted all


administrative remedies before filing the case in court;

2. The title issued to SUNBEAM and CORAL BEACH had become


indefeasible and imprescriptible;

3. The action for reversion was defective, having been initiated by


the Solicitor General and not by the Director of Lands. 3

The then Court of First Instance of Bataan dismissed the complaint in the
Order of October 7, 1977, 4 adopting mainly the theory that since the titles
sought to be cancelled emanated from the administrative act of the
Bureau of Lands Director, the latter, not the courts, had jurisdiction over the
disposition of the land. LLphil

The Solicitor General received the copy of the Order on October 11, 1977 and
filed a Notice of Appeal dated October 25, 1977. 5 The Solicitor General then
moved for an extension of thirty days within which to file the Record on Appeal
and to pay the docket fee in order to perfect the appeal. This was to be followed
by another motion for extension filed by the Solicitor General, resulting in
the Court of Appeals granting the petitioner another extension of fifteen days
from December 10, 1977. Finally before this period of extension lapsed,
instead of an appeal, a petition for certiorari with the
respondent Court of Appeals was filed.

According to the Solicitor General, the Court of First Instance committed grave
abuse of discretion in dismissing the complaint and in —

a. Not finding that since the lower court acted in a Motion to


Dismiss, the correctness of its decision must be decided in the
assumed truth and accuracy of the allegations ofthe complaint.
The complaint alleges that the lands in question are forest lands;
hence, inalienable.

b. Finding that Lots 1 and 2 are alienable and disposable


lands of the public domain under the jurisdiction of the
Director of Lands despite clear and positive evidence to the
contrary.

c. Concluding that the complaint for reversion is defective as it


was not initiated by the Director of Lands.

d. Finding that the complaint for reversion states no


cause of action for alleged failure of petitioner to exhaust
administrative remedies. 6

The Court of Appeals gave due course to the petition for certiorari, set aside the
Order of Dismissal rendered by the Court of First Instance in Civil Case No.
4062, and ordered the presiding judge Hon. Pedro T. Santiago to receive the
answers of the private respondents SUNBEAM and CORAL BEACH in the
action for reversion.

Hence Sunbeam and Coral Beach filed this petition for review. prLL

A review is not a matter of right but of sound judicial discretion, and is granted
only when there are special and important reasons therefore. The following,
while neither controlling nor fully measuring the Court's discretion,
enumerates the premises for granting a review:

(a) When the Court of Appeals has decided a


question of substance, not theretofore determined by the
Supreme Court or has decided it in a way probably not in
accord with law or the applicable decisions of the
Supreme Court; and

(b) When the Court of Appeals has so far departed from the
accepted and usual course of judicial proceedings or so far
sanctioned such departure by a lowercourt as to call for
supervision. 7

We agree with the Court of Appeals' granting of the petition filed by the
Republic of the Philippines charging the then Court of First Instance with grave
abuse of discretion.

The filing of the Motion to Dismiss the complaint for reversion


by SUNBEAM and CORAL BEACH on the ground of lack of cause of action,
necessarily carried with it the admission, for purposes of the motion, of the
truth of all material facts pleaded in the complaint instituted by the Republic.

An important factual issue raised in the complaint was the classification of the
lands as forest lands. This material allegation stated in the Republic's
complaint 8 was never denied specifically 9 by the defendants (petitioners
herein) SUNBEAM and CORAL BEACH.

If it is true that the lands are forest lands, then all these proceedings become
moot and academic. Land remains unclassified land until it is released
therefrom and rendered open to disposition. 10

Our adherence to the Regalian doctrine subjects all agricultural, timber, and
mineral lands to the dominion of the State. 11 Thus, before any land may be
declassified from the forest group and converted into alienable or disposable
land for agricultural or other purposes, there must be a positive act from the
government. Even rules on the confirmation of imperfect titles do not apply
unless and until the land classified as forest land is released in an official
proclamation to that effect so that it may form part of the disposable
agricultural lands of the public domain. 12

The mere fact that a title was issued by the Director of Lands does not confer
any validity on such title if the property covered by the title or patent is
part of the public forest. 13

The only way to resolve this question of fact as to the classification of the land
is by remanding the case to the lower court for a full-dress trial on the issues
involved.

Generally, the rules of procedure must be observed so that the efficient


administration of justice is ensured. However, the rules of procedure should be
viewed as mere tools designed to facilitate the attainment of justice. 14 They
must lead to the proper and just determination of litigation, without tying the
hands of the law or making it indifferent to realities. cdll

Certiorari is one such remedy. Considered extraordinary, it is made available


only when there is no appeal, nor any plain, speedy or adequate remedy in the
ordinary course ofthe law. 15 The long line of decisions denying the petition for
certiorari, either before appeal was availed of or specially in instances where
the appeal period has lapsed, far outnumbers the instances when certiorari
was given due course. The few significant exceptions were: when public welfare
and the advancement of public policy dictate; or when the broader
interests of justice so require, or when the writs issued are null, 16 or when
the questioned order amounts to an oppressive exercise of judicial
authority. 17
We find nothing disagreeable with the action of the Court of Appeals to give due
course to the petition considering that the issue affected a matter of public
concern which is the disposition of the lands of our patrimony. No less than
the Constitution protects this policy. prLL

We therefore find no compelling reason to disturb the findings of the


appellate court, in the absence of a clear showing that the Court of Appeals has
decided a question ofsubstance in a manner inconsistent with jurisprudence,
or that the respondent Court has departed from the accepted and usual
course of judicial proceedings. In sum, no reversible error has been committed
by the respondent court. 18

WHEREFORE, the petition is DENIED and the


decision of the Court of Appeals is affirmed. Costs against the petitioners.

SO ORDERED.

Melencio-Herrera, Paras, Padilla and Regalado, JJ., concur.

||| (Sunbeam Convenience Foods Inc. v. Court of Appeals, G.R. No. 50464,
[January 29, 1990], 260 PHIL 470-477)
[G.R. No. 60413. October 31, 1990.]

REPUBLIC OF THE PHILIPPINES, petitioner, vs. HON.


SOFRONIO G. SAYO, Judge, Br. I, CFI, Nueva Vizcaya, HEIRS
OF CASIANO SANDOVAL, HEIRS OF LIBERATO BAYAUA,
JOSE C. REYES, and PHILIPPINE CACAO AND FARM
PRODUCTS INC., respondents.

Celso D. Gangan respondent Heirs of Liberato Bayaua.

Acosta & Associates for Phil. Cacao and Farm Products, Inc.

Jose Reyes & Associates for Heirs of Casiano Sandoval, et al.

DECISION

NARVASA, J p:

Sought to be annulled and set aside in this special civil action of certiorari is
the decision of respondent Judge Sofronio G. Sayo rendered on March 5, 1981
in Land Registration Case No. N-109, LRC Record No. 20850, confirming, by
virtue of a compromise agreement, the title of the private respondents over a
tract of land. LLjur

The spouses, Casiano Sandoval and Luz Marquez, filed an original application
for registration of a tract of land identified as Lot No. 7454 of the Cadastral
Survey of Santiago, BL Cad. 211 (July 17, 1961) and having an area of 33,950
hectares. The land was formerly part of the Municipality of Santiago, Province
of Isabela, but had been transferred to Nueva Vizcaya in virtue of Republic Act
No. 236.

Oppositions were filed by the Government, through the Director of Lands and
the Director of Forestry, and some others, including the Heirs of Liberato
Bayaua. 1 In due course, an order of general default was thereafter entered on
December 11, 1961 against the whole world except the oppositors.

The case dragged on for about twenty (20) years until March 3, 1981 when a
compromise agreement was entered into by and among all the parties, assisted
by their respective counsel, namely: the Heirs of Casiano Sandoval (who had
since died), the Bureau of Lands, the Bureau of Forest Development, the Heirs
of Liberato Bayaua, and the Philippine Cacao and Farm Products, Inc. Under
the compromise agreement, the Heirs of Casiano Sandoval (as applicants)
renounced their claims and ceded —

1) in favor of the Bureau of Lands, an area of 4,109 hectares;

2) in favor of the Bureau of Forest Development, 12,341 hectares;

3) in favor of the Heirs of Liberato Bayaua, 4,000 hectares; and

4) in favor of Philippine Cacao & Farm Products, Inc., 8,000 hectares.

The remaining area of 5,500 hectares was, under the compromise agreement,
adjudicated to and acknowledged as owned by the Heirs of Casiano Sandoval,
but out of this area, 1,500 hectares were assigned by the Casiano Heirs to their
counsel, Jose C. Reyes, in payment of his attorney's fees. In consideration of
the areas respectively allocated to them, all the parties also mutually waived
and renounced all their prior claims to and over Lot No. 7454 of the Santiago
Cadastre.

In a decision rendered on March 5, 1981, the respondent Judge approved the


compromise agreement and confirmed the title and ownership of the parties in
accordance with its terms.

The Solicitor General, in behalf of the Republic of the Philippines, has taken
the present recourse in a bid to have that decision of March 5, 1981 annulled
as being patently void and rendered in excess of jurisdiction or with grave
abuse of discretion. The Solicitor General contends that —
1) no evidence whatever was adduced by the parties in support of their
petitions for registration;

2) neither the Director of Lands nor the Director of Forest Development had
legal authority to enter into the compromise agreement;

3) as counsel of the Republic, he should have been but was not given notice of
the compromise agreement or otherwise accorded an opportunity to take part
therein;

4) that he was not even served with notice of the decision approving the
compromise; it was the Sangguniang Panlalawigan of Quirino Province that
drew his attention to the "patently erroneous decision" and requested him to
take immediate remedial measures to bring about its annulment.

The respondents maintain, on the other hand, that the Solicitor General's
arguments are premised on the proposition that Lot 7454 is public land, but it
is not. According to them, as pointed out in the application for registration, the
private character of the land is demonstrated by the following circumstances,
to wit:

1) the possessory information title of the applicants and their predecessors-in-


interest;

2) the fact that Lot 7454 was never claimed to be public land by the Director of
Lands in the proper cadastral proceedings;

3) the pre-war certification of the National Library dated August 16, 1932 to the
effect that the Estadistica de Propiedades of Isabela issued in 1896 and
appearing in the Bureau of Archives, the property in question was registered
under the 'Spanish system of land registration as private property owned by
Don Liberato Bayaua, applicants' predecessors-in-interest;

4) the proceeding for registration, brought under Act 496 (the Torrens Act)
presupposes that there is already a title to be confirmed by the court,
distinguishing it from proceedings under the Public Land Act where the
presumption is always that the land involved belongs to the State.

Under the Regalian Doctrine, 2 all lands not otherwise appearing to be clearly
within private ownership are presumed to belong to the State. Hence it is that
all applicants in land registration proceedings have the burden of overcoming
the presumption that the land thus sought to be registered forms part of the
public domain. 3 Unless the applicant succeeds in showing by clear and
convincing evidence that the property involved was acquired by him or his
ancestors either by composition title from the Spanish Government or by
possessory information title, or any other means for the proper acquisition of
public lands, the property must be held to be part of the public domain. 4 The
applicant must present competent and persuasive proof to substantiate his
claim; he may not rely on general statements, or mere conclusions of law other
than factual evidence of possession and title. 5

In the proceeding at bar, it appears that the principal document relied upon
and presented by the applicants for registration, to prove the private character
of the large tract of land subject of their application, was a photocopy of a
certification of the National Library dated August 16, 1932 (already above
mentioned) to the effect that according to the Government's Estadistica de
Propiedades of Isabela issued in 1896, the property in question was registered
under the Spanish system of land registration as private property of Don
Liberato Bayaua. But, as this Court has already had occasion to rule, that
Spanish document, the Estadistica de Propiedades, cannot be considered a title
to property, it not being one of the grants made during the Spanish regime, and
obviously not constituting primary evidence of ownership. 6 It is an
inefficacious document on which to base any finding of the private character of
the land in question.

And, of course, to argue that the initiation of an application for registration of


land under the Torrens Act is proof that the land is of private ownership, not
pertaining to the public domain, is to beg the question. It is precisely the
character of the land as private which the applicant has the obligation of
establishing. For there can be no doubt of the intendment of the Land
Registration Act, Act 496, that every applicant show a proper title for
registration; indeed, even in the absence of any adverse claim, the applicant is
not assured of a favorable decree by the Land Registration Court, if he fails to
establish a proper title for official recognition.

It thus appears that the decision of the Registration Court a quo is based solely
on the compromise agreement of the parties. But that compromise agreement
included private persons who had not adduced any competent evidence of their
ownership over the land subject of the registration proceeding. Portions of the
land in controversy were assigned to persons or entities who had presented
nothing whatever to prove their ownership of any part of the land. What was
done was to consider the compromise agreement as proof of title of the parties
taking part therein, a totally unacceptable proposition. The result has been the
adjudication of lands of no little extension to persons who had not submitted
any substantiation at all of their pretensions to ownership, founded on nothing
but the agreement among themselves that they had rights and interests over
the land.cdll

The assent of the Directors of Lands and Forest Development to the


compromise agreement did not and could not supply the absence of evidence of
title required of the private respondents.

As to the informacion posesoria invoked by the private respondents, it should


be pointed out that under the Spanish Mortgage Law, it was considered a mode
of acquiring title to public lands, subject to two (2) conditions: first, the
inscription thereof in the Registry of Property, and second, actual, public,
adverse, and uninterrupted possession of the land for twenty (20) years (later
reduced to ten [10] years); but where, as here, proof of fulfillment of these
conditions is absent, the informacion posesoria cannot be considered as
anything more than prima facie evidence of possession. 7

Finally, it was error to disregard the Solicitor General in the execution of the
compromise agreement and its submission to the Court for approval. It is, after
all, the Solicitor General, who is the principal counsel of the Government; this
is the reason for our holding that "Court orders and decisions sent to the fiscal,
acting as agent of the Solicitor General in land registration cases, are not
binding until they are actually received by the Solicitor General." 8

It thus appears that the compromise agreement and the judgment approving it
must be, as they are hereby, declared null and void, and set aside.
Considerations of fairness however indicate the remand of the case to the
Registration Court so that the private parties may be afforded an opportunity to
establish by competent evidence their respective claims to the property. Cdpr

WHEREFORE, the decision of the respondent Judge complained of is


ANNULLED and SET ASIDE. Land Registration Case No. N-109 subject of the
petition is REMANDED to the court of origin which shall conduct further
appropriate proceedings therein, receiving the evidence of the parties and
thereafter rendering judgment as such evidence and the law may warrant. No
pronouncement as to costs.

SO ORDERED.

||| (Republic v. Sayo, G.R. No. 60413, [October 31, 1990], 269 PHIL 74-80)
[G.R. No. 129401. February 2, 2001.]

FELIPE SEVILLE in his capacity as judicial administrator of


the estate of JOAQUIN ORTEGA and/or FELIPE SEVILLE,
EMILIA ESTRADA, MARIA S. TELLDER, MA. ISABEL SEVILLE,
MA. TERESITA LICARDO, FRANCISCO SEVILLE, RAMON
O. SEVILLE, JOSE MARIE SEVILLE, GEMMA ALVAREZ-
ASAYAS, ANNABELLE ALVAREZ-GONZALES, SYLVIA
ALVAREZ-LIOK, ADOLFO O. ALVAREZ JR., DIANA ALVAREZ-
DABON, MARIA SALVADOR O. POLANCOS and JOAQUIN
ORTEGA II as successors-in-interest of JOAQUIN ORTEGA
and his
estate, petitioners, vs. NATIONAL DEVELOPMENT COMPANY,
LEYTE SAB-A BASIN DEVELOPMENTAUTHORITY, PHILIPPINE
ASSOCIATED SMELTING AND REFINING CORPORATION,
LEPANTO-CONSOLIDATED MINING CO., PHILIPPINE
PHOSPHATE FERTILIZER CORPORATION, CALIXTRA YAP and
REGISTER OF DEEDS OF LEYTE, respondents.

Arthur D. Lim Law Office for petitioners.

Government Corporate Counsel for respondent NDC & LSDA.

Castillo Laman Tan Pantaleon & San Jose for respondent PASAR &
LEPANTO.

Quiros Dumas Capistrano & Teleron, Law Offices for PHILPHOS.

SYNOPSIS
Leyte Sab-A Basin Development Authority (LSBDA) was created to integrate
government and private sector efforts for a planned development and balanced
growth of the Sab-A Basin in the Province of Leyte, empowered to acquire real
property in the successful prosecution of its business.

Respondent Calixtra Yap sold to LSBDA Lot No. 057 SWO 08-000047
consisting of 464,920- sq. m. Original Certificate of Title was issued for the said
property in the name of respondent LSBDA in 1983.

In 1989, LSBDA assigned all its rights over the subject property to its co-
respondent National Development Company (NDC), as a result of which a new
Transfer Certificate of Title was issued in the name of NDC.

Meanwhile also in 1989, the Estate of Joaquin Ortega, represented by


administrator Felipe Seville filed with the Regional Trial Court an action for the
recovery of real property, rentals and damages against all the respondents
herein. The Estate claimed ownership by acquisitive prescription over the real
property acquired by LSBDA and later by NDC, containing a land area of
735,333-sq. m., which included that portion sold by Calixtra Yap.

The trial court rendered its decision in favor of the petitioners herein (Feliper
Seville) and against the respondents LSBDA. Respondents LSBDA herein
appealed their case to the Court of Appeals (CA), which in turn reversed the
decision of the trial court. The Court of Appeals citing the Regalian doctrine
ruled that the land in question, which did not appear to be privately owned,
should be presumed as part of the public domain. Hence, petitioners filed a
Petition for Review on Certiorari before the Supreme Court after the CA denied
their Motion for Reconsideration. The Court was called upon to determine the
validity of LSBDA's title. In resolving the issue, the Court has to rule on
whether the land in question was private or public before the issuance of title
thereof.

The Supreme Court ruled in favor of the respondents herein and affirmed the
ruling of the Court of Appeals. According to the Court there was no showing
that the land had been classified as alienable before the title was issued to
LSBDA; hence, petitioners could not be the owners thereof through acquisitive
prescription. Moreover, LSBDA's title was derived from a miscellaneous sales
patent, not from Yap. The Court ruled that the petitioner's challenge to
LSDBA's title cannot be granted since a certificate that had already became
indefeasible and incontrovertible cannot be challenged by a collateral attack.

SYLLABUS

1. POLITICAL LAW; LANDS OF THE PUBLIC DOMAIN; REGALIAN DOCTRINE;


DEFINED AND CONSTRUED; ACQUISITION OF IMPERFECT TITLE TO PUBLIC
LANDS; REQUISITES THEREOF; APPLICABLE ONLY TO ALIENABLE AND
DISPOSABLE LANDS OF THE PUBLIC DOMAIN. — Under the Regalian
doctrine, all the lands of the public domain belong to the State, which is the
source of any asserted right to ownership of land. All lands not otherwise
appearing to be clearly within private ownership are presumed to belong to the
State. In Menguito v. Republic, the court held that "[u]nless public land is
shown to have been reclassified or alienated to a private person by the State, it
remains part of the inalienable public domain. Indeed, 'occupation thereof in
the concept of owner, no matter how long, cannot ripen into ownership and be
registered as a title.' To overcome such presumption, incontrovertible evidence
must be shown by the applicant. Absent such evidence, the land sought to be
registered remains inalienable." A person in open, continuous, exclusive and
notorious possession of a public land for more than thirty years acquires an
imperfect title thereto. That title may be the subject of judicial confirmation,
pursuant to Section 48 of the Public Land Act, which provides: "SECTION 48.
The following described citizens of the Philippines, occupying lands of public
domain or claiming to own any such lands or an interest thereon, but whose
titles have not been perfected or completed, may apply to the Court of First
Instance of the province where the land is located for confirmation of their
claims, and the issuance of a certificate of title therefore, under the Land
Registration Act, to wit: . . . (b) those who by themselves or through their
predecessor-in-interest have been in open, continuous, exclusive and notorious
possession and occupation of agricultural lands of the public domain, under a
bona fide claim of acquisition or ownership, for at least thirty years
immediately preceding the filing of the application for confirmation of title
except when prevented by war orforce majeure. They shall be conclusively
presumed to have performed all the conditions essential to a Government grant
and shall be entitled to a certificate of title under the provisions of this
Chapter." Under Section 4 of Presidential Decree (PD) No. 1073, paragraph "b"
of the aforecited provision applies only to alienable and disposable lands of the
public domain. The provision reads: "SEC. 4. The provisions of Section 48 (b)
and Section 48 (c), Chapter VIII, of the Public Land Act, are hereby amended in
the sense that these provisions shall apply only to alienable and disposable
lands of the public domain which have been in open, continuous, exclusive and
notorious possession and occupation by the applicant himself or thru his
predecessor-in-interest, under a bona fide claim of acquisition of ownership,
since June 12, 1945." It should be stressed that petitioners had no certificate
of title over the disputed property. Although they claim that their title was
based on acquisitive prescription, they fail to present incontrovertible proof
that the land had previously been classified as alienable. They simply brush
aside the conclusion of the CA on this crucial point by saying that it was
"without factual basis." Instead, they maintain that the private character of the
land was evidenced by various tax declarations, Deeds of Sale, and Decisions of
the trial court and even the Supreme Court.

2. CIVIL LAW; LAND REGISTRATION; TAX DECLARATION; NOT CONCLUSIVE


PROOF OF LAND OWNERSHIP NOR THAT THE LAND IS PRIVATE;
APPLICATION IN CASE AT BAR. — Tax declarations are not conclusive proofs of
ownership, let alone of the private character of the land. At best, they are
merely "indicia of a claim of ownership." In Spouses Palomo vs. CA, the court
also rejected tax declarations as proof of private ownership, absent any
showing that the forest land in question had been reclassified as alienable.
Moreover, the Deeds of Sale of portions of the disputed property, which
Joaquin Ortega and several vendors executed, do not prove that the land was
private in character. The question remains: What was the character of the land
when Ortega purchased it? Indeed, a vendee acquires only those rights
belonging to the vendor. But petitioners failed to show that, at the time, the
vendors were already its owners, or that the land was already classified as
alienable.

3. ID.; ID.; BURDEN OF PROOF THAT THE LAND HAD BEEN CLASSIFIED AS
ALIENABLE IS ON THE CLAIMANT; ABSENCE THEREOF IN CASE AT BAR. —
Clearly, the burden of proof that the land has been classified as alienable is on
the claimant. In the present case, petitioners failed to discharge this burden.
Hence, their possession of the disputed property, however long, cannot ripen
into ownership.

4. ID.; ID.; WHEN CERTIFICATE OF TITLE OF LAND OF PUBLIC DOMAIN MAY


BE ISSUED; CASE AT BAR. — Equally unmeritorious is the argument of
petitioners that the title of LSBDA is void. As earlier stated, they claim that
such title was derived from Calixtra Yap, who was allegedly not the owner of
the property. Petitioners assume that LSBDA, having acquired the rights of
Yap, resorted to a confirmation of her imperfect title under Section 48 of the
Public Land Act. This argument is devoid of factual or legal basis. Petitioners
fail to consider that the title of LSBDA was based, not on the conveyance made
by Yap, but on Miscellaneous Sales Patent No. 9353 issued by the director of the
Bureau of Lands.In fact, after LSBDA had filed an application for patent, the
Bureau of Lands conducted an investigation and found that the land was part
of the public domain. After compliance with the notice and publication
requirements, LSBDA acquired the property in a public auction conducted by
the Bureau of Lands. In any case, the actions of LSBDA after Yap's conveyance
demonstrated its position that the disputed land was part of the public
domain. That this was so can be inferred from LSBDA's subsequent application
for a Miscellaneous Sales Patent and, in a public auction, its purchase of the
property from the Bureau of Lands. Indeed, Yap merely conveyed a claim, not a
title which she did not have.

5. ID.; ID.; CERTIFICATE OF TITLE; NATURE THEREOF; MAY BE


QUESTIONED ONLY IN A DIRECT PROCEEDING PERMITTED BY LAW. — It
has been held that a certificate of title, once registered, should not thereafter be
impugned, altered, changed, modified, enlarged or diminished, except in a
direct proceeding permitted by law. Otherwise, the reliance on registered titles
would be lost. Moreover, the title became indefeasible and incontrovertible after
the lapse of one year from the time of its registration and issuance. Section 32
of PD 1529 provides that "[u]pon the expiration of said period of one year, the
decree of registration and the certificate of title shall become incontrovertible.
Any person aggrieved by such decree of registration in any case may pursue his
remedy by action for damages against the applicant or other persons
responsible for the fraud." Although LSBDA's title was registered in 1983,
petitioners filed the amended Complaint only in 1990.

6. ID.;.ID.; ID.; RECONVEYANCE THEREOF; NOT ALLOWED IN CASE, AT


BAR. — Petitioners also claim that the disputed property should be reconveyed
to them. This cannot be allowed. Considering that the land was public before
the Miscellaneous Sales Patent was issued to LSBDA, petitioners have no
standing to ask for the reconveyance of the property to them. The proper
remedy is an action for reversion, which may be instituted only by the Office of
the Solicitor General, pursuant to Section 101 of the Public Land Act, which
reads as follows: "SEC. 101. All actions for the reversion to the Government of
lands of the public domain or improvements thereon shall be instituted by the
Solicitor General or the officer acting in his stead, in the proper courts, in the
name of the [Republic] of the Philippines." Verily, the prayer for reconveyance
and, for that matter, the entire case of petitioners rest on the theory that they
have acquired the property by acquisitive prescription; and that Yap, without
any right or authority, sold the same to LSBDA. TCcDaE

DECISION

PANGANIBAN, J p:

Unless a public land is shown to have been reclassified as alienable or actually


alienated by the State to a private person, that piece of land remains part of the
public domain. Hence, occupation thereof, however long, cannot ripen into
ownership.

The Case

Before us is a Petition for Review on Certiorari assailing the November 29, 1996
Decision of the Court of Appeals 1 (CA), as well as the May 19, 1997 CA
Resolution 2 denying the Motion for Reconsideration. The dispositive part of
the CA Decision reads as follows:

"WHEREFORE, the appealed decision is REVERSED and SET


ASIDE. Another judgment is hereby rendered dismissing the
complaint. 'The counterclaim of appellants are denied. Costs
against plaintiffs-appellees." 3

The Facts

The appellate court narrated the undisputed facts in this manner:

"1. By virtue of Presidential Decree No. 625, Leyte Sab-A


Basin Development Authority (LSBDA) was created to integrate
government and private sector efforts for a
planneddevelopment and balanced growth of the Sab-a Basin in
the [P]rovince of Leyte, empowered to acquire real property in the
successful prosecution of its business. Letter of Instruction No.
962 authorized LSBDA to acquire privately-owned lands
circumscribed in the Leyte Industrial Development Estate (LIDE)
by way of negotiated sales with the landowners.

"2. On June 14; 1980, [Respondent] Calixtra Yap sold to LSBDA


Lot No. 057 SWO 08-000047 consisting of 464,920 square
meters, located at Barangay Sto. Rosario, Isabel, Leyte, covered
under Tax Declarations Nos. 3181, 3579, 3425, 1292 and 4251
under the name of said vendor.

"3. On June 1, 1982, appellant LSBDA filed a Miscellaneous


Sales Application with the Bureau of Lands covering said lot
together with other lots acquired by LSBDA with an aggregate
area of '442,7508 square meters.'

"4. After due notice and investigation conducted by the Bureau of


Lands, Miscellaneous Sales Patent No. 9353 was issued in the
name of [Respondent] LSBDA on the basis of which Original
Certificate of Title No. P-28131 was transcribed in the
Registration Book for the [P]rovince of Leyte on August 12, 1983
in the name of [Respondent] LSBDA. On December 14, 1989,
LSBDA assigned all its rights over the subject property to its [Co-
respondent] National Development Company (NDC) as a result of
which a new Transfer Certificate of Title was issued on March 2,
1990 by the Registry of Deeds for the Province of Northern Leyte
in the name of NDC. The subject property was leased to
[Respondents] Philippine Associated Smelting & Refining
Corporation (PASAR), Philippine Phosphate Fertilizer Corporation
(PHILPHOS) and Lepanto Consolidated Mining Co., Inc.
(LEPANTO).

"5. On November 29, 1988, the Estate of Joaquin Ortega


represented by judicial administrator Felipe Seville filed with the
Regional Trial Court (Branch 12) of Ormoc City, a complaint for
recovery of real property, rentals and damages against the above-
named [respondents] which complaint was later on amended on
May 11, 1990. [Respondents] filed their respective Answers. After
trial, the trial court rendered judgment the dispositive portion of
which reads as follows:

'WHEREFORE, [a] decision is hereby rendered for


[petitioners] and against [respondents].

'1. The Deed of Sale executed by Calixtra Yap on June 14,


1980 in favor of LSBDA, (Exhibit PP and 25) conveying the
subject property to said LSBDA is declared NULL and VOID
ab initio;

'2. The intestate estate of JOAQUIN ORTEGA is declared


the owner in fee simple of the 735,333 square meters real
property subject of the present action and defendant NDC
is ordered to segregate the same area from OCT P-28131
and CONVEY the same to the Estate of Joaquin Ortega;

'3. Upon the segregation of the 735,333 square meters from


OCT No. P-28131 the Register of Deeds of the Province of
Leyte is ordered to issue a new title to the said portion in
the name of the Intestate Estate of Joaquin Ortega;

'4. [Respondents] LSBDA, NDC, PASAR, are ordered to pay


jointly and severally to [petitioners] the sum of FOUR
MILLION SEVEN HUNDRED EIGHTY FOUR THOUSAND
EIGHT HUNDRED FORTY SIX PESOS (P4,784,846.00) as
rentals due from 1979 to the present, plus accrued interest
pursuant to par. 2 of the Lease Contract between NDC and
PASAR. (Exhibit 54)
'5. [Respondents] LSBDA, NDC, and PHILPHOS are also
ordered to pay jointly and severally [petitioners] the sum of
TWO MILLION EIGHTY SIX THOUSAND THREE HUNDRED
NINETY EIGHT PESOS AND SIXTY CENTAVOS
(P2,086,398.60) as accrued rentals of PHILPHOS from 1979
to present, plus the accrued interest for non-payment
pursuant to paragraph 2 of the same Lease Contract cited
above;

'6. [Respondents] are ordered to pay jointly and severally


[petitioners] P200,000.00 as indemnity for the value of the
ancestral home;

'7. [Respondents] are also ordered to pay jointly and


severally [petitioners] the sum of P250,000.00 as
reimbursement for attorney's fees and the further sum of
P50,000.00 as expenses for litigation;

'8. Finally, [petitioners] and [respondents] are ordered to sit


down together and discuss the possibility of a compromise
agreement on how the improvements introduced on the
landholding subject of the present suit should be disposed
of and for the parties to submit to this Court a joint
manifestation relative thereto. In the absence of any such
compromise agreement, such improvements shall be
disposed of pursuant to Article 449 of the New Civil Code.

'Costs against [respondents].

'SO ORDERED.'" 4

Ruling of the Court of Appeals

Citing the Regalian doctrine that lands not appearing to be privately owned are
presumed to be part of the public domain, the CA held that, first, there was no
competent evidence to prove that the property in question was private in
character. Second, possession thereof, no matter how long, would not ripen
into ownership, absent any showing that the land had been classified as
alienable. Third, the property had been untitled before the issuance of the
Miscellaneous Sales Patent in favor of LSBDA. Fourth, petitioners were guilty of
laches, because they had failed to apply for the judicial confirmation of their
title, if they had any. Fifth, there was no evidence of bad faith on the part of
LSBDA in dealing with Yap regarding the property.

Hence, this Petition. 5

The Issues

In their Memorandum, petitioners submit the following issues for the


consideration of the Court: 6

"A. Whether or not the sale by Calixtra Yap of the Estate of the
Late Joaquin Ortega in favor of LSBDA was null and void.

"B. Whether or not the issuance of a Miscellaneous Sales Patent


and an Original Certificate of Title in favor of LSBDA was valid.

"C. Whether or not petitioners are guilty of laches.

"D. Whether or not petitioners are entitled to the remedy of


reconveyance and the damages awarded by the trial court."

In the main, the Court is called upon to determine the validity of LSBDA's title.
In resolving this issue, it will also ascertain whether, before the issuance of the
title, the land was private or public.

The Court's Ruling

The Petition has no merit.

Main Issue: Validity of LSBDA's Title

Petitioners argue that LSBDA's title to 73 hectares of the 402-hectare Leyte


Industrial Development Estate was void, having allegedly been obtained from
Calixtra Yap who had no right to it. They maintain that they acquired title to
the disputed property by acquisitive prescription, because they and their
predecessors in interest had been in possession of it for more than thirty
years. 7 Although it was the subject of settlement proceedings, petitioners
further claim that Yap sold the same to LSBDA without the permission of the
trial court.

Disputing these contentions, respondents and the appellate court maintain


that petitioners have not shown that the land had previously been classified as
alienable and disposable. Absent such classification, they argue that
possession of it, no matter how long, could not ripen into ownership.

We agree with respondents and the appellate court. First, there was no showing
that the land had been classified as alienable before the title was issued to
LSBDA; hence, petitioners could not have become owners thereof through
acquisitive prescription. Second, petitioners' challenge to LSBDA's title cannot
be granted, because it is based on a wrong premise and amounts to a collateral
attack, which is not allowed by law.

Public Character of the Land

Under the Regalian doctrine, all lands of the public domain belong to the State,
which is the source of any asserted right to ownership of land. All lands not
otherwise appearing to be clearly within private ownership are presumed to
belong to the State. 8 In Menguito v. Republic, 9 the Court held that "[u]nless
public land is shown to have been reclassified or alienated to a private person
by the State, it remains part of the inalienable public domain. Indeed,
'occupation thereof in the concept of owner, no matter how long, cannot ripen
into ownership and be registered as a title.' To overcome such presumption,
incontrovertible evidence must be shown by the applicant. Absent such
evidence, the land sought to be registered remains inalienable." CDEaAI
A person in open, continuous, exclusive and notorious possession of a public
land for more than thirty years acquires an imperfect title thereto. That title
may be the subject of judicial confirmation, pursuant to Section 48 of the
Public Land Act, which provides:

"SECTION 48. The following described citizens of the Philippines,


occupying lands of public domain or claiming to own any such
lands or an interest thereon, but whose titles have not been
perfected or completed, may apply to the Court of First Instance
of the province where the land is located for confirmation of their
claims, and the issuance of a certificate of title therefor, under
the Land Registration Act, to wit:

xxx xxx xxx

(b) those who by themselves or through their predecessor in-


interest have been in open, continuous, exclusive and notorious
possession and occupation of agricultural lands of the public
domain, under a bona fide claim of acquisition or ownership, for
at least thirty years immediately preceding the filing of the
application for confirmation of title except when prevented by war
or force majeure. They shall be conclusively presumed to have
performed all the conditions essential to a Government grant and
shall be entitled to a certificate of title under the provisions of
this Chapter."

Under Section 4 of Presidential Decree (PD) No. 1073, 10 paragraph "b" of the
aforecited provision applies only to alienable and disposable lands of the public
domain. The provision reads:

"SECTION 4. The provisions of Section 48 (b) and Section 48 (c),


Chapter VIII, of the Public Land Act, are hereby amended in the
sense that these provisions shall apply only to alienable and
disposable lands of the public domain which have been in open,
continuous, exclusive and notorious possession and occupation
by the applicant himself or thru his predecessor-in-interest,
under a bona fide claim of acquisition of ownership, since June
12, 1945."

It should be stressed that petitioners had no certificate of title over the


disputed property. Although they claim that their title was based on acquisitive
prescription, they fail to present incontrovertible proof that the land had
previously been classified as alienable. They simply brush aside the conclusion
of the CA on this crucial point by saying that it was "without factual
basis." 11 Instead, they maintain that the private character of the land was
evidenced by various tax declarations, Deeds of Sale, and Decisions of the trial
court and even the Supreme Court. 12

Petitioners' arguments are not convincing. Tax declarations are not conclusive
proofs of ownership, let alone of the private character of the land. At best, they
are merely "indicia of a claim of ownership." 13 In Spouses Palomo v.
CA, 14 the Court also rejected tax declarations as proof of private ownership,
absent any showing that the forest land in question had been reclassified as
alienable.

Moreover, the Deeds of Sale of portions of the disputed property, which


Joaquin Ortega and several vendors executed, do not prove that the land was
private in character. The question remains: What was the character of the land
when Ortega purchased it? Indeed, a vendee acquires only those rights
belonging to the vendor. But petitioners failed to show that, at the time, the
vendors were already its owners, or that the land was already classified as
alienable.

Also misplaced is petitioners' reliance on Ortega v. CA, 15 in which the


Supreme Court allegedly recognized the private character of the disputed
property. In that case, the sole issue was "whether the respondent judge . . .
acted in excess of jurisdiction when he converted Civil Case No. 1184-O, an
action for quieting of title, declaration of nullity of sale, and annulment of tax
declaration of a parcel of land, into an action for the declaration of who is the
legal wife, who are the legitimate children, if any, and who are the compulsory
heirs of the deceased Joaquin Ortega." 16 The Court did not at all make any
ruling that the property had been classified as alienable.

In any event, Ortegaarose from a suit for quieting of title, an action quasi in
rem that was binding only between the parties. 17 The present respondents as
well as the Bureau of Lands, which subsequently declared that the land was
public, are not bound by that ruling, because they were not impleaded therein.

While petitioners refer to the trial court proceedings supposedly recognizing the
private character of the disputed property, they make no claim that these cases
directly involve the classification of the land, or that the Bureau of Lands is a
party thereto.

Clearly, the burden of proof that the land has been classified as alienable is on
the claimant. 18 In the present case, petitioners failed to discharge this
burden. Hence, their possession of the disputed property, however long, cannot
ripen into ownership.

LSBDA's Title

Equally unmeritorious is the argument of petitioners that the title of LSBDA is


void. As earlier stated, they claim that such title was derived from Calixtra Yap,
who was allegedly not the owner of the property. Petitioners assume that
LSBDA, having acquired the rights of Yap, resorted to a confirmation of her
imperfect title under Section 48 of the Public Land Act. This argument is devoid
of factual or legal basis.

Petitioners fail to consider that the title of LSBDA was based, not on the
conveyance made by Yap, but on Miscellaneous Sales Patent No. 9353 issued by
the director of the Bureau of Lands. In fact, after LSBDA had filed an application
for patent, the Bureau of Lands conducted an investigation and found that the
land was part of the public domain. After compliance with the notice and
publication requirements, LSBDA acquired the property in a public auction
conducted by the Bureau of Lands. 19

Petitioners insist, however, that LSBDA was estopped from claiming that the
land was public, because the Deed of Sale executed by Yap in its favor
stipulated that "the seller is the absolute owner in fee simple of the . . .
described property." 20 It is scarcely necessary to address this point. To begin
with, the power to classify a land as alienable belongs to the State, not to
private entities. Hence, the pronouncements of Yap or LSBDA cannot effect the
reclassification of the property. Moreover, the assailed misrepresentation was
made by Yap as seller. Hence, objections thereto should be raised not by
petitioners but by LSBDA. the contracting party obviously aggrieved.

In any case, the actions of LSBDA after Yap's conveyance demonstrated its
position that the disputed land was part of the public domain. That this was so
can be inferred from LSBDA's subsequent application for a Miscellaneous Sales
Patent and, in a public auction, its purchase of the property from the Bureau
of Lands. Indeed, Yap merely conveyed a claim, not a title which she did not
have.

Collateral Attack

There is another reason for denying the present Petition. Petitioners insist that
they "are not seeking the re-opening of a decree under the Torrens system."
Supposedly, they are only "praying for the segregation of 735,333 square
meters of land, or 73 hectares more or less from the OCT No. P-28131 issued to
LSBDA." 21 This disputation is mere quibbling over words, plain and simple.

Semantics aside, petitioners are effectively seeking the modification of LSBDA's


OCT, which allegedly encompassed even a parcel of land allegedly belonging to
them. Hence, the present suit, purportedly filed for the "recovery of real
property and damages," is tantamount to a collateral attack not sanctioned by
law. Section 48 of PD 1529, the Property Registration Decree, expressly
provides:

"SECTION 48. Certificate not subject to collateral attack. — A


certificate of title shall not be subject to collateral attack. It
cannot be altered, modified, or cancelled except in a direct
proceeding in accordance with law."

It has been held that a certificate of title, once registered, should not thereafter
be impugned, altered, changed, modified, enlarged or diminished, except in a
direct proceeding permitted by law. Otherwise, the reliance on registered titles
would be lost. 22

Moreover, the title became indefeasible and incontrovertible after the lapse of
one year from the time of its registration and issuance. 23 Section 32 of PD
1529 provides that "[u]pon the expiration of said period of one year, the decree
of registration and the certificate of title shall become incontrovertible. Any
person aggrieved by such decree of registration in any case may pursue his
remedy by action for damages against the applicant or other persons
responsible for the fraud." Although LSBDA's title was registered in 1983,
petitioners filed the amended Complaint only in 1990.

Reconveyance

Petitioners also claim that the disputed property should be reconveyed to them.
This cannot be allowed. Considering that the land was public before the
Miscellaneous Sales Patent was issued to LSBDA, petitioners have no standing
to ask for the reconveyance of the property to them. The proper remedy is an
action for reversion, which may be instituted only by the Office of the Solicitor
General, pursuant to Section 101 of the Public Land Act, which reads as
follows:

"SECTION 101. All actions for the reversion to the Government of


lands of the public domain or improvements thereon shall be
instituted by the Solicitor General or the officer acting in his
stead, in the proper courts, in the name of the [Republic] of the
Philippines."

Verily, the prayer for reconveyance and, for that matter, the entire case of
petitioners rest on the theory that they have acquired the property by
acquisitive prescription; and that Yap, without any right or authority, sold the
same to LSBDA.

Conclusion

In the light of our earlier disquisition, the theory has no leg to stand on. Absent
any showing that the land has been classified as alienable, their possession
thereof, no matter how lengthy, cannot ripen into ownership. In other words,
they have not become owners of the disputed property. Moreover, LSBDA's title
was derived from a Miscellaneous Sales Patent, not from Yap. Finally,
petitioners cannot, by a collateral attack, challenge a certificate of title that has
already become indefeasible and incontrovertible.

If petitioners believe that they have been defrauded by Yap, they should seek
redress, not in these proceedings, but in a proper action in accordance with
law.

WHEREFORE, the Petition is hereby DENIED and the assailed Decision


AFFIRMED. Costs against petitioners.

SO ORDERED.

||| (Seville v. National Development Co., G.R. No. 129401, [February 2, 2001],
403 PHIL 843-860)
[G.R. No. 73246. March 2, 1993.]

DIRECTOR OF LANDS AND DIRECTOR OF FOREST


DEVELOPMENT, petitioners, vs. INTERMEDIATE APPELLATE
COURT AND J. ANTONIO ARANETA, respondents.

The Solicitor General for petitioners.

Jimenez, Leynes & Associates for private respondent.

SYLLABUS

1. LAND REGISTRATION; SUBMISSION OF TRACING CLOTH PLAN IS A


MANDATORY REQUIREMENT FOR REGISTRATION; SAME SHOULD BE
RETRIEVED FROM THE LAND REGISTRATION COMMISSION AND
SUBMITTED IN EVIDENCE. — In a very recent decision of this Court,
entitled The Director of Lands v. The Honorable Intermediate AppellateCourt and
Lino Anit (G.R. No. No. 65663, October 16, 1992), We have ruled that the
submission of the tracing cloth plan is a mandatory requirement for
registration. Reiterating Our ruling in Director of Lands v.
Reyes (68 SCRA 177), We asserted that failure to submit in evidence the
original tracing cloth plan is fatal it being a statutory requirement ofmandatory
character. It is of no import that petitioner failed to object to the
presentation of the certified copy of the said plan. What is required is the
original tracing cloth planof the land applied for and objection to such
requirement cannot be waived either expressly or impliedly. This case is no
different from the case of Director of Lands v. Reyes, supra wherein We said
that if the original tracing cloth plan was indeed with the Land Registration
Commission, there is no reason why the applicant cannot easily retrieve the
same and submit it in evidence, it being an essential requirement for
registration.
2. ID.; PROHIBITION AGAINST PRIVATE CORPORATIONS OR ASSOCIATIONS
ON HOLDING OF ALIENABLE LANDS OF THE PUBLIC DOMAIN; EXCEPTION.
— OurConstitution,whether the 1973 or 1987, prohibits private corporations or
associations from holding alienable lands of the public domain except by lease.
Apparently realizing such prohibition, respondent amended its application to
conform with the mandates of the law.

3. ID.; REPUBLICATION OF AN AMENDED APPLICATION, WHEN AND WHEN


NOT NECESSARY. — We cannot go along with petitioners' position that the
absence ofrepublication of an amended application for registration is a
jurisdictional flaw. We should distinguish. Amendments to the application may
be due to change in parties or substantial change in the boundaries or increase
in the area of the land applied for. In the former case, neither the Land
Registration Act, as amended, nor Presidential Decree No. 1529, otherwise
known as the Property Registration Decree, requires republication and
registration may be allowed by the court at any stage of the proceeding upon
just and reasonable terms. On the other hand, republication is required if the
amendment is due to substantial change in the boundaries or increase in the
area of the land applied for.

4. ID.; ACT NO. 496 AND P.D. 1529 ARE EXISTING LAWS AND CAN STAND
TOGETHER; PURPOSE OF P.D. 1529. — We do not see any relevant dispute in
the lower court's application of Presidential Decree No. 1529, instead of Act No.
496, in adjudicating the land to the then applicant, assuming that the land
involved is registrable. Both laws are existing and can stand together. P.D.
1529 was enacted to codify the various laws relative to registration of property,
in order to facilitate effective implementation of said laws.

5. ID.; CLASSIFICATION OF LANDS OF THE PUBLIC DOMAIN; IN WHOM


AUTHORITY TO CLASSIFY IS VESTED; EFFECT OF CLASSIFICATION;
CAUSE OF CLASSIFICATION — STATEMENT OF THE REGALIAN DOCTRINE;
EFFECT OF LACK OF CLASSIFICATION. — Lands of the public domain are
classified under three main categories, namely: Mineral, Forest and Disposable
or Alienable Lands. Under the Commonwealth Constitution,only
agricultural lands were allowed to be alienated. Their disposition was provided
for underCommonwealth Act No. 141 (Secs. 6-7), which states that it is only
the President, upon the recommendation of the proper department head, who
has the authority to classify thelands of the public domain into alienable or
disposable, timber and mineral lands. Mineral and Timber or forest lands are
not subject to private ownership unless they are first reclassified as
agricultural lands and so released for alienation. In the absence of such
classification, the land remains as unclassified land until released therefrom
and rendered open to disposition. Courts have no authority to do so. This is in
consonance with the Regalian doctrine that all lands of the public domain
belong to the State, and that the State is the source of any asserted right to
ownership in land and charged with the conservation of such patrimony. Under
the Regalian Doctrine, all lands not otherwise appearing to be clearly within
private ownership are presumed to belong to the State. Hence, a positive
act of the government is needed to declassify a forest land into alienable or
disposable land for agricultural or other purposes.

6. ID.; POSSESSION, HOWEVER LONG, AND CONVERSION OF AN


UNCLASSIFIED PUBLIC LAND CANNOT RIPEN INTO PRIVATE OWNERSHIP. —
Respondent even admitted that Tambac Island is still an unclassified public
land as of 1927 and remains to be unclassified. Since the subject property is
still unclassified, whatever possession the applicant may have had and
however long, cannot ripen into private ownership. The conversion of subject
property does not automatically render the property as alienable and
disposable.

7. ID.; BURDEN OF PROOF AND EVIDENCE REQUIRED IN APPLICATION FOR


REGISTRATION OF LAND. — The burden of proof in overcoming the
presumption of state ownership of the lands of the public domain is on the
person applying for registration that the land subject of the application is
alienable or disposable. Unless the applicant succeeds in showing by
convincing evidence that the property involved was acquired by him or his
ancestors either by composition title from the Spanish Government or by
possessory information title, or any other means for the proper
acquisition of public lands, the property must be held to be part of the public
domain. The applicant must present evidence and persuasive proof to
substantiate his claim.

8. ID.; EVIDENTIARY WEIGHT OF TAX DECLARATIONS AND RECEIPTS. — In


any case tax declarations and receipts are not conclusive evidence of ownership
or of the right to possess land when not supported by evidence. The fact that
the disputed property may have been declared for taxation purposes in the
names of the applicants or of their predecessors-in-interest way back in 1921
does not necessarily prove ownership. They are merely indicia of a
claim of ownership.

DECISION

NOCON, J p:

For review before Us is the decision of the Court of Appeals in the land
registration case entitled J. Antonio Araneta v.
The Director of Lands and Director of Forest Development, AC-G.R. CV No.
00636, 1 affirming the lower court's approval of the application for
registration of a parcel of land in favor of applicant therein, J. Antonio Araneta.

Evidence show that the land involved is actually an island known as Tambac
Island in Lingayen Gulf. Situated in the Municipality of Bani, Pangasinan, the
area consists of 187,288 square meters, more or less. The initial application for
registration was filed for Pacific Farms, Inc. under the provisions of the Land
Registration Act, Act No. 496, as amended. prcd
The Republic of the Philippines, thru the Director of Lands opposed the
application alleging that the applicant, Pacific Farms, Inc. does not possess a
fee simple title to the land nor did its predecessors possess the land for at least
thirty (30) years immediately preceding the filing of application. The opposition
likewise specifically alleged that the applicant is a private corporation
disqualified under the (1973) new Philippine Constitution from acquiring
alienable lands of the public domain citing Section 11, Article 14. 2

The Director of Forest Development also entered its opposition alleging that the
land is within the unclassified public land and, hence, inalienable. Other
private parties also filed their oppositions, but were subsequently withdrawn.

In an amended application, Pacific Farms, Inc. filed a manifestation-motion to


change the applicant from Pacific Farms, Inc. to J. Antonio Araneta. Despite the
supposed amendment, there was no republication.

Evidence presented by the applicant include the testimony of Placido Orlando,


fishery guard of Pacific Farms, Inc., who said that he has known the disputed
land since he attained the age of reason for some forty (40) years now; that
when he first came to know the property it was then owned by and in the
possession of Paulino Castelo, Juan Ambrosio and Julio Castelo, and later on
the whole island was bought by Atty. Vicente Castelo who in turn sold it to J.
Antonio Araneta.

Deposition by oral examination of Araneta was also presented, together with


documents of sale, tax declarations and receipts, and survey of property.
Applicant, however, failed to present the tracing cloth plan and instead
submitted to the court certified copies thereof.

While this case is pending here in Court, respondent filed an Omnibus Motion
for Substitution of private respondent. 3 Apparently, Antonio Araneta had
assigned his rights to and interest in Tambac Island to Amancio R.
Garcia 4 who in turn assigned his rights and interest in the same property
to Johnny A. Khonghun whose nationality was not alleged in the
pleadings.

On October 4, 1979, the trial court rendered a decision adjudicating the


subject property to J. Antonio Araneta. On appeal to the
then Intermediate Appellate Court, the decisionof the lower court was affirmed
on December 12, 1985.

Petitioners raised the following errors:

I. The lower court erred in adjudicating


the lands subject of registration to applicant-appellee despite his
failure to present the original tracing cloth plan the
submission ofwhich is a statutory requirement of mandatory
character.

II. The lower court erred in not denying registration in favor of J.


Antonio Araneta since the amendment of the application was
simply an attempt to avoid the application ofthe constitutional
provision disqualifying a private corporation — the Pacific Farms,
Inc. in this case — from acquiring lands of public domain. cdrep

III. The lower court erred in declaring the land known as the
"Tambac Island" subject of registration it being an island formed
on the seas.

IV. The lower court erred in adjudicating the land to the applicant
under the provisions of Presidential Decree No. 1529, otherwise
known as the Property Registration Decree, despite
absence of any specific invocation of this law in the original and
amended application.

V. The lower court erred in not granting the government's motion


for reconsideration at least to enable it to present proof of the
status of the land as within the unclassified public forest, and
hence beyond the court's jurisdiction to adjudicate as private
property.

VI. The lower court erred in declaring that the applicant has
overthrown the presumption that the land is a portion of the
public domain belonging to the Republicof the Philippines.

From the foregoing it appears that the more important issues are:

1) whether the presentation of the tracing cloth plan is necessary; and 2)


whether the land known as "Tambac Island" can be subject to registration.

By mere consideration of the first assignment of error, We can right away glean
the merit of the petition.

Respondent claims that the tracing cloth plan is with the files of the Land
Registration Commission, and the only evidence that can be presented to that
fact is the request for the issuance of a certified copy thereof and the certified
copy issued pursuant to the request. 5 Respondent further argues that
failure of the petitioners to object to the presentationof the certified copy of the
tracing cloth plan was the basis of the trial court's denial of petitioner's motion
for reconsideration.

In a very recent decision of this Court, entitled The Director of Lands v. The
Honorable Intermediate Appellate Court and Lino Anit, 6 We have ruled that the
submission of the tracing cloth plan is a mandatory requirement for
registration. Reiterating Our ruling in Director of Lands v. Reyes, 7 We asserted
that failure to submit in evidence the original tracing cloth plan is fatal it being
a statutory requirement of mandatory character.

It is of no import that petitioner failed to object to the presentation of the


certified copy of the said plan. What is required is the original tracing cloth
plan of the land applied for and objection to such requirement cannot be
waived either expressly or impliedly. 8 This case is no different from the
case of Director of Lands v. Reyes, supra wherein We said that if the original
tracing cloth plan was indeed with the Land Registration Commission, there is
no reason why the applicant cannot easily retrieve the same and submit it in
evidence, it being an essential requirement for registration.

As to the second assignment of error, We are inclined to agree with petitioners


that the amendment of the application from the name of Pacific Farms, Inc., as
applicant, to the name of J. Antonio Araneta, was a mere attempt to evade
disqualification. Our Constitution,whether the 1973 9 or 1987, 10 prohibits
private corporations or associations from holding alienable lands of the public
domain except by lease. Apparently realizing such prohibition, respondent
amended its application to conform with the mandates of the law. cdphil

However, We cannot go along with petitioners' position that the


absence of republication of an amended application for registration is a
jurisdictional flaw. We should distinguish. Amendments to the application may
be due to change in parties or substantial change in the boundaries or increase
in the area of the land applied for.

In the former case, neither the Land Registration Act, as amended,


nor Presidential Decree No. 1529, otherwise known as the Property Registration
Decree, requires republication and registration may be allowed by the court at
any stage of the proceeding upon just and reasonable terms. 11 On the other
hand, republication is required if the amendment is due to substantial change
in the boundaries or increase in the area of the land applied for.

As to the fourth assignment of error, We do not see any relevant dispute in the
lower court's application of Presidential Decree No. 1529, instead of Act No.
496, in adjudicating the land to the then applicant, assuming that the land
involved is registrable. Both laws are existing and can stand together. P.D.
1529 was enacted to codify the various laws relative to registration of property,
in order to facilitate effective implementation of said laws. 12
The third, fifth and sixth assignment of errors are likewise meritorious and
shall be discussed forthwith together.

Respondent asserts that contrary to the allegation of petitioners, the


reports of the District Land Officer of Dagupan City, Land Inspector Perfecto
Daroy and Supervising Land Examiner Teodoro P. Nieva show that the subject
property is an unclassified public land, not forest land. This claim is rather
misleading. The report of Supervising Land Examiner Nieva specifically states
that the "land is within the unclassified forest land" under the administrative
jurisdiction of the then Bureau of Forest Development. 13 This was based on
the reports of Land Inspector Daroy and District Land Officer Feliciano
Liggayu.

Lands of the public domain are classified under three main categories, namely:
Mineral, Forest and Disposable or Alienable Lands. 14 Under the
Commonwealth Constitution,only agricultural lands were allowed to be
alienated. Their disposition was provided for under Commonwealth Act No. 141
(Secs. 6-7), which states that it is only the President, upon the
recommendation of the proper department head, who has the authority to
classify the lands of the public domain into alienable or disposable, timber and
mineral lands. Mineral and Timber or forest lands are not subject to private
ownership unless they are first reclassified as agricultural lands and so
released for alienation. 15 In the absence ofsuch classification, the land
remains as unclassified land until released therefrom and rendered open to
disposition. Courts have no authority to do so. 16

This is in consonance with the Regalian doctrine that all lands of the public
domain belong to the State, and that the State is the source of any asserted
right to ownership in land and charged with the conservation of such
patrimony. Under the Regalian Doctrine, all lands not otherwise appearing to
be clearly within private ownership are presumed to belong to the State. Hence,
a positive act of the government is needed to declassify a forest land into
alienable or disposable land for agricultural or other purposes. 17

The burden of proof in overcoming the presumption of state


ownership of the lands of the public domain is on the person applying for
registration that the land subject of the application is alienable or
disposable. 18

Unless the applicant succeeds in showing by convincing evidence that the


property involved was acquired by him or his ancestors either by composition
title from the Spanish Government or by possessory information title, or any
other means for the proper acquisition of public lands, the property must be
held to be part of the public domain. The applicant must present evidence and
persuasive proof to substantiate his claim. 19

In this particular case, respondent presented proof that as early as 1921, the
subject property has been declared for tax purposes with receipts attached, in
the names ofrespondent's predecessors-in-interest. Nevertheless, in that
span of time there had been no attempt to register the same either under Act
496 or under the Spanish Mortgage Law. It is also rather intriguing that
Vicente Castelo who acquired almost 90% of the property from Alejo Ambrosio,
et al. on June 18, 1958 and from Julio Castelo on June 19, 1958 immediately
sold the same to applicant J. Antonio Araneta on 3 July 1958. LexLib

According to the report of Land Investigator Daroy, the land was declared for
taxation purposes in the name of Vicente Castelo only in 1958 and the
purported old tax declarations are not on file with the Provincial Assessor's
Office.

In any case tax declarations and receipts are not conclusive


evidence of ownership or of the right to possess land when not supported by
evidence. 20 The fact that the disputed property may have been declared for
taxation purposes in the names of the applicants or of their predecessors-in-
interest way back in 1921 does not necessarily prove ownership. They are
merely indicia of a claim of ownership. 21

Respondent's contention that the BFD, LC Map No. 681, certified on August 8,
1927 which was the basis of the report and recommendation of the Land
Examiner, is too antiquated; that it cannot be conclusively relied upon and was
not even presented in evidence, is not well taken. As We have said in the
case of Director of Lands v. CA: 22

"And the fact that BF Map LC No. 673 dated March 1, 1927
showing subject property to be within unclassified region was not
presented in evidence will not operate against the State
considering the stipulation between the parties and under the
well-settled rule that the State cannot be estopped by the
omission, mistake or error of its officials or agents, if omission
there was, in fact."

Respondent even admitted that Tambac Island is still an unclassified public


land as of 1927 and remains to be unclassified.

Since the subject property is still unclassified, whatever possession the


applicant may have had and however long, cannot ripen into private
ownership. 23 The conversion ofsubject property does not automatically render
the property as alienable and disposable.

In effect what the courts a quo have done is to release the subject property
from the unclassified category, which is beyond their competence and
jurisdiction. We reiterate that the classification of public lands is an exclusive
prerogative of the Executive Department of the Government and not of the
Courts. In the absence of such classification, the land remains unclassified
until released therefrom and rendered open to disposition. 24
In fairness to respondent, the petitioners should seriously consider the
matter of the reclassification of the land in question. The attempt of people to
have disposable lands they have been tilling for generations titled in their name
should not only be viewed with understanding attitude, but as a
matter of policy encouraged. 25

WHEREFORE, the petition is hereby GRANTED and the decisions of the


courts a quo are REVERSED.

SO ORDERED.

||| (Director of Lands v. Intermediate Appellate Court, G.R. No. 73246, [March 2,
1993])
[G.R. No. 63786-87. April 7, 1993.]

UNITED PARACALE MINING COMPANY, INC., AND COCO


GROVE, INC., petitioners, vs. HON. JOSELITO DELA ROSA, in
his capacity as the former Judge of the Court of First
Instance of Camarines Norte, Branch 2, et al., respondents.

[G.R. No. 70423. April 7, 1993.]

ZAMBALES CHROMITE MINING COMPANY,


INC., petitioner, vs. HON. ALFREDO L. BENIPAYO, as Judge,
Regional Trial Court of Manila, and PHILZEA MINING &
DEVELOPMENT CORPORATION, respondents.

[G.R. No. 73931. April 7, 1993.]

JOSEPH V. LOPEZ and MIGUEL C. ANDRADE, petitioners, vs.


THE INTERMEDIATE APPELLATE COURT MARSMAN &
COMPANY, INC. and UNITED PARACALEMINING COMPANY,
INC., respondents.

Sycip, Salazar, Hernandez and Gatmaitan for United Paracale and Coco
Grove, Inc.

Pedro A. Venida for respondents in G.R. 63786-87 and petitioners in G.R.


73931.

J.V. Natividad & Associates for Zambales Chromite.

SYLLABUS
1. STATUTORY CONSTRUCTION; INTERPRETATION OF A STATUTE; PROPER
ONLY WHEN THERE IS DOUBT OR AMBIGUITY IN ITS LANGUAGE; CASE AT
BAR. — The view of the petitioner that by virtue of the registration of the
mining claims under the Philippine Bill of 1902 and Act No. 624, the mining
claims became private property and thereby brought outside the control and
supervision of the Director of Mines is without legal basis. The abovecited law
does not distinguish between private property and lands of the public domain.
The provision of law involved is clear and is not susceptible to interpretation. A
condition sine qua non before the court may construe or interpret a statute is
that there be doubt or ambiguity in its language. Section 7 of P.D. 1281 quoted
above defining the original and exclusive jurisdiction of the Director of Mines is
clear. Time and again, it has been repeatedly declared by this Court that where
the law speaks in clear and categorical language, there is no room for
interpretation. There is only room for application. [Cebu Portland Cement
Company vs. Municipality of Naga, Cebu, 35 SCRA 708 (1968)] Where the law
is clear and unambiguous, it must be taken to mean exactly what it says and
the court has no choice but to see to it that its mandate is obeyed. [Chartered
Bank Employees Association vs. Ople, 138 SCRA 273 (1985)].

2. REMEDIAL LAW; CIVIL PROCEDURE; JUDICIAL RELIEF; NOT A VESTED


RIGHT; A MERE STATUTORY PRIVILEGE, NOT A PROPERTY RIGHT. — There
can be no vested right in a judicial relief for this is a mere statutory privilege
and not a property right. The distinction between statutory privileges and
vested rights must be borne in mind for persons have no vested rights in
statutory privileges. The state may change or take away rights which were
created by the law of the state, although it may not take away property which
was vested by virtue of such rights.(16A Am. Jur. 2d, pp. 652-653) Besides, the
right to judicial relief is not a right which may constitute vested right because
to be vested, a right must have become a title, legal or equitable, to the present
or future enjoyment of property, or to the present or future enforcement of a
demand or legal exemption from a demand made by another. (National
Carloading Corporation v. Phoenix-E1 Paso Express, Inc., cited in 16A Am, Jur.
2d, p. 651) Definitely, the judicial relief herein referred to by the petitioner does
not fall under any of these.

3. CIVIL LAW; PRESIDENTIAL DECREE 1214; A VALID EXERCISE OF THE


SOVEREIGN POWER OF THE STATE AS OWNER OF LAND OF PUBLIC
DOMAIN; SUSTAINED IN CASE AT BAR. — The heart of these twin petitions is
the question of constitutionality of P.D. 1214. Unless P.D. 1214 is successfully
assailed, the petitioners will be but mere applicants for the lease of the mining
claims involved and would thus have no causes of action against private
respondents. This question has been resolved by this Court in Santa Rosa
Mining Company, Inc. vs. Leido, Jr. [156 SCRA 1 (1987), which ruling was
reiterated in Zambales Chromite Mining Company, Inc. vs. Leido, Jr., 176
SCRA 602 (1989)] thus: "(W)e hold thatPresidential Decree No. 1214 is not
unconstitutional.' It is a valid exercise of the sovereign power of the State, as
owner, over lands of the public domain, of which petitioner's mining claims still
form a part, and over the patrimony of the nation, of which mineral deposits
are a valuable asset. It may be underscored, in this connection, that the Decree
does not cover all mining claims located under the Phil. Bill of 1902, but only
those claims over which their locators had failed to obtain a patent. And even
then, such locators may still avail of the renewable twenty-five year (25) lease
prescribed by Pres. Decree No. 463, the Mineral Development Resources Decree
of 1974. Mere location does not mean absolute ownership over the affected
land or the mining claim. It merely segregates the located land or area from the
public domain by barring other would-be locators from locating the same and
appropriating for themselves the minerals found therein. To rule otherwise
would imply that location is all that is needed to acquire and maintain rights
over a located mining claim. This, we cannot approve or sanction because it is
contrary to the intention of the lawmaker that the locator should faithfully and
consistently comply with the requirements for annual work and improvements
in the located mining claim. This, we cannot approve or sanction because it is
contrary to the intention of the lawmaker that the locator should faithfully and
consistently comply with the requirements for annual work and improvements
in the located mining claim. Presidential Decree No. 1214 is in accord with Sec.
8, Art. XIV of the 1973 Constitution. The same Constitutional mandate is
found in Sec. 2, Art. XII of the 1987 Constitution.

DECISION

CAMPOS, JR., J p:

The cases herein were consolidated upon the representations of petitioners that
they involve the same issues or questions of law or at the very least, revolve
around the same set of facts. A perusal of the records, however, reveals the
contrary. Only two petitions are properly consolidated. Thus, it behooves Us to
discuss the cases separately.

In blatant violation of Section 2, Rule 45 of the Rules of Court which in part,


provides:

"Sec. 2. Contents of the petition — The petition shall contain a


concise statement of the matters involved, the assignment of
errors made in the court below, and the reasons relied on for the
allowance of the petition, . . . (Emphasis Ours.).

"Only questions of law may be raised in the petition and must be


distinctly set forth . . ."

this petition devotes nine (9) pages under the subtitle "Summary statement
of the matters involved" to a discussion of matters off tangent from the real
issues in the case. Definitely, the question of whether or not the Court of
Appeals erred in ruling that the Regional Trial Court did not commit grave
abuse of discretion in issuing an order suspending hearing pending the
resolution of their motion to dismiss, does not involve the Philippine Bill of
1902, Executive Order No. 141, much less P.D. 1214. The counsel for
petitioners even discussed pending cases in this Court (G.R. No. 63786 - 87
and 69203) which have completely nothing to do with the instant petition
except for the fact that the parties therein are being represented by the
same counsel as in this petition. In several pleadings subsequent to their
petition, petitioners insisted that the proceedings in the court below must be
restrained until this Court resolves the pending cases abovecited. For this
reason this case was consolidated thereto.

A summary of the real matters involved in this petition is found in the


respondent Court's decision, to wit:

"This is a petition for certiorari and prohibition to enjoin the


Regional Trial Court, Branch XL, in Camarines Norte from
issuing a writ of preliminary injunction in Civil Case No. 5148
and to disqualify the respondent judge from acting in that case.
The case was brought by the respondents Marsman and Co., Inc.
and United Paracale Mining, Inc., to enjoin the petitioners,
Joseph V. Lopez and Miguel Andrade, from entering and
conducting mining operations within the "McDonald" and "San
Antonio" Tunnels in Paracale, Camarines Norte, in which the
private respondents have mining claims ("Tulisan," "Santa
Marta," "California," and "Rocky Mountain Fraction"). On
December 11, 1984 the RTC issued a restraining order against
the petitioners. cdphil

On December 12 the petitioners filed their answer alleging that,


in accordance with PD 1214, the private respondents had
forfeited their right to the mining claims. They likewise argued
that in view of PD 605, the RTC had no jurisdiction to entertain
the case. On the same day the petitioners moved for the
disqualification of the respondent judge of the RTC, claiming (sic)
that in issuing the restraining order of December 11, 1984, he
showed his "bias, prejudice and personal hatred of and hostility
to the [petitioners'] counsel [Atty. Pedro A. Venida]."

On December 24, 1984 the petitioners filed a motion for a


preliminary hearing on their defense that the RTC lacked
jurisdiction under PD 605 to issue a temporary restraining order
or injunction in cases involving or growing out of the action of
administrative officials on the applications for permits for the
disposition, exploitation, utilization, or exploration or
development of the natural resources. Accordingly the RTC, in its
order of September 5, 1985, suspended the hearing of the case
until the resolution of the petitioners' motion to dismiss. It is at
this point that the present petition was filed." 1

The respondent Court denied this petition on grounds that: (1) the questions
being raised are not proper in a petition for certiorari under Rule 65 but rather
defenses which should be raised in the action itself; (2) the question of
jurisdiction which has yet to be resolved by the trial court pending resolution of
the motion to dismiss is prematurely raised; and (3) there was no basis for
determining whether or not the judge must be disqualified. 2

The review of this decision is what is on appeal before.

We refuse to be persuaded by the petitioners that the RTC must be enjoined


from exercising its jurisdiction in settling the case presented before it for the
reason that the constitutionality of the law involved in the said case is being
questioned before this Court. This case should have been disposed of
independently of the other petitions herein.

The respondent Court of Appeals committed no reversible error. Neither did it


commit grave abuse of discretion as what petitioners want this Court to
believe. The petitioners fail to point out any assigned error which the
respondent Court had supposedly committed but simply narrate the action
taken by it. Much less have they stated the reasons relied upon for the
allowance of the instant petition. For being insufficient in substance and in
form, the instant petition lacks merit and must be dismissed.

G.R. No. 70423

This is a petition involving the question of jurisdiction of regular courts in


cases which had been placed under the original and exclusive jurisdiction of
the Bureau of Mines underP.D. 1281.

This petition seeks to reverse the order of then Judge, now Associate Justice of
the Court of Appeals, Hon. Alfredo L. Benipayo, dismissing the complaint filed
by petitioner herein on the ground of lack of jurisdiction, citing Section 7
of P.D. 1281 and the doctrine enunciated in Twin Peaks Mining Association, et
al. vs. Navarro, 3 that an action for the enforcement of mining contracts, in this
case cancellation of a mining contract, is outside the competence of regular
courts in view of the law cited. 4

The complaint filed with the then CFI of Manila, Branch XVI, was one for the
rescission of its mining contract with herein private respondent on grounds of
violations of the terms and conditions thereof, with prayer for the issuance of a
preliminary injunction and/or temporary restraining order. The trial court,
however, upon motion of the defendant therein, dismissed the case.

Petitioner wants Us to construe Section 7 of P.D. 1281 as applicable only to


mineral lands forming part of the public domain and not to mining claims
located and registered under Philippine Bill of 1902 and Act No. 624 as is its
case.

Section 7 of P.D. 1281 reads as follows:

Sec. 7. In addition to its regulatory and adjudicative functions


over companies, partnerships or persons engaged in mining
exploration, development and exploitation, the Bureau of Mines
shall have original and exclusive jurisdiction to hear and decide
cases involving:

(a) a mining property subject of different agreements


entered into by the claim holder thereof with several mining
operators;

(b) complaints from claimowners that the mining


property subject of an operating agreement has not been
placed into actual operations within the period stipulated
therein; and

(c) cancellation and/or reinforcement of mining


contracts due to the refusal of the claimowner/operator to
aside by the terms and conditions thereof.

All actions and decisions of the Director of Mines on the above


cases are subject to review, motu proprio or upon appeal by any
person aggrieved thereby, by the Secretary of Natural Resources
whose decision shall be final and executory after the lapse of
thirty (30) days from receipt by the aggrieved party of said
decision, unless appealed to the President in accordance with the
applicable provisions of Presidential Decree No. 309 and Letter of
Instructions Nos. 119 and 135. LLpr

The view of the petitioner that by virtue of the registration of the mining claims
under the Philippine Bill of 1902 and Act No. 624, the mining claims became
private property and thereby brought outside the control and supervision of the
Director of Mines is without legal basis. The abovecited law does not
distinguish between private property and lands of the public domain. The
provision of law involved is clear and is not susceptible to interpretation. A
condition sine qua non before the court may construe or interpret a statute is
that there be doubt or ambiguity in its language. 5 Section 7 of P.D.
1281 quoted above defining the original and exclusive jurisdiction of the
Director of Mines is clear. Time and again, it has been repeatedly declared by
this Court that where the law speaks in clear and categorical language, there is
no room for interpretation. There is only room for application. 6 Where the law
is clear and unambiguous, it must be taken to mean exactly what it says and
the court has no choice but to see to it that its mandate is obeyed. 7

This Court in Benguet Corporation vs. Leviste, 8 made these pronouncements:

"We grant the petition. Presidential Decree No. 1281 which took
effect on January 16, 1978 vests the Bureau of Mines with
jurisdictional supervision and control over all holders of mining
claims or applicants for and/or grantees of mining licenses,
permits, leases and/or operators thereof, including mining
service contracts and service contractors insofar as their mining
activities are concerned. To effectively discharge its task as the
Government's arm in the administration and disposition of
mineral resources, Section 7 of P.D. 1281 confers upon the
Bureau quasi-judicial powers as follows:

xxx xxx xxx

Analyzing the objective of P.D. 1281, particularly said Section 7


thereof, the Court in Twin Peaks Mining Association, the case
relied upon by petitioner, noted that the trend is to make the
adjudication of mining cases a purely administrative matter. This
observation was reiterated in the more recent case of Atlas
Consolidated Mining & Development Corporation vs. Court of
Appeals."

The petitioner further argues that to hold that P.D. 1281 retroactively applies
to its mining claims which according to it is private property would constitute
impairment of vested rights since by shifting the forum of the petitioner's case
from the courts to the Bureau of Mines, as urged by private respondent, the
substantive rights to full protection of its property rights shall be greatly
impaired and prejudiced. The judicial relief available for the redress of private
property rights violated, now being enjoyed by petitioner shall be lost
altogether.

This argument does not merit Our approval. There can be no vested right in a
judicial relief for this is a mere statutory privilege and not a property right. The
distinction between statutory privileges and vested rights must be borne in
mind for persons have no vested rights in statutory privileges. The state may
change or take away rights which were created by the law of the state,
although it may not take away property which was vested by virtue of such
rights. 9 Besides, the right to judicial relief is not a right which may constitute
vested right because to be vested, a right must have become a title, legal or
equitable, to the present or future enjoyment of property, or to the present or
future enforcement of a demand or legal exemption from a demand made by
another. 10 Definitely, the judicial relief herein referred to by the petitioner
does not fall under any of these.

The case at bar falls within the original and exclusive jurisdiction of the Bureau
of Mines, hence, the trial court did not err in dismissing the petitioner's
complaint on the ground of lack of jurisdiction.

G.R. Nos. 63786-87

In these petitions filed by petitioners United Paracale Mining Company, Inc.


and Coco Grove, Inc., petitioners seek to set aside the Order of dismissal of the
case they filed with the trial court for the ejectment of their respective
defendants from the mining claims which were allegedly privately owned by
them having been located and perfected under the provisions of the Philippine
Bill of 1902 and Act No. 624.

The heart of these twin petitions is the question of constitutionality of P.D.


1214. Unless P.D. 1214 is successfully assailed, the petitioners will be but
mere applicants for the lease of the mining claims involved and would thus
have no causes of action against private respondents.

This question has been resolved by this Court in Santa Rosa Mining Company,
Inc. vs. Leido, Jr. 11 thus:

"(W)e hold that Presidential Decree No. 1214 is not


unconstitutional. ** It is a valid exercise of the sovereign power of
the State, as owner, over lands of the public domain, of which
petitioner's mining claims still form a part, and over the
patrimony of the nation, of which mineral deposits are a valuable
asset. It may be underscored, in this connection, that the Decree
does not cover all mining claims located under the Phil. Bill of
1902, but only those claims over which their locators had failed to
obtain a patent. And even then, such locators may still avail of
the renewable twenty-five year (25) lease prescribed by Pres.
Decree No. 463, the Mineral Development Resources Decree of
1974.

Mere location does not mean absolute ownership over the


affected land or the mining claim. It merely segregates the located
land or area from the public domain by barring other would-be
locators from locating the same and appropriating for themselves
the minerals found therein. To rule otherwise would imply that
location is all that is needed toacquire and maintain rights over a
located mining claim. This, we cannot approve or sanction
because it is contrary to the intention of the lawmaker that the
locator should faithfully and consistently comply with the
requirements for annual work and improvements in the located
mining claim.

Presidential Decree No. 1214 is in accord with Sec. 8, Art. XIV of


the 1973 Constitution which states:
'All lands of the public domain, waters, minerals,
coal, petroleum, and other mineral oils, all forces of
potential energy, fisheries, wildlife, and other natural
resources of the Philippines belong to the State. With the
exception of agricultural, industrial or commercial,
residential and resettlement lands of the public domain,
natural resources shall not be alienated, and no license,
concession, or lease for the exploration, development, and
exploitation, or utilization of any of the natural resources
shall be granted for a period exceeding twenty-five years,
renewable for not more than twenty-five years, except as to
water rights for irrigation, water supply, fisheries, or
industrial uses other than development of water power, in
which cases, beneficial use may be the measure and the
limit of the grant.'

The same Constitutional mandate is found in Sec. 2, Art. XII of


the 1987 Constitution, which declares: LLjur

'All lands of the public domain, waters, minerals,


coal, petroleum, and other mineral oils, all forces of
potential energy, fisheries, forests or timber, wildlife, flora
and fauna, and other natural resources are owned by the
State. With the exception of agricultural lands, all other
natural resources shall not be alienated. The exploration,
development, and utilization of natural resources shall be
under the full control and supervision of the State . . .'"

Notwithstanding Our ruling , in favor of the constitutionality of P.D. 1214,


petitioners contend that having filed mining lease applications on the mining
claims they have previously located and registered under then existing laws,
pursuant to the requirements of this Presidential Decree, and despite
the waiver of their rights to the issuance of mining patents therefor (emphasis
theirs), they cannot be placed in equal footing with those who forfeit all rights
by virtue of non-filing of an application within the prescribed period such that
they (petitioners) have no causes of action against private respondents.

We are not persuaded by this contention.

Although We may agree that those who filed their mining lease applications
have better rights than those who forfeited all their right by not filing at all,
this, however, does not amount to any vested right which could be the basis for
their cause of action against herein private respondents. What is precisely
waived is their right to the issuance of a mining patent upon application. This
in effect grants the government the power, in the exercise of its sound
discretion, to award the patent to the applicant most qualified to explore,
develop and exploit the mineral resources of the country in line with the
objectives of P.D. 463, and not necessarily to the original locator of the mining
claim. To sustain their contention that they can question the award of mining
patents to applicants other than them would put to naught the objectives
of P.D. 1214 as enunciated in its WHEREASclauses.

We agree with the trial court that with the waiver of their right to the issuance
of a mining patent upon their application for a mining lease, their status is
reduced to a mere applicant, their only advantage over the others is the fact
that they have already conducted explorations at the site and this exploration
may be ongoing. But still, this credential, so to speak, is not intended to tie the
hands of the government so as to prevent it from awarding the mining patent to
some other applicants, which in its belief may be more qualified than them.

WHEREFORE, the petition in G.R. No. 73931 is hereby DISMISSED for lack of
merit; the Order of dismissal assailed in G.R. No. 70423 is AFFIRMED and this
petition is hereby likewise DISMISSED; the Order of dismissal assailed in G.R.
Nos. 63786-87 is AFFIRMED and these petitions are hereby DISMISSED. No
pronouncements as to costs.

SO ORDERED.

||| (United Paracale Mining Co., Inc. v. Dela Rosa, G.R. No. 63786-87, 70423,
73931, [April 7, 1993])
THIRD DIVISION

[G.R. No. 73974. May 31, 1995.]

REPUBLIC OF THE PHILIPPINES (Represented by the


Director of Lands), petitioner, vs. THE REGISTER OF DEEDS
OF QUEZON, MANUEL G. ATIENZA, DEVELOPMENT
BANK OF THE PHILIPPINES (Lucena Branch) and
INTERMEDIATE APPELLATE COURT, respondents.

The Solicitor General for petitioner.

Godofredo Manipod for respondent DBP.

Caluntad-Alfaro & Associate for private respondent M.G. Atienza.

SYLLABUS

1. REMEDIAL LAW; ACTIONS; APPEAL; BEING A STATUTORY RIGHT,


REQUISITES MUST BE STRICTLY COMPLIED WITH. — Appeal is an essential
part of our judicial system. As such, courts should proceed with caution so as
not to deprive a party of the right to appeal, particularly if the appeal is
meritorious. Respect for the appellant's right, however, carries with it the
corresponding respect for the appellee's similar rights to fair play and justice.
Thus, appeal being a purely statutory right, an appealing party must strictly
comply with the requisites laid down in the Rules of Court.

2. ID.; ID.; ID.; ID.; SERVICE OF COPY OF APPELLANTS BRIEF, MANDATORY.


— Of paramount importance is the duty of an appellant to serve a copy of his
brief upon the appellee with proof of service thereof. This procedural
requirement is consonant with Section 2 of Rule 13, which mandates that all
pleadings and papers "shall be filed with the court, and served upon the parties
affected thereby." The importance of serving copies of the brief upon the
adverse party is underscored in Mozar v. Court of Appeals, where the Court
held that the appellees "should have been given an opportunity to file their
appellee's brief in the Court of Appeals if only to emphasize the necessity of due
process."

3. ID.; ID.; ID.; ID.; ID.; CASE AT BENCH. — In this case, however, the
Court of Appeals, oblivious of the fact that this case involves public lands
requiring as it does the exerciseof extraordinary caution lest said lands be
dissipated and erroneously alienated to undeserving or unqualified private
individuals, decided the appeal without hearing the government's side.

4. ID.; ID.; ONLY THE SOLICITOR GENERAL CAN BRING OR DEFEND


ACTIONS ON BEHALF OF THE REPUBLIC AND SHOULD BE FURNISHED
WITH COPIES OF ALL COURT ORDERS, NOTICES AND DECISIONS; CASE AT
BENCH. — Atienza avers that he furnished Atty. Francisco Torres, a lawyer in
the Bureau of Lands and designated special attorney for the Office of the
Solicitor General, with two copies of the appellant's brief, thereby implying that
it was not his fault that petitioner failed to file its appellee's brief. Such an
assertion betrays a lack of comprehension of the role of the Solicitor General as
government counsel or of the OSG as the government's "law office." Only the
Solicitor General, as the lawyer of the government, can bring or defend actions
on behalf of the Republic of the Philippines and, therefore, actions filed in the
name of the Republic, if not initiated by the Solicitor General, will be
summarily dismissed. Specifically, he is empowered to represent the
Government in all land registration and related proceedings, such as, an action
for cancellation of title and for reversion of a homestead to the government.
Hence, he is entitled to be furnished with copies of all court orders, notices and
decisions. Consequently, service of decisions on the Solicitor General is the
proper basis for computing the reglementary period for filing appeals and for
finality of decisions. His representative, who may be a lawyer from the
Bureau of Lands, has no legal authority to decide whether or not an appeal
should be made. Service of the appellant's brief on Atty. Torres was no service
at all upon the Solicitor General. It may be argued that Atty. Torres could have
transmitted one of the two copies of appellant's brief upon the Solicitor
General, but such omission does not excuse Atienza's failure to serve a
copy of his brief directly on the Solicitor General.

5. ID.; SPECIAL CIVIL ACTIONS; CERTIORARI; RENDITION BY THE


APPELLATE COURT OF DECISION BASED SOLELY ON APPELLANT'S BRIEF
CONSTITUTES GRAVE ABUSE OFDISCRETION. — On the part of the appellate
court, its decision based solely on, and even quoting verbatim from, the
appellant's brief was certainly arrived at in grave abuse ofdiscretion. It denied
appellee (petitioner herein) of the opportunity to be heard and to rebut
Atienza's allegations, in rank disregard of its right to due process. Such
violation ofdue process could have been rectified with the
granting of petitioner's motion for reconsideration by the appellate court, but
even the door to this recourse was slammed by the appellate court with the
denial of petitioner's motion for extension of time to file motion for
reconsideration in a resolution dated February 12, 1986, which ruling
erroneously applied the Habaluyas doctrine. Such denial notwithstanding,
petitioner filed its motion for reconsideration. Considering the clear allegations
thereunder, the appellate court would have done well, in the interest of justice,
not to blindly adhere to technical rules of procedure by dismissing outright
said motion. As we declared in Villareal v. Court ofAppeals: ". . . The
requirements of due process are satisfied when the parties are afforded a fair
and reasonable opportunity to explain and air their side. The essence of due
process is simply the opportunity to be heard or as applied to administrative
proceedings, an opportunity to explain one's side or an opportunity to seek a
reconsideration of the action or ruling taken."

6. ID.; EVIDENCE; CREDIBILITY; FINDINGS OF FACT OF THE APPELLATE


COURT, GENERALLY UPHELD ON APPEAL; CASE AT BENCH, AN
EXCEPTION. — In view of the foregoing and the long-standing procedural rule
that this Court may review the findings of facts of the Court of Appeals in the
event that they may be contrary to those of the trial court, in order to attain
substantial justice, the Court now reviews the facts of the case.

7. CIVIL LAW; PUBLIC LAND ACT; ALL LANDS NOT OTHERWISE CLEARLY
APPEARING TO BE OWNED PRIVATELY, PRESUMED TO BELONG TO THE
STATE. — Under the Regalian Doctrine, all lands not otherwise clearly
appearing to be privately-owned are presumed to belong to the State. Forest
lands, like mineral or timber lands which are public lands, are not subject to
private ownership unless they under the Constitution, become private
properties. In the absence of such classification, the land remains unclassified
public land until released therefrom and rendered open to disposition.

8. ID.; ID.; DIRECTOR OF LANDS; TASKED WITH ADMINISTRATION AND


DISPOSAL OF PUBLIC LANDS. — In our jurisdiction, the task of administering
and disposing lands of the public domain belongs to the Director of Lands, and
ultimately, the Secretary of Agriculture and Natural Resources (now the
Secretary of Environment and Natural Resources).

9. ID.; ID.; CLASSIFICATION OF PUBLIC LAND, PREROGATIVE OF THE


EXECUTIVE. — Classification of public lands is, thus, an exclusive
prerogative of the Executive Department through the Office of the President.
Courts have no authority to do so.

10. ID.; ID.; BURDEN OF OVERCOMING PRESUMPTION OF STATE


OWNERSHIP IN CONTROVERSIES, INVOLVING DISPOSITION OF PUBLIC
LAND LIES UPON PRIVATE CLAIMANT. — In controversies involving the
disposition of public agricultural lands, the burden of overcoming the
presumption of state ownership of lands of the public domain lies upon the
private claimant who, in this case, is Atienza. The records show, however, that
he failed to present clear, positive and absolute evidence to overcome said
presumption and to support his claim.

11. ID.; ID.; ID.; ID.; PRESUMPTION NOT OVERCOME IN CASE AT BENCH. —
Atienza's claim is rooted in the March 9, 1932 decision of the then
Court of First Instance ofTayabas in Cadastral Case No. 76, which was not
given much weight by the court a quo, and for good reasons. Apart from his
assertions before this Court, Atienza failed to present proof that he or his
predecessor-in-interest was one of the claimants who answered the petition
filed by the then Attorney-General in the said cadastral proceedings. The
document reflecting said cadastral decision, a xerox copy, indicated the
claimants simply as "Jose Abastillas et al." In support of that decision, Atienza
presented a certification purportedly issued by someone from the Technical
Reference Section of the Surveys Division, apparently of the Bureau of Lands,
stating that "Lot 5886 is a portion of Lot 5139 Pagbilao Cadastre," which
evidence is, however, directly controverted by the sketch plan showing that the
land in controversy is actually outside the alienable and disposable public
lands, although part of Lot 5139.

12. ID.; ID.; TORRENS SYSTEM OF LAND REGISTRATION;


INDEFEASIBILITY OF TORRENS TITLE CANNOT BE INVOKED BY ONE WHO
PROCURED IT BY FRAUD. — The fact that Atienza acquired a title to the land
is of no moment, notwithstanding the indefeasibility of titles issued under the
Torrens system. In Bornales v. Intermediate Appellate Court, we ruled that the
indefeasibility of a certificate of title cannot be invoked by one who procured
the same by means of fraud. The "fraud" contemplated by the law (Sec. 32, P.D.
1529) is actual and extrinsic that is, "an intentional omission of fact required
by law," which in the case at bench consisted in the failure of Atienza to state
that the land sought to be registered still formed part of the unclassified forest
lands.

DECISION

ROMERO, J p:
This petition for review on certiorari seeks to nullify and set aside the
decision 1 of the then Intermediate Appellate Court reversing the decision of the
former Court of First Instance of Quezon, Branch II at Lucena City 2 which
annulled Original Certificate of Title (OCT) No. P-13840 and Free Patent (FP)
No. 324198 issued to Manuel Atienza for a 17-hectare piece of land which
turned out to be within the forest zone in Pagbilao, Quezon.

On April 18, 1967, Atienza was awarded FP No. 324198 over a parcel of land
located in Ila, Malicboy, Pagbilao, Quezon, with an area of 172,028 square
meters. By virtue of such award, he was issued on May 5, 1967, OCT No. P-
13840.

Sometime in 1968, an investigation was conducted by the Bureau of Lands in


connection with alleged land grabbing activities in Pagbilao. It appeared that
some of the free patents, including that of Atienza's, were fraudulently
acquired. Thus, on March 19, 1970, a criminal complaint for
falsification of public documents was filed in the then Court ofFirst
Instance of Quezon, Branch II, against Atienza and four other persons for
allegedly falsifying their applications for free patent, the survey plans, and
other documents pertinent to said applications.

In its decision dated October 4, 1972, the court acquitted the accused of the
crime charged but, finding that the land covered by the application for free
patent of private respondent (ATIENZA) was within the forest zone, declared as
null and void OCT No. P-13840 in Atienza's name and ordered
the Register of Deeds of Quezon to cancel the same.

Meanwhile, before the promulgation of said decision, or on May 10, 1972, then
Acting Solicitor General Conrado T. Limcaoco filed for the petitioner a
complaint against Atienza, the Register of Deeds of Quezon, and the Rural
Bank of Sariaya, which was later dropped as defendant and, in an amended
complaint, substituted by the Development Bank ofthe Philippines as actual
mortgagee of the subject parcel of land. Docketed as Civil Case No. 7555, the
complaint prayed for the declaration of nullity of FP No. 324198 and OCT No.
P-13840.

In his answer, Atienza claimed that the land in question was no longer within
the unclassified public forest land because by the approval of his application
for free patent by the Bureau of Lands, the land "was already alienable and
disposable public agricultural land." Since the subject land was a very small
portion of Lot 5139 of the Pagbilao Cadastre, an area which had been declared
disposable public land by the cadastral court on March 9, 1932 in Cadastral
Case No. 76 entitled "El Govierno Filipino de las Islas Filipinas contra Jose
Abastillas, et al., G.L.R.O. Cadastral Record No. 1124," he also averred that the
Director of Lands had given due course to free and homestead patent
applications ofclaimants to Lot 5139. He further alleged that through a certain
Sergio Castillo, he had been in possession of the land since the Japanese
occupation, cultivating it and introducing improvements thereon. The DBP,
after due and proper investigation and inspection of his title, even granted him
a loan with the subject property as collateral. Finally, he stated that his
acquittal in the criminal case proved that he committed no fraud in his
application for free patent.

On July 27, 1981, the lower court rendered a decision with the categorical
finding based on "solid evidence" that "the land in question was found definitely
within the forest zone denominated as Project 21-A."

The dispositive portion thereof reads as follows:

"WHEREFORE, in view of the foregoing, (J)udgment is hereby


rendered:

(a) Declaring as null and void Original Certificate of Title No. P-


13840 in the name of defendant Manuel G. Atienza, as well as
Free Patent No. V-324198;
(b) Ordering defendant Manuel G. Atienza to pay the
Development Bank of the Philippines, Lucena City Branch, the
sum of P15,053.97, and all interests due thereon; and

(c) Ordering defendant Manuel G. Atienza to pay the costs of this


suit.

SO ORDERED."

On appeal, Atienza maintained that the land in question was not within the
unclassified public forest land and therefore alienable land of the public
domain. The then Intermediate Appellate Court relied only on the arguments
he raised since petitioner had not filed any brief, and arrived at the conclusion
that "(t)he litigated land is part ofpublic land alienable and disposable for
homestead and [F]ree Patent." On December 27, 1985, the appellate court set
aside the lower court's decision, declared as valid and subsisting Atienza's
OCT, and dismissed the cross-claim of the DBP.

After receiving a copy of said decision, Assistant Solicitor General Oswaldo D.


Agcaoili informed the Director of Lands of the adverse decision of the appellate
court, which noted that no appellee's brief had been filed in said court. Agcaoili
also stated that the Office of the Solicitor General (OSG) had not been
furnished with the appellant's brief; that the Bureau of Lands received
notice of hearing of the record on appeal filed by the appellant but the OSG had
not been informed of the "action taken thereon;" that since the Bureauof Lands
had been furnished directly with relevant pleadings and orders, the same office
should "take immediate appropriate action on the decision;" and that it may file
a motion for reconsideration within fifteen (15) days from January 6, 1986, the
date of receipt by the OSG of the copy of the decision sought to be
appealed. llcd

On January 28, 1986, petitioner filed a motion for extension of time to file
motion for reconsideration which was denied in a resolution dated February
12, 1986. Petitioner's motion for reconsideration of said resolution was likewise
denied.

The instant petition for review on certiorari raises the following arguments: (a)
petitioner was denied due process and fair play when Atienza did not furnish it
with a copy of his appellant's brief before the then Intermediate Appellate Court
thereby depriving it of the opportunity to rebut his assertions which later
became the sole basis of the assailed decision of December 27, 1985; (b) the
appellate court erred in holding that the land in question is part of the
alienable and disposable public land in complete disregard of the trial court's
finding that it forms part of the unclassified public forest zone; and (c) the
appellate court erred in declaring that the land in question could be alienated
and disposedof in favor of Atienza.

We find for the petitioner.

Appeal is an essential part of our judicial system. As such, courts should


proceed with caution so as not to deprive a party of the right to appeal,
particularly if the appeal is meritorious. 3 Respect for the appellant's right,
however, carries with it the corresponding respect for the appellee's similar
rights to fair play and justice. Thus, appeal being a purely statutory right, an
appealing party must strictly comply with the requisites laid down in the
Rules of Court. 4

Of paramount importance is the duty of an appellant to serve a copy of his brief


upon the appellee with proof of service thereof. 5 This procedural requirement
is consonant withSection 2 of Rule 13, which mandates that all pleadings and
papers "shall be filed with the court, and served upon the parties affected
thereby." The importance of serving copiesof the brief upon the adverse party is
underscored in Mozar v. Court of Appeals, 6 where the Court held that the
appellees "should have been given an opportunity to file their appellee's brief in
the Court of Appeals if only to emphasize the necessity of due process."
In this case, however, the Court of Appeals, oblivious of the fact that this case
involves public lands requiring as it does the exercise of extraordinary caution
lest said lands be dissipated and erroneously alienated to undeserving or
unqualified private individuals, decided the appeal without hearing the
government's side.

Atienza avers that he furnished Atty. Francisco Torres, a lawyer in the


Bureau of Lands and designated special attorney for the Office of the Solicitor
General, with two copies ofthe appellant's brief, thereby implying that it was
not his fault that petitioner failed to file its appellee's brief.

Such an assertion betrays a lack of comprehension of the role of the Solicitor


General as government counsel or of the OSG as the government's "law
office." 7 Only the Solicitor General, as the lawyer of the government, can bring
or defend actions on behalf of the Republic of the Philippines and, therefore,
actions filed in the name of the Republic, if not initiated by the Solicitor
General, will be summarily dismissed. 8 Specifically, he is empowered to
represent the Government in all land registration and related proceedings, 9
such as, an action for cancellation of title and for reversion of a homestead to
the government. 10 Hence, he is entitled to be furnished with copies of all court
orders, notices and decisions. Consequently, service of decisions on the
Solicitor General is the proper basis for computing the reglementary period for
filing appeals and for finality of decisions. His representative, who may be a
lawyer from the Bureau of Lands, has no legal authority to decide whether or
not an appeal should be made. 11

Service of the appellant's brief on Atty. Torres was no service at all upon the
Solicitor General. It may be argued that Atty. Torres could have transmitted
one of the two copies ofappellant's brief upon the Solicitor General, but such
omission does not excuse Atienza's failure to serve a copy of his brief directly
on the Solicitor General.
On the part of the appellate court, its decision based solely on, and even
quoting verbatim from, the appellant's brief was certainly arrived at in grave
abuse of discretion. It denied appellee (petitioner herein) of the opportunity to
be heard and to rebut Atienza's allegations, in rank disregard of its right to due
process. Such violation of due process could have been rectified with the
granting of petitioner's motion for reconsideration by the appellate court, 12
but even the door to this recourse was slammed by the appellate court with the
denial of petitioner's motion for extension of time to file motion for
reconsideration in a resolution dated February 12, 1986, which ruling
erroneously applied theHabaluyas doctrine. 13

Such denial notwithstanding, petitioner filed its motion for reconsideration.


Considering the clear allegations thereunder, the appellate court would have
done well, in the interest of justice, not to blindly adhere to technical
rules of procedure by dismissing outright said motion. As we declared
in Villareal v. Court of Appeals: 14

". . . . The requirements of due process are satisfied when the


parties are afforded a fair and reasonable opportunity to explain
and air their side. The essence of due process is simply the
opportunity to be heard or as applied to administrative
proceedings, an opportunity to explain one's side or an
opportunity to seek a reconsideration of the action or ruling
taken." (Emphasis supplied)

In view of the foregoing and the long-standing procedural rule that this Court
may review the findings of facts of the Court of Appeals in the event that they
may be contrary to those of the trial court, 15 in order to attain substantial
justice, the Court now reviews the facts of the case.

Under the Regalian Doctrine, all lands not otherwise clearly appearing to be
privately-owned are presumed to belong to the State. Forest lands, like mineral
or timber lands which are public lands, are not subject to private ownership
unless they under the Constitution, become private properties. In the
absence of such classification, the land remains unclassified public land until
released therefrom and rendered open to disposition. 16

In our jurisdiction, the task of administering and disposing lands of the public
domain belongs to the Director of Lands, and ultimately, the
Secretary of Environment and Natural Resources 17 (now the
Secretary of Environment and Natural Resources). 18 Classification of public
lands is, thus, an exclusive prerogative of the Executive Department through
the Office of the President. 19 Courts have no authority to do so. 20

Thus, in controversies involving the disposition of public agricultural lands, the


burden of overcoming the presumption of state ownership of lands of the public
domain lies upon the private claimant 21 who, in this case, is Atienza. The
records show, however, that he failed to present clear, positive and absolute
evidence 22 to overcome said presumption and to support his claim.

Atienza's claim is rooted in the March 9, 1932 decision of the then


Court of First Instance of Tayabas in Cadastral Case No. 76, which was not
given much weight by the court a quo, and for good reasons. llcd

Apart from his assertions before this Court, Atienza failed to present proof that
he or his predecessor-in-interest was one of the claimants who answered the
petition filed by the then Attorney-General in the said cadastral proceedings.
The document reflecting said cadastral decision, a xerox copy, indicated the
claimants simply as "Jose Abastillas et al." In support of that decision, Atienza
presented a certification purportedly issued by someone from the Technical
Reference Section of the Surveys Division, apparently of the Bureauof Lands,
stating that "Lot 5886 is a portion of Lot 5139 Pagbilao Cadastre," which
evidence is, however, directly controverted by the sketch plan showing that the
land in controversy is actually outside the alienable and disposable public
lands, although part of Lot 5139.
The fact that Atienza acquired a title to the land is of no moment,
notwithstanding the indefeasibility of titles issued under the Torrens system.
In Bornales v. Intermediate Appellate Court, 23 we ruled that the
indefeasibility of a certificate of title cannot be invoked by one who procured
the same by means of fraud. The "fraud" contemplated by the law (Sec. 32, P.D.
1529) is actual and extrinsic, that is "an intentional omission of fact required
by law," 24 which in the case at bench consisted in the failure of Atienza to
state that the land sought to be registered still formed part of the unclassified
forest lands.

WHEREFORE, the decision appealed from is hereby REVERSED and SET


ASIDE and the decision of the court a quo dated July 27, 1981, is
REINSTATED.

SO ORDERED.

||| (Republic v. Register of Deeds of Quezon, G.R. No. 73974, [May 31, 1995],
314 PHIL 473-489)
FIRST DIVISION

[G.R. No. 128017. January 20, 1999.]

RAMON ITURALDE, petitioner, vs.


ALFREDO FALCASANTOS, respondent.

Leo Jay T. Principe for petitioner.

Bienvenido G. Martin for private respondent.

SYNOPSIS

ITURALDE acquired by purchase a 6-hectare land located at Bañas, Lantawan,


Basilan Province on October 17, 1986. However, said lot was applied with the
Bureau of Lands by FALCASANTOS for a free patent. The application of
Falcasantos was dismissed by the Regional Director of Lands for failure to
exercise the right to repurchase and allowed petitioner to file a public land
application for the subject land.

Thereafter, ITURALDE filed a complaint for recovery of ownership and


possession against FALCASANTOS. The trial court rendered judgment in favor
of ITURALDE who was declared the owner and possessor of the subject land.

On appeal, the Court of Appeals reversed the appealed decision in finding that
the land is within the forest reserve area, hence, not capable of private
appropriation and occupation.

Hence, this recourse, ITURALDE by claiming that allowance of the Director of


Lands to file a public land application for said property is equivalent to a
declaration that said land was no longer part of the public domain.

Lands within the forest reserve are not capable of private appropriation and
occupation; that a positive act of the government is needed to declassify a forest
land into alienable or disposable land for agricultural or other purposes; and that
possession of forest lands, however long, cannot ripen into private
ownership. ICHcTD

SYLLABUS

1. CIVIL LAW; PUBLIC LAND ACT; LANDS WITHIN FOREST RESERVE, NOT
CAPABLE OF PRIVATE APPROPRIATION AND OCCUPATION. — The Court of
Appeals correctly held that "the evidence is unrebutted that the subject land is
within the Forest Reserve Area as per L.C. Map No. 1557 certified on August
13, 1951." and, hence, not capable of private appropriation and occupation.
In Republic vs. Register of Deeds of Quezon, we held that "Forest lands, like
mineral or timber lands which are public lands, are not subject to private
ownership unless they under the Constitution, become private properties. In
the absence of such classification, the land remains unclassified public land
until released therefrom and rendered open to disposition." HDTCSI

2. ID.; ID.; DECLASSIFICATION OF PUBLIC LANDS; POSITIVE ACT FROM


GOVERNMENT, INDISPENSABLE. — In Sunbeam Convenience Foods Inc. vs.
Court of Appeals, we said: "Thus, before any land may be declassified from the
forest group and converted into alienable or disposable land for agricultural or
other purposes, there must be a positive act from the government. Even rules
on the confirmation of imperfect titles do not apply unless and until the land
classified as forest land is released in an official proclamation to that effect so
that it may form part of the disposable agricultural lands of the public
domain." Hence, a positive act of the government is needed to declassify a
forest land into alienable or disposable land for agricultural or other purposes."

3. ID.; POSSESSION; POSSESSION OF FOREST LANDS HOWEVER LONG


CANNOT RIPEN INTO PRIVATE OWNERSHIP. — The rule is "Possession of
forest lands, however long, cannot ripen into private ownership."
DECISION

PARDO, J p:

The case is an appeal via certiorari from a decision of the Court of Appeals
reversing that of the Regional Trial Court, Branch 2, Basilan province, and
dismissing petitioner's complaint for recovery of possession and ownership of a
parcel of land with the improvements existing thereon, situated at Barangay
Upper Bañas, municipality of Lantawan, province of Basilan, with an area
of 7.1248 hectares. cdlex

The facts may be related as follows:

On October 17, 1986, ITURALDE acquired by purchase from the heirs of Pedro
Mana-ay a parcel of land located at Bañas, Lantawan, Basilan Province, with
an area of 6.0000 hectares, more or less, more particularly described as
follows:

"A parcel of land, situated at Bañas, Lantawan Basilan. Bounded


on the North by property of Alejandro Marso; on the East by
property of Ramon Bacor; on the South by property of Atty.
Ricardo G. Mon and on the West by property of Librada Guerrero.
Containing an area of 6.0000 hectares, more or less."

However, on November 3, 1986, FALCASANTOS applied with the Bureau of


Lands in Isabela, Basilan province, for the award to him of the same parcel of
land under free patent. On November 17, 1986, ITURALDE filed a protest to
such application.

On February 7, 1989, the Regional Director of Lands rendered a decision giving


respondent a period of one hundred twenty (120) days to exercise the right to
repurchase the land by reimbursing petitioner of all expenses he incurred in
the purchase of the property in question, and held in abeyance respondent's
application for free patent.
On October 11, 1989, the Regional Director issued an order declaring that
FALCASANTOS had waived his right of repurchase, and rejected his application
for free patent for lack of interest, and allowed ITURALDE to file a public land
application for the subject land.

On May 8, 1990, the Regional Director ordered FALCASANTOS to vacate the


land in question, but respondent refused.

On July 24, 1990, ITURALDE filed with the Regional Trial Court, Basilan
province, a complaint for recovery of ownership and possession with
preliminary injunction of the subject parcel of land.

In answer to the complaint, FALCASANTOS alleged that the land occupied by


him belonged to the Republic of the Philippines, and that he had introduced
improvements thereon such as coconut and other fruit trees. cdtai

After trial on the merits, on March 20, 1993, the trial court rendered decision
declaring ITURALDE the owner and possessor of the subject parcel of land with
all the improvements existing thereon, situated at Barangay Upper Bañas,
municipality of Lantawan, province of Basilan, with an area of 3.1248 hectares,
and ordering respondent to vacate the land in question, to pay petitioner the
amount of ten thousand pesos (P10,000.00) as attorneys fee, the amount of five
thousand pesos (P5,000.00) as litigation expenses, and three hundred pesos
(P300.00) as judicial cost.

In due time, petitioner appealed the trial court's decision to the Court of
Appeals.

On December 20, 1996, the Court of Appeals rendered decision reversing the
appealed decision, and entering a new judgment dismissing ITURALDE’S
complaint without prejudice to any action that HE may take if the subject land
was declassified from forest land to alienable and disposable land of the public
domain.

Hence, the present recourse.


ITURALDE submits that the Court of Appeals erred in setting aside the trial
court's decision in his favor and dismissing the complaint because when the
Director of Lands allowed petitioner to file a public land application for said
property, it was equivalent to a declaration that said land was no longer part of
the public domain.

We deny the petition. The Court of Appeals correctly held that "the evidence is
unrebutted that the subject land is within the Forest Reserve Area as per L.C.
Map No. 1557 certified on August 13, 1951' " 1 and, hence, not capable of
private appropriation and occupation. 2

In Republic vs. Register of Deeds of Quezon, we held that "Forest lands, like
mineral or timber lands which are public lands, are not subject to private
ownership unless they under the Constitution, become private properties. In
the absence of such classification, the land remains unclassified public land
until released therefrom and rendered open to disposition." 3

In Sunbeam Convenience Foods Inc. vs. Court of Appeals, we said: "Thus, before
any land may be declassified from the forest group and converted into alienable
or disposable land for agricultural or other purposes, there must be a positive
act from the government. Even rules on the confirmation of imperfect titles do
not apply unless and until the land classified as forest land is released in an
official proclamation to that effect so that it may form part of the disposable
agricultural lands of the public domain." 4

"Hence, a positive act of the government is needed to declassify a forest land


into alienable or disposable land for agricultural or other purposes." 5

And the rule is "Possession of forest lands, however long, cannot ripen into
private ownership." 6

What is more, there is yet no award or grant to petitioner of the land in


question by free patent or other ways of acquisition of public land.
Consequently, he can not lawfully claim to be the owner of the land in
question.

WHEREFORE, the Court hereby AFFIRMS the appealed decision of the Court of
Appeals in CA-G.R. CV No. 42306, dismissing the complaint of petitioner before
the Regional Trial Court, Basilan province, in Civil Case No. 441-63. cdrep

No costs.

SO ORDERED.

Davide, Jr., C.J., Melo, Kapunan and Martinez, JJ., concur.

||| (Ituralde v. Falcasantos, G.R. No. 128017, [January 20, 1999], 361 PHIL
245-250)
G.R. No. L-43938 April 15, 1988

REPUBLIC OF THE PHILIPPINES (DIRECTOR OF FOREST DEVELOPMENT), petitioner,


vs.
HON. COURT OF APPEALS (THIRD DIVISION) and JOSE Y. DE LA ROSA, respondents.

G.R. No. L-44081 April 15, 1988

BENGUET CONSOLIDATED, INC., petitioner,


vs.
HON. COURT OF APPEALS, JOSE Y. DE LA ROSA, VICTORIA, BENJAMIN and EDUARDO, all
surnamed DE LA ROSA, represented by their father JOSE Y. DE LA ROSA, respondents.

G.R. No. L-44092 April 15, 1988

ATOK-BIG WEDGE MINING COMPANY, petitioner,


vs.
HON. COURT OF APPEALS, JOSE Y. DE LA ROSA, VICTORlA, BENJAMIN and EDUARDO, all
surnamed DE LA ROSA, represented by their father, JOSE Y. DE LA ROSA, respondents.

CRUZ, J.:

The Regalian doctrine reserves to the State all natural wealth that may be found in the bowels of the earth even if the land where the
discovery is made be private. 1 In the cases at bar, which have been consolidated because they pose a common issue, this doctrine was not
correctly applied.

These cases arose from the application for registration of a parcel of land filed on February 11,
1965, by Jose de la Rosa on his own behalf and on behalf of his three children, Victoria, Benjamin
and Eduardo. The land, situated in Tuding, Itogon, Benguet Province, was divided into 9 lots and
covered by plan Psu-225009. According to the application, Lots 1-5 were sold to Jose de la Rosa
and Lots 6-9 to his children by Mamaya Balbalio and Jaime Alberto, respectively, in 1964. 2

The application was separately opposed by Benguet Consolidated, Inc. as to Lots 1-5, Atok Big
Wedge Corporation, as to Portions of Lots 1-5 and all of Lots 6-9, and by the Republic of the
Philippines, through the Bureau of Forestry Development, as to lots 1-9. 3

In support of the application, both Balbalio and Alberto testified that they had acquired the subject
land by virtue of prescription Balbalio claimed to have received Lots 1-5 from her father shortly after
the Liberation. She testified she was born in the land, which was possessed by her parents under
claim of ownership. 4 Alberto said he received Lots 6-9 in 1961 from his mother, Bella Alberto, who
declared that the land was planted by Jaime and his predecessors-in-interest to bananas, avocado,
nangka and camote, and was enclosed with a barbed-wire fence. She was corroborated by Felix
Marcos, 67 years old at the time, who recalled the earlier possession of the land by Alberto's
father. 5 Balbalio presented her tax declaration in 1956 and the realty tax receipts from that year to
1964, 6 Alberto his tax declaration in 1961 and the realty tax receipts from that year to 1964. 7

Benguet opposed on the ground that the June Bug mineral claim covering Lots 1-5 was sold to it on
September 22, 1934, by the successors-in-interest of James Kelly, who located the claim in
September 1909 and recorded it on October 14, 1909. From the date of its purchase, Benguet had
been in actual, continuous and exclusive possession of the land in concept of owner, as evidenced
by its construction of adits, its affidavits of annual assessment, its geological mappings, geological
samplings and trench side cuts, and its payment of taxes on the land. 8

For its part, Atok alleged that a portion of Lots 1-5 and all of Lots 6-9 were covered by the Emma
and Fredia mineral claims located by Harrison and Reynolds on December 25, 1930, and recorded
on January 2, 1931, in the office of the mining recorder of Baguio. These claims were purchased
from these locators on November 2, 1931, by Atok, which has since then been in open, continuous
and exclusive possession of the said lots as evidenced by its annual assessment work on the
claims, such as the boring of tunnels, and its payment of annual taxes thereon. 9

The location of the mineral claims was made in accordance with Section 21 of the Philippine Bill of
1902 which provided that:

SEC. 21. All valuable mineral deposits in public lands in the philippine Islands both
surveyed and unsurveyed are hereby declared to be free and open to exploration,
occupation and purchase and the land in which they are found to occupation and
purchase by the citizens of the United States, or of said islands.

The Bureau of Forestry Development also interposed its objection, arguing that the land sought to be
registered was covered by the Central Cordillera Forest Reserve under Proclamation No. 217 dated
February 16, 1929. Moreover, by reason of its nature, it was not subject to alienation under the
Constitutions of 1935 and 1973. 10

The trial court * denied the application, holding that the applicants had failed to prove their claim of possession and ownership of the
land sought to be registered. 11 The applicants appealed to the respondent court, * which reversed the trial court and recognized the claims of
the applicant, but subject to the rights of Benguet and Atok respecting their mining claims. 12 In other words, the Court of Appeals affirmed
the surface rights of the de la Rosas over the land while at the same time reserving the sub-surface rights of Benguet and Atok by virtue of
their mining claims.

Both Benguet and Atok have appealed to this Court, invoking their superior right of ownership. The
Republic has filed its own petition for review and reiterates its argument that neither the private
respondents nor the two mining companies have any valid claim to the land because it is not
alienable and registerable.

It is true that the subject property was considered forest land and included in the Central Cordillera
Forest Reserve, but this did not impair the rights already vested in Benguet and Atok at that time.
The Court of Appeals correctly declared that:

There is no question that the 9 lots applied for are within the June Bug mineral claims
of Benguet and the "Fredia and Emma" mineral claims of Atok. The June Bug
mineral claim of plaintiff Benguet was one of the 16 mining claims of James E. Kelly,
American and mining locator. He filed his declaration of the location of the June Bug
mineral and the same was recorded in the Mining Recorder's Office on October 14,
1909. All of the Kelly claims ha subsequently been acquired by Benguet
Consolidated, Inc. Benguet's evidence is that it had made improvements on the June
Bug mineral claim consisting of mine tunnels prior to 1935. It had submitted the
required affidavit of annual assessment. After World War II, Benguet introduced
improvements on mineral claim June Bug, and also conducted geological mappings,
geological sampling and trench side cuts. In 1948, Benguet redeclared the "June
Bug" for taxation and had religiously paid the taxes.

The Emma and Fredia claims were two of the several claims of Harrison registered in
1931, and which Atok representatives acquired. Portions of Lots 1 to 5 and all of Lots
6 to 9 are within the Emma and Fredia mineral claims of Atok Big Wedge Mining
Company.

The June Bug mineral claim of Benguet and the Fredia and Emma mineral claims of
Atok having been perfected prior to the approval of the Constitution of the Philippines
of 1935, they were removed from the public domain and had become private
properties of Benguet and Atok.

It is not disputed that the location of the mining claim under


consideration was perfected prior to November 15, 1935, when the
Government of the Commonwealth was inaugurated; and according
to the laws existing at that time, as construed and applied by this
court in McDaniel v. Apacible and Cuisia (42 Phil. 749), a valid
location of a mining claim segregated the area from the public
domain. Said the court in that case: The moment the locator
discovered a valuable mineral deposit on the lands located, and
perfected his location in accordance with law, the power of the United
States Government to deprive him of the exclusive right to the
possession and enjoyment of the located claim was gone, the lands
had become mineral lands and they were exempted from lands that
could be granted to any other person. The reservations of public
lands cannot be made so as to include prior mineral perfected
locations; and, of course, if a valid mining location is made upon
public lands afterwards included in a reservation, such inclusion or
reservation does not affect the validity of the former location. By such
location and perfection, the land located is segregated from the public
domain even as against the Government. (Union Oil Co. v. Smith,
249 U.S. 337; Van Mess v. Roonet, 160 Cal. 131; 27 Cyc. 546).

"The legal effect of a valid location of a mining claim is not only to


segregate the area from the public domain, but to grant to the locator
the beneficial ownership of the claim and the right to a patent therefor
upon compliance with the terms and conditions prescribed by law.
Where there is a valid location of a mining claim, the area becomes
segregated from the public domain and the property of the locator."
(St. Louis Mining & Milling Co. v. Montana Mining Co., 171 U.S. 650;
655; 43 Law ed., 320, 322.) "When a location of a mining claim is
perfected it has the effect of a grant by the United States of the right
of present and exclusive possession, with the right to the exclusive
enjoyment of all the surface ground as well as of all the minerals
within the lines of the claim, except as limited by the extralateral right
of adjoining locators; and this is the locator's right before as well as
after the issuance of the patent. While a lode locator acquires a
vested property right by virtue of his location made in compliance with
the mining laws, the fee remains in the government until patent
issues."(18 R.C.L. 1152) (Gold Creek Mining Corporation v. Hon.
Eulogio Rodriguez, Sec. of Agriculture and Commerce, and Quirico
Abadilla, Director of the Bureau of Mines, 66 Phil. 259, 265-266)

It is of no importance whether Benguet and Atok had secured a patent for as held in
the Gold Creek Mining Corp. Case, for all physical purposes of ownership, the owner
is not required to secure a patent as long as he complies with the provisions of the
mining laws; his possessory right, for all practical purposes of ownership, is as good
as though secured by patent.

We agree likewise with the oppositors that having complied with all the requirements
of the mining laws, the claims were removed from the public domain, and not even
the government of the Philippines can take away this right from them. The reason is
obvious. Having become the private properties of the oppositors, they cannot be
deprived thereof without due process of law. 13

Such rights were not affected either by the stricture in the Commonwealth Constitution against the
alienation of all lands of the public domain except those agricultural in nature for this was made
subject to existing rights. Thus, in its Article XIII, Section 1, it was categorically provided that:

SEC. 1. All agricultural, timber and mineral lands of the public domain, waters,
minerals, coal, petroleum and other mineral oils, all forces of potential energy and
other natural resources of the Philipppines belong to the State, and their disposition,
exploitation, development, or utilization shall be limited to citizens of the Philippines
or to corporations or associations at least 60% of the capital of which is owned by
such citizens, subject to any existing right, grant, lease or concession at the time of
the inauguration of the government established under this Constitution. Natural
resources with the exception of public agricultural lands, shall not be alienated, and
no license, concession, or lease for the exploitation, development or utilization of any
of the natural resources shall be granted for a period exceeding 25 years, except as
to water rights for irrigation, water supply, fisheries, or industrial uses other than the
development of water power, in which case beneficial use may be the measure and
the limit of the grant.

Implementing this provision, Act No. 4268, approved on November 8, 1935, declared:

Any provision of existing laws, executive order, proclamation to the contrary


notwithstanding, all locations of mining claim made prior to February 8, 1935 within
lands set apart as forest reserve under Sec. 1826 of the Revised Administrative
Code which would be valid and subsisting location except to the existence of said
reserve are hereby declared to be valid and subsisting locations as of the date of
their respective locations.

The perfection of the mining claim converted the property to mineral land and under the laws then in
force removed it from the public domain. 14 By such act, the locators acquired exclusive rights over
the land, against even the government, without need of any further act such as the purchase of the
land or the obtention of a patent over it. 15As the land had become the private property of the
locators, they had the right to transfer the same, as they did, to Benguet and Atok.

It is true, as the Court of Appeals observed, that such private property was subject to the
"vicissitudes of ownership," or even to forfeiture by non-user or abandonment or, as the private
respondents aver, by acquisitive prescription. However, the method invoked by the de la Rosas is
not available in the case at bar, for two reasons.

First, the trial court found that the evidence of open, continuous, adverse and exclusive possession
submitted by the applicants was insufficient to support their claim of ownership. They themselves
had acquired the land only in 1964 and applied for its registration in 1965, relying on the earlier
alleged possession of their predecessors-in-interest. 16The trial judge, who had the opportunity to
consider the evidence first-hand and observe the demeanor of the witnesses and test their credibility
was not convinced. We defer to his judgment in the absence of a showing that it was reached with
grave abuse of discretion or without sufficient basis. 17

Second, even if it be assumed that the predecessors-in-interest of the de la Rosas had really been in
possession of the subject property, their possession was not in the concept of owner of the mining
claim but of the property as agricultural land, which it was not. The property was mineral land, and
they were claiming it as agricultural land. They were not disputing the lights of the mining locators
nor were they seeking to oust them as such and to replace them in the mining of the land. In fact,
Balbalio testified that she was aware of the diggings being undertaken "down below" 18 but she did
not mind, much less protest, the same although she claimed to be the owner of the said land.

The Court of Appeals justified this by saying there is "no conflict of interest" between the owners of
the surface rights and the owners of the sub-surface rights. This is rather doctrine, for it is a well-
known principle that the owner of piece of land has rights not only to its surface but also to
everything underneath and the airspace above it up to a reasonable height. 19 Under the aforesaid
ruling, the land is classified as mineral underneath and agricultural on the surface, subject to
separate claims of title. This is also difficult to understand, especially in its practical application.

Under the theory of the respondent court, the surface owner will be planting on the land while the
mining locator will be boring tunnels underneath. The farmer cannot dig a well because he may
interfere with the operations below and the miner cannot blast a tunnel lest he destroy the crops
above. How deep can the farmer, and how high can the miner, go without encroaching on each
other's rights? Where is the dividing line between the surface and the sub-surface rights?

The Court feels that the rights over the land are indivisible and that the land itself cannot be half
agricultural and half mineral. The classification must be categorical; the land must be either
completely mineral or completely agricultural. In the instant case, as already observed, the land
which was originally classified as forest land ceased to be so and became mineral — and completely
mineral — once the mining claims were perfected. 20 As long as mining operations were being
undertaken thereon, or underneath, it did not cease to be so and become agricultural, even if only
partly so, because it was enclosed with a fence and was cultivated by those who were unlawfully
occupying the surface.

What must have misled the respondent court is Commonwealth Act No. 137, providing as follows:

Sec. 3. All mineral lands of the public domain and minerals belong to the State, and
their disposition, exploitation, development or utilization, shall be limited to citizens of
the Philippines, or to corporations, or associations, at least 60% of the capital of
which is owned by such citizens, subject to any existing right, grant, lease or
concession at the time of the inauguration of government established under the
Constitution.

SEC. 4. The ownership of, and the right to the use of land for agricultural, industrial,
commercial, residential, or for any purpose other than mining does not include the
ownership of, nor the right to extract or utilize, the minerals which may be found on or
under the surface.

SEC. 5. The ownership of, and the right to extract and utilize, the minerals included
within all areas for which public agricultural land patents are granted are excluded
and excepted from all such patents.
SEC. 6. The ownership of, and the right to extract and utilize, the minerals included
within all areas for which Torrens titles are granted are excluded and excepted from
all such titles.

This is an application of the Regalian doctrine which, as its name implies, is intended for the benefit
of the State, not of private persons. The rule simply reserves to the State all minerals that may be
found in public and even private land devoted to "agricultural, industrial, commercial, residential or
(for) any purpose other than mining." Thus, if a person is the owner of agricultural land in which
minerals are discovered, his ownership of such land does not give him the right to extract or utilize
the said minerals without the permission of the State to which such minerals belong.

The flaw in the reasoning of the respondent court is in supposing that the rights over the land could
be used for both mining and non-mining purposes simultaneously. The correct interpretation is that
once minerals are discovered in the land, whatever the use to which it is being devoted at the time,
such use may be discontinued by the State to enable it to extract the minerals therein in the exercise
of its sovereign prerogative. The land is thus converted to mineral land and may not be used by any
private party, including the registered owner thereof, for any other purpose that will impede the
mining operations to be undertaken therein, For the loss sustained by such owner, he is of course
entitled to just compensation under the Mining Laws or in appropriate expropriation proceedings. 21

Our holding is that Benguet and Atok have exclusive rights to the property in question by virtue of
their respective mining claims which they validly acquired before the Constitution of 1935 prohibited
the alienation of all lands of the public domain except agricultural lands, subject to vested rights
existing at the time of its adoption. The land was not and could not have been transferred to the
private respondents by virtue of acquisitive prescription, nor could its use be shared simultaneously
by them and the mining companies for agricultural and mineral purposes.

WHEREFORE, the decision of the respondent court dated April 30, 1976, is SET ASIDE and that of
the trial court dated March 11, 1969, is REINSTATED, without any pronouncement as to costs.

SO ORDERED.

Teehankee, C.J., Narvasa, Gancayco and Griño-Aquino, JJ., concur.


FIRST DIVISION

[G.R. No. 195990. August 5, 2015.]

HEIRS OF RAFAEL GOZO represented by


CASTILLO GOZO and RAFAEL GOZO,
JR., petitioners, vs. PHILIPPINE UNION MISSION CORPORATI
ON OF THESEVENTH DAY ADVENTIST CHURCH (PUMCO),
SOUTH PHILIPPINE UNION MISSION OF SDA (SPUMCO)
and SEVENTH DAY ADVENTIST CHURCH AT SIMPAK, LALA,
LANAO DEL NORTE represented by BETTY
PEREZ, respondents.

DECISION

PEREZ, J p:

This is a Petition for Review on Certiorari 1 filed by


petitioners Heirs of Rafael Gozo seeking to reverse and set aside the 10
November 2010 Decision 2 of the Court ofAppeals and its 14 February 2011
Resolution 3 in CA-G.R. CV No. 00188. The assailed decision and resolution
reversed the 30 June 2004 Decision of the Regional Trial Court
(RTC) of Kapatagan, Lanao del Norte and held that the action for
nullification and recovery of possession filed by the petitioners is already
barred by laches. The dispositive portion of the Court of Appeals Decision
reads:

ACCORDINGLY, the Decision dated 30 June


2004 of the court a quo is REVERSED and SET
ASIDE. The South Philippine Union Mission of the Seventh Day
Adventist Churchremains the absolute owner of the donated
property. 4

The Facts

HEIRS OF GOZO (PETITIONERS) claim that they


are the heirs of the Spouses Rafael and Concepcion Gozo (Spouses Gozo)
who, before their death, were the original owners of a parcel ofland with an
area 236,638 square meters located in Sitio Simpak, Brgy. Lala,
Municipality of Kolambugan, Lanao del Norte.

The PUMCO/SPUMCO claim that they own a 5,000 square-meter


portion of the property. The assertion is based on the 28 February 1937
Deed of Donation favor of respondent Philippine Union Mission Corporation
of the Seventh Day Adventist(PUMCO-SDA). Respondents took
possession of the subject property by introducing improvements thereon
through the construction of a church building, and later on, an elementary
school. On the date the Deed of Donation is executed in
1937, theSpouses Gozo were not the registered owners of the property yet
although they were the lawful possessors thereof. It was only on 5 October
1953 that the Original Certificate of Title (OCT) No. P-642 covering the entire
property was issued in thename of Rafael Gozo (Rafael) married to
Concepcion Gozo (Concepcion) pursuant to the Homestead Patent granted
by the President of the Philippines on 22 August 1953. 6

In view of Rafael's prior death, however, his heirs, Concepcion, and


their six children, namely, Abnera, Benia, Castillo, Dilbert, Filipinas and
Grace caused the extrajudicial partition of the property.
Accordingly, the Register of Deeds of Lanao del Norte issued a new
certificate of title under Transfer Certificate of Title (TCT) No. (T-347)-
292 7 underthe names of the heirs on 13 January 1954. CAIHTE
On 30 July 1992, Concepcion caused the survey
and the subdivision of the entire property including the portion occupied by
PUMCO-SDA. 8 It was at this point that respondents brought
to the attention of Concepcion that the 5,000 square-meter
portion of the property is already owned by respondent PUMCO-SDA in
view of the Deed of Donation she executed together with her husband in
their favor in 1937. When Concepcion, however, verified the matter
with the Register Deeds, it appeared that the donation was not annotated
in the title. The absence of annotation of the so-called encumbrance
in the title prompted petitioners not to recognize the donation claimed
by the respondents. The matter was left unresolved until Concepcion died
and the rest of the owners continued to pursue their claims to
recover the subject property from the respondents.

A compromise was initially reached by the parties


wherein the petitioners were allowed by respondents to harvest
from the coconut trees planted on the subject property but a
misunderstanding ensued causing respondents to file a case for qualified
theft against the petitioners.

On 19 June 2000 or around six decades after the Deed of Donation


was executed, petitioners filed an action for
Declaration of Nullity of Document, Recovery ofPossession and Ownership
with Damages against PUMCO-SDA before the RTC of Kapatagan, Lanao del
Norte. 9 In their Complaint docketed as Civil Case No. 21-201, petitioners
claimed that the possession of PUMCO-SDA on the subject property was
merely tolerated by petitioners and therefore could not ripen into
ownership. 10 In addition, petitioners argued
that the signatures of the Spouses Gozo were forged underscoring the stark
contrast between the genuine signatures of their parents from theones
appearing in the deed. 11 Finally, petitioners averred that granting
for the sake of argument that the said signatures were
genuine, the deed of donation will remain invalid for lack of acceptance
which is an essential requisite for a valid contract of donation. 12

For their part, respondents insisted on the validity of the donation


and on the genuineness of the signatures of the donors who had voluntarily
parted with their property as faithful
devotees of the church for the pursuit of social and religious ends. 13 They
further contended that
from the moment the Spouses Gozo delivered thesubject property to
respondents in 1937, they were already in open, public, continuous and
adverse possession thereof in the concept of an owner. 14 A considerable
improvement was claimed to have been introduced into the property
in the form of church and school
buildings. 15 The argument of the petitioners, therefore, that thedonation
was invalid for lack of acceptance, a question which came 63 years after it
was executed, is already barred by laches.

After the pre-trial conference, trial on the merits ensued. Both parties
adduced documentary and testimonial evidence to support their respective
positions.

On 30 June 2004, the RTC rendered a Decision 16 in


favor of the petitioners thereby declaring that they are the rightful
owners of the subject property since the contract of donation which
purportedly transferred the ownership of the subject property to PUMCO-
SDA is void for lack of acceptance. In upholding the right of the petitioners
to theland, the court a quo held that an action or defense
for the declaration of nullity of a contract does not prescribe.
Anent the claim that petitioners slept on their rights, the RTC adjudged
that the equitable doctrine of laches is inapplicable in the case at bar
because the action of the registered owners to recover possession is based
on Torrens title which cannot be barred by laches. The RTC disposed in this
wise: DETACa
WHEREFORE, in view of the foregoing consideration,
judgment is hereby rendered in favor of the [petitioners], to wit:

(1) Declaring the 5,000 square meter portion of the land covered
by TCT [No.] (T-347)-292 part of the common
property of the [petitioners]; and

(2) Declaring the Deed of Donation as void.

The [petitioners], however, are not entitled to damages,


attorney's fees and cost of litigation prayed for. 17

On appeal, the Court of Appeals reversed the RTC Decision 18 and


ordered the dismissal of petitioners' complaint
on the ground of laches. The appellate court opined that petitioners failed to
assert their rights over the land for more than 60 years, thus, laches had set
in. Even if petitioners were the registered owners of the land in
dispute,laches would still bar them from recovering possession of the same.

The Motion for Reconsideration filed by the petitioners was likewise


denied by the appellate court in a Resolution 19 dated 14 February 2011.

Petitioners are now before this Court via this instant Petition for
Review on Certiorari seeking the reversal of the Court of Appeals Decision
and Resolution on the sole ground that:

THE HONORABLE COURT OF APPEALS ERRED IN


REVERSING AND SETTING
ASIDE THE DECISION OF THE COURT A
QUO ON THE GROUND OF LACHES. 20

The Court's Ruling

While the opposing parties center their arguments and counter-


arguments
on the timeliness of raising the question of the validity of donation, a careful
scrutiny of therecords, however, reveals a significant fact that
at the time the Deed of Donation was executed by the Spouses Gozo on 28
February 1937, the subject property was part ofthe inalienable public
domain. It was only almost after two decades later or on 5 October
1953 that the State ceded its right over the land in
favor of the Spouses Gozo by granting their patent application and issuing
an original certificate of title in their favor. Prior to such
conferment of title, the Spouses Gozo possessed no right to
dispose ofthe land which, by all intents and purposes, belongs to the State.

Under the Regalian doctrine, which is embodied in Article XII, Section


2 of our Constitution, all lands of the public domain belong to the State,
which is the source of any asserted right to any ownership of land. All lands
not appearing to be clearly within private ownership are presumed to belong
to the State. Accordingly, public lands not shown to have been reclassified
or released as alienable agricultural land or alienated to a private person
by the State remain part of the inalienable public domain. 21

The classification of public lands is an exclusive


prerogative of the executive department of the government and
not the Courts. In the absence of such classification, theland remains as an
unclassified land until it is released therefrom and rendered open to
disposition. This is in consonance with the Regalian doctrine that all
lands of thepublic domain belong to the State and that the State
is the source of any asserted right to ownership in land and charged
with the conservation of such patrimony. 22

All lands not appearing to be clearly within private ownership are


presumed to belong to the State. Accordingly, all public lands not shown to
have been reclassified or released as alienable agricultural land or alienated
to a private person by the State remain part of the alienable public domain.
As already well-settled in jurisprudence, nopublic land can be acquired by
private persons without any grant, express or implied, from the government;
and it is indispensable that the person claiming title to public land should
show that his title was acquired from the State or any other
mode of acquisition recognized by law. To prove that the land subject of an
application for registration is alienable, the applicant must
establish the existence of a positive act of the government such as a
presidential proclamation or an executive order, an administrative action,
investigation reports of Bureau of Lands investigators, and a legislative act
or a statute. The applicant may also secure a certification
from the Government that the land applied for is alienable and
disposable. 23 aDSIHc

Commonwealth Act No. 141, also known as the Public Land Act, as
amended by Presidential Decree No. 1073, remains to this day the existing
general law governing theclassification and disposition of lands of the public
domain, other than timber and mineral lands. The following provisions
under Title I, Chapter II of the Public Land Act, as amended, is very specific
on how lands of the public domain become alienable or disposable: 24

SEC. 6. The President,


upon the recommendation of the Secretary of Agriculture and
Natural Resources, shall from time to time
classify the lands of the public domain into:

(a) Alienable or disposable,

(b) Timber, and

(c) Mineral lands,

and may at any time and in a like manner transfer such lands
from one class to another, for the purposes of their
administration and disposition.

SEC. 7. For the purposes of the administration and


disposition of alienable or disposable public lands, the Batasang
Pambansa or the President, upon recommendation
by the Secretary of Natural Resources, may from time to time
declare what public lands are open to disposition or concession
under this Act.

xxx xxx xxx

SEC. 8. Only those lands shall be declared open to


disposition or concession which have been officially delimited
and classified and, when practicable, surveyed, and which have
not been reserved for public or quasi-public uses, nor
appropriated by the Government, nor in any manner become
private property, nor those on which a private right authorized
and recognized by this Act or any other valid law may be
claimed, or which, having been reserved or appropriated, have
ceased to be so. However, thePresident may, for
reasons of public interest, declare lands of the public domain
open to disposition before the same have had their boundaries
established or been surveyed, or may, for the same reason,
suspend their concession or disposition until they are again
declared open to concession or disposition by proclamation duly
published or by Actof the Congress. ATICcS

SEC. 9. For the purpose of their administration and


disposition, the lands of the public domain alienable or open to
disposition shall be classified, according to theuse or purposes
to which such lands are destined, as follows:

(a) Agricultural;

(b) Residential, commercial, industrial, or for


similar productive purposes;

(c) Educational, charitable, or other similar


purposes; and
(d) Reservations for townsites and for public and
quasi-public uses.

The President, upon recommendation


by the Secretary of Agriculture and Natural Resources, shall
from time to time make the classifications provided for in this
section, and may, at any time and in a similar manner, transfer
lands from one class to another. 25

By virtue of Presidential Decree No. 705, otherwise known


as the Revised Forestry Code, the President delegated to the DENR
Secretary the power to determine whichof the unclassified
lands of the public domain are (1) needed for forest purposes and declare
them as permanent forest to form part of the forest reserves; and (2) not
needed for forest purposes and declare them as alienable and disposable
lands. 26

Per the Public Land Act, alienable and disposable public lands
suitable for agricultural purposes can be disposed of only as follows:

1. For homestead settlement;

2. By sale;

3. By lease; and

4. By confirmation of imperfect or incomplete titles:

(a) By judicial legalization;

(b) By administrative legalization (free patent). 27

Homestead over alienable and disposable public agricultural land


is granted after compliance by an applicant with the conditions and
requirements laid down under Title II, Chapter IV of the Public Land
Act, the most basic of which are quoted below:

SEC. 12. Any citizen of the Philippines


over the age of eighteen years, or the head of a family, who does
not own more than twenty-four hectares of land
in thePhilippines or has not had the benefit of any gratuitous
allotment of more than twenty-four hectares of land
since the occupation of the Philippines by the United States,
may enter a homestead of not exceeding twenty-four
hectares of agricultural land of the public domain.

SEC. 13. Upon the filing of an application for a


homestead, the Director of Lands, if he finds
that the application should be approved, shall do so and
authorize theapplicant to take possession of the land
upon the payment of five pesos, Philippine currency, as entry
fee. Within six months from and
after the date of the approval of theapplication, the applicant
shall begin to work the homestead, otherwise he shall lose his
prior right to the land.

SEC. 14. No certificate shall be given or patent issued


for the land applied for until at least one-fifth of the land has
been improved and cultivated. The period within which the land
shall be cultivated shall not be less than one nor more than five
years, from and
after the date of the approval of the application. The applicant
shall, within thesaid period, notify the Director of Lands as soon
as he is ready to acquire the title. If at the date of such
notice, the applicant shall prove
to the satisfaction of the Director ofLands, that he has resided
continuously for at least one year in the municipality in
which the land is located, or in a municipality adjacent
to the same and has cultivated at least one-fifth of the land
continuously since the approval of the application, and shall
make affidavit that no part of said land has been alienated or
encumbered, and that he has complied with
all the requirements of this Act, then, upon the payment of five
pesos, as final fee, he shall be entitled to a patent. 28 TIADCc

It is clear under the law that before compliance with the foregoing
conditions and requirements the applicant has no right over the land
subject of the patent and therefore cannot dispose the same even if such
disposal was made gratuitously. It is an established principle that no one
can give what one does not have, nemo dat quod non habet. 29 It is true that
gratuitous disposal in donation may consist of a thing or a right
but the term right must be understood in a "proprietary" sense over
which thepossessor has jus disponendi. 30 This is because in true
donations there results a consequent impoverishment of the donor or
diminution of his assets. 31 In Republic v. Courtof Appeals, 32 the Court
declared the contract of donation, executed by the donor who
has no proprietary right over the object of the contract, null and void, viz.:

Even on the gratuitous assumption that a


donation of the military "camp site" was executed between
Eugenio de Jesus and Serafin Marabut, such donation would
anyway be void because Eugenio de Jesus held no dominical
rights over the site when it was allegedly donated by him in
1936. In that year, Proclamation No. 85 ofPresident Quezon
already withdrew the area from sale or settlement and reserved
it for military purposes. . . . Eugenio de Jesus cannot be said to
be possessed of that "proprietary" right over the whole 33
hectares in 1936 including the disputed 12.8081 hectares for
at the time this 12.8081-hectare lot had already been severed
from the mass disposable public lands by Proclamation No. 85
and excluded from the Sales Award.
Impoverishment of Eugenio's asset as a result of such donation
is therefore farfetched. (Emphasis supplied)
It is beyond question that at the time the gratuitous transfer was
effected by the Spouses Gozo on 28 February 1937, the subject property
was part of the public domain and is outside the commerce of man. It was
only on 5 October 1953 that the ownership of the property was vested
by the State to the Spouses Gozo by virtue of its issuance of the OCT
pursuant to the Homestead Patent granted
by the President of the Philippines on 22 August 1953.
Hence, the donation of the subject property which took place
before 5 October 1953 is null and void from the very start. 33

As a void contract, the Deed of Donation produces no legal effect


whatsoever. Quod nullum est, nullum producit effectum. 34 That which is a
nullity produces no effect.35 Logically, it could not have transferred title
to the subject property from the Spouses Gozo to PUMCO-SDA and there
can be no basis for the church's demand for theissuance of title under its
name. Neither does the church have the right to subsequently
dispose the property nor invoke acquisitive prescription to justify its
occupation. A void contract is not susceptible to ratification, and the action
for the declaration of absolute nullity of such contract is imprescriptible. 36

The lack of respondents' right over the property was confirmed


when the Spouses Gozo had the entire property, including the portion
occupied by the church, surveyed and patented, and covered by their
homestead patent. Further, after a certificate of title was issued under their
names, the Spouses Gozo did not effect the annotation
thereon of the supposed donation. Registration is the operative act that
gives validity to the transfer or creates a lien upon the land. 37 Indeed it
has been ruled that where there was nothing in the certificate of title to
indicate any cloud or vice in the ownership of the property, or any
encumbrance thereon, the purchaser is not required to explore farther than
what the Torrens title upon its face indicates in quest for any hidden defect
or inchoate right that may subsequently defeat his right
thereto. 38 If the rule were otherwise, the efficacy and
conclusiveness of the certificate of title which the Torrens system seeks to
insure would entirely be futile and nugatory. 39 The public shall then be
denied of its foremost motivation for respecting and observing the Land
Registration Act. 40

Just as significantly, the homestead


application of the Spouses Gozo over the entire area of the property
including that occupied by respondents and the issuance in their
favor of the corresponding title without any complaint or objection
from the respondents, remove the case of the petitioners
from the operation of the doctrine of laches.

And, further than the issuance of an original title, the entire property
was made subject of an extrajudicial partition of the property
by the Gozo heirs resulting in theissuance of TCTs in their names in 1954.
Again, in no instance during the partition did the respondents make known
their claim over the property. AIDSTE

Clearly from the facts, the petitioners asserted their rights repeatedly;
it was the respondents who kept silent all throughout about the supposed
donee's rights.

WHEREFORE, premises considered, the instant petition is


hereby GRANTED. The assailed Decision dated 10 November 2010 and
Resolution dated 14 February 2011 ofthe Court of Appeals in CA-
G.R. CV No. 00188 are hereby REVERSED and SET ASIDE.

SO ORDERED.

||| (Heirs of Gozo v. Philippine Union Mission Corp. of the Seventh Day
Adventist Church, G.R. No. 195990, [August 5, 2015])
IMPERIUM AND DOMINIUM

G.R. No. L-30389 December 27, 1972

PEDRO LEE HONG HOK, SIMEON LEE HONG HOK, ROSITA LEE HONG
HOK and LEONCIO LEE HONG HOK, petitioners,
vs.
ANIANO DAVID, THE HON. SECRETARY OF AGRICULTURE AND NATURAL
RESOURCES, THE DIRECTOR OF LANDS and COURT OF
APPEALS, respondents.

Augusto A. Pardalis for petitioners.

Luis General, Jr. for respondent Aniano David.

Office of the Solicitor General for other respondents.

FERNANDO, J.:p

Petitioners 1 in this appeal by certiorari would have us reverse a decision of


respondent Court of Appeals affirming a lower court judgment dismissing their
complaint to have the Torrens Title 2 of respondent Aniano David declared null
and void.

What makes the task for petitioners quite difficult is that their factual support
for their pretension to ownership of such disputed lot through accretion was
rejected by respondent Court of Appeals. Without such underpinning, they
must perforce rely on a legal theory, which, to put it mildly, is distinguished by
unorthodoxy and is therefore far from persuasive.

A grant by the government through the appropriate public


officials 3 exercising the competence duly vested in them by law is not to
be set at naught on the premise, unexpressed but implied, that land not
otherwise passing into private ownership may not be disposed of by the
state. Such an assumption is at war with settled principles of constitutional
law. It cannot receive our assent. We affirm.

ANIANO DAVID was granted rights to the disputed land pursuant to


his miscellaneous sales patent by the Director of Lands on June 18, 1958.
Subsequent thereto, an OCT was issued in favor of Aniano David.

It appears that from the moment of filing of miscellaneous sales


application of David until the issuance of OCT, Lee Hong hok interposed
no objections. This is fatal to them because after the registration and
issuance of the certificate and duplicate certificate of title based on a
public land patent, the land covered thereby automatically comes under
the operation of Republic Act 496 subject to all the safeguards provided
therein.... Under Section 38 of Act 496 any question concerning the
validity of the certificate of title based on fraud should be raised within
one year from the date of the issuance of the patent.

Ruling of Director of Lands and Secretary of DANR: The land is private


land

Actual classification of the land: In this case the land in question is not a
private property as the Director of Lands and the Secretary of Agriculture
and Natural Resources have always sustained the public character thereof
for having been formed by reclamation.

(Therefore, if land is reclaimed it is still land of the public domain which


may be disposed of by the State)

What is the remedy of Lee Hong Hok to claim back the land? Petition for
Reconveyance on the ground of fraud.
ISSUE: W/N the land may be registered in the name of ANiano David?

RULING: YES. IT IS PUBLIC DOMAIN WHICH MAY BE DISPOSED OF BY


THE STATE.

ISSUE II: W/N there was fraud sufficient to warrant the filing of the action
for reconveyance? – NONE.

In this case we do not see any fraud committed by defendant-appellant


Aniano David in applying for the purchase of the land involved through
his Miscellaneous Sales Application No. MSA-V-26747, entered in the
records of the Bureau of Lands [Miscellaneous Sales] Entry No. V-9033,
because everything was done in the open. The notices regarding the
auction sale of the land were published, the actual sale and award thereof
to Aniano David were not clandestine but open and public official acts of
an officer of the Government. The application was merely a renewal of his
deceased wife's application, and the said deceased occupied the land since
1938." 4

ISSUE: WHAT IS THE RELEVANCE OF IMPERIUM AND DOMINIUM?

RULING: As there are overtones indicative of skepticism, if not of outright


rejection, of the well-known distinction in public law between the
government authority possessed by the state which is appropriately
embraced in the concept of sovereignty, and its capacity to own or
acquire property, it is not inappropriate to pursue the matter further. The
former comes under the heading of imperium and the latter of dominium.
The use of this term is appropriate with reference to lands held by the
state in its proprietary character. In such capacity, it may provide for the
exploitation and use of lands and other natural resources, including their
disposition, except as limited by the Constitution. Dean Pound did speak
of the confusion that existed during the medieval era between such two
concepts, but did note the existence of res publicae as a corollary to
dominium." 11 As far as the Philippines was concerned, there was a
recognition by Justice Holmes in Cariño v. Insular Government, 12 a case
of Philippine origin, that "Spain in its earlier decrees embodied the
universal feudal theory that all lands were held from the Crown...." 13
That was a manifestation of the concept of jura regalia, 14 which was
adopted by the present Constitution, ownership however being vested in
the state as such rather than the head thereof. What was stated by
Holmes served to confirm a much more extensive discussion of the
matter in the leading case of Valenton v. Murciano, 15 decided in 1904.
One of the royal decrees cited was incorporated in the Recopilacion de
Leyes de las Indias 16 in these words: "We having acquired full
sovereignty over the Indies and all lands, territories, and possessions not
heretofore ceded away by our royal predecessors, or by us, or in our name,
still pertaining to the royal crown and patrimony, it is our will that all
lands which are held without proper and true deeds of grant be restored to
us according as they belong to us, in order that after reserving before all
what to us or to our viceroys audiences, and governors may seem
necessary for public squares, ways, pastures, and commons in those
places which are peopled, taking into consideration not only their present
condition, but also their future and their probable increase, and after
distributing to the natives what may be necessary for tillage and
pasturage, confirming them in what they now have and giving them more
if necessary, all the rest of said lands may remain free and unencumbered
for us to dispose of as we may wish." 17

It could therefore be affirmed in Montano v. Insular Government" 18 that


"as to the unappropriated public lands constituting the public domain the
sole power of legislation is vested in Congress, ..." 19 They continue to
possess that character until severed therefrom by state grant. 20 Where,
as in this case, it was found by the Court of Appeals that the disputed lot
was the result of reclamation, its being correctly categorized as public
land is undeniable. 21 What was held in Heirs of Datu Pendatun v.
Director of Lands 22 finds application. Thus: "There being no evidence
whatever that the property in question was ever acquired by the
applicants or their ancestors either by composition title from the Spanish
Government or by possessory information title or by any other means for
the acquisition of public lands, the property must be held to be public
domain." 23 For it is well-settled "that no public land can be acquired by
private persons without any grant, express or implied, from the
government." 24 It is indispensable then that there be a showing of a title
from the state or any other mode of acquisition recognized by law. 25 The
most recent restatement of the doctrine, found in an opinion of Justice
J.B.L. Reyes, follows: 26 "The applicant, having failed to establish his
right or title over the northern portion of Lot No. 463 involved in the
present controversy, and there being no showing that the same has been
acquired by any private person from the Government, either by purchase
or by grant, the property is and remains part of the public domain." 27 To
repeat, the second assignment of error is devoid of merit.

The decision of respondent Court of Appeals following that of the lower court
makes clear that there is no legal justification for nullifying the right of
respondent Aniano David to the disputed lot arising from the grant made in his
favor by respondent officials.

As noted in the decision under review, he "acquired lawful title thereby


pursuant to his miscellaneous sales application in accordance with which an
order of award and for issuance of a sales patent was made by the Director of
Lands on June 18, 1958, covering Lot 2892 containing an area of 226 square
meters, which is a portion of Lot 2863 of the Naga Cadastre. On the basis of
the order of award of the Director of Lands the Undersecretary of Agriculture
and Natural Resources issued on August 26, 1959, Miscellaneous Sales Patent
No. V-1209 pursuant to which OCT No. 510 was issued by the Register of
Deeds of Naga City to defendant-appellee Aniano David on October 21, 1959.
According to the Stipulation of Facts, since the filing of the sales application of
Aniano David and during all the proceedings in connection with said
application, up to the actual issuance of the sales patent in his favor, the
plaintiffs-appellants did not put up any opposition or adverse claim thereto.
This is fatal to them because after the registration and issuance of the
certificate and duplicate certificate of title based on a public land patent, the
land covered thereby automatically comes under the operation of Republic Act
496 subject to all the safeguards provided therein.... Under Section 38 of Act
496 any question concerning the validity of the certificate of title based on
fraud should be raised within one year from the date of the issuance of the
patent. Thereafter the certificate of title based thereon becomes indefeasible....

In this case the land in question is not a private property as the Director of
Lands and the Secretary of Agriculture and Natural Resources have always
sustained the public character thereof for having been formed by
reclamation.... The only remedy therefore, available to the appellants is an
action for reconveyance on the ground of fraud. In this case we do not see any
fraud committed by defendant-appellant Aniano David in applying for the
purchase of the land involved through his Miscellaneous Sales Application No.
MSA-V-26747, entered in the records of the Bureau of Lands [Miscellaneous
Sales] Entry No. V-9033, because everything was done in the open. The notices
regarding the auction sale of the land were published, the actual sale and
award thereof to Aniano David were not clandestine but open and public
official acts of an officer of the Government. The application was merely a
renewal of his deceased wife's application, and the said deceased occupied the
land since 1938." 4
On such finding of facts, the attempt of petitioners to elicit a different
conclusion is likely to be attended with frustration. The first error assigned
predicated an accretion having taken place, notwithstanding its rejection by
respondent Court of Appeals, would seek to disregard what was accepted by
respondent Court as to how the disputed lot came into being, namely by
reclamation. It does not therefore call for any further consideration. Neither of
the other two errors imputed to respondent Court, as to its holding that
authoritative doctrines preclude a party other than the government to dispute
the validity of a grant and the recognition of the indefeasible character of a
public land patent after one year, is possessed of merit. Consequently, as set
forth at the outset, there is no justification for reversal.

1. More specifically, the shaft of criticism was let loose by petitioner aimed at
this legal proposition set forth in the exhaustive opinion of then Justice
Salvador Esguerra of the Court of Appeals, now a member of this Court: "There
is, furthermore, a fatal defect of parties to this action. Only the Government,
represented by the Director of Lands, or the Secretary of Agriculture and
Natural Resources, can bring an action to cancel a void certificate of title
issued pursuant to a void patent (Lucas vs. Durian, 102 Phil. 1157; Director of
Lands vs. Heirs of Ciriaco Carlo, G.R. No. L-12485, July 31, 1959). This was
not done by said officers but by private parties like the plaintiffs, who cannot
claim that the patent and title issued for the land involved are void since they
are not the registered owners thereof nor had they been declared as owners in
the cadastral proceedings of Naga Cadastre after claiming it as their private
property. The cases cited by appellants are not in point as they refer to private
registered lands or public lands over which vested rights have been acquired
but notwithstanding such fact the Land Department subsequently granted
patents to public land applicants." 5 Petitioner ought to have known better. The
above excerpt is invulnerable to attack. It is a restatement of a principle that
dates back to Maninang v. Consolacion, 6a 1908 decision. As was there
categorically stated: "The fact that the grant was made by the government is
undisputed. Whether the grant was in conformity with the law or not is a
question which the government may raise, but until it is raised by the
government and set aside, the defendant can not question it. The legality of the
grant is a question between the grantee and the government." 7 The above
citation was repeated ipsissimis verbis in Salazar v. Court of Appeals. 8 Bereft
as petitioners were of the right of ownership in accordance with the findings of
the Court of Appeals, they cannot, in the language of Reyes v.
Rodriguez, 9 "question the [title] legally issued." 10 The second assignment of
error is thus disposed of.

2. As there are overtones indicative of skepticism, if not of outright rejection, of


the well-known distinction in public law between the government authority
possessed by the state which is appropriately embraced in the concept of
sovereignty, and its capacity to own or acquire property, it is not inappropriate
to pursue the matter further. The former comes under the heading
of imperium and the latter of dominium. The use of this term is appropriate with
reference to lands held by the state in its proprietary character. In such
capacity, it may provide for the exploitation and use of lands and other natural
resources, including their disposition, except as limited by the Constitution.
Dean Pound did speak of the confusion that existed during the medieval era
between such two concepts, but did note the existence of res publicae as a
corollary to dominium." 11 As far as the Philippines was concerned, there was a
recognition by Justice Holmes in Cariño v. Insular Government, 12 a case of
Philippine origin, that "Spain in its earlier decrees embodied the universal
feudal theory that all lands were held from the Crown...." 13 That was a
manifestation of the concept of jura regalia, 14 which was adopted by the
present Constitution, ownership however being vested in the state as such
rather than the head thereof. What was stated by Holmes served to confirm a
much more extensive discussion of the matter in the leading case of Valenton v.
Murciano, 15 decided in 1904. One of the royal decrees cited was incorporated
in the Recopilacion de Leyes de las Indias 16 in these words: "We having
acquired full sovereignty over the Indies and all lands, territories, and
possessions not heretofore ceded away by our royal predecessors, or by us, or
in our name, still pertaining to the royal crown and patrimony, it is our will
that all lands which are held without proper and true deeds of grant be
restored to us according as they belong to us, in order that after reserving
before all what to us or to our viceroys audiences, and governors may seem
necessary for public squares, ways, pastures, and commons in those places
which are peopled, taking into consideration not only their present condition,
but also their future and their probable increase, and after distributing to the
natives what may be necessary for tillage and pasturage, confirming them in
what they now have and giving them more if necessary, all the rest of said
lands may remain free and unencumbered for us to dispose of as we may
wish." 17

It could therefore be affirmed in Montano v. Insular Government" 18 that "as to


the unappropriated public lands constituting the public domain the sole power
of legislation is vested in Congress, ..." 19 They continue to possess that
character until severed therefrom by state grant. 20 Where, as in this case, it
was found by the Court of Appeals that the disputed lot was the result of
reclamation, its being correctly categorized as public land is
undeniable. 21 What was held in Heirs of Datu Pendatun v. Director of
Lands 22 finds application. Thus: "There being no evidence whatever that the
property in question was ever acquired by the applicants or their ancestors
either by composition title from the Spanish Government or by possessory
information title or by any other means for the acquisition of public lands, the
property must be held to be public domain." 23 For it is well-settled "that no
public land can be acquired by private persons without any grant, express or
implied, from the government." 24 It is indispensable then that there be a
showing of a title from the state or any other mode of acquisition recognized by
law. 25 The most recent restatement of the doctrine, found in an opinion of
Justice J.B.L. Reyes, follows: 26 "The applicant, having failed to establish his
right or title over the northern portion of Lot No. 463 involved in the present
controversy, and there being no showing that the same has been acquired by
any private person from the Government, either by purchase or by grant, the
property is and remains part of the public domain." 27 To repeat, the second
assignment of error is devoid of merit.

3. The last error assigned would take issue with this portion of the opinion of
Justice Esguerra: "According to the Stipulation of Facts, since the filing of the
sales application of Aniano David and during all the proceedings in connection
with said application, up to the actual issuance of the sales patent in his favor,
the
plaintiffs-appellants did not put up any opposition or adverse claim thereto.
This is fatal to them because after the registration and issuance of the
certificate and duplicate certificate of title based on a public land patent, the
land covered thereby automatically comes under the operation of Republic Act
496 subject to all the safeguards provided therein ... Under Section 38 of Act
496 any question concerning the validity of the certificate of title based on
fraud should be raised within one year from the date of the issuance of the
patent. Thereafter the certificate of title based thereon becomes indefeasible
..." 28 Petitioners cannot reconcile themselves to the view that respondent
David's title is impressed with the quality of indefeasibility. In thus manifesting
such an attitude, they railed to accord deference to controlling precedents. As
far back as 1919, in Aquino v. Director of
Lands, 29 Justice Malcolm, speaking for the Court, stated: "The proceedings
under the Land Registration Law and under the provisions of Chapter VI of the
Public Land Law are the same in that both are against the whole world, both
take the nature of judicial proceedings, and for both the decree of registration
issued is conclusive and final." 30 Such a view has been followed since
then. 31 The latest case in point is Cabacug v. Lao. 32 There is this revealing
excerpt appearing in that decision: "It is said, and with reason, that a holder of
a land acquired under a free patent is more favorably situated than that of an
owner of registered property. Not only does a free patent have a force and effect
of a Torrens Title, but in addition the person to whom it is granted has likewise
in his favor the right to repurchase within a period of five years." 33 It is quite
apparent, therefore, that petitioners' stand is legally indefensible.

WHEREFORE, the decision of respondent Court of Appeals of January 31,


1969 and its resolution of March 14, 1969 are affirmed. With costs against
petitioners-appellants.

Concepcion, C.J., Makalintal, Zaldivar, Castro, Teehankee Barredo, Makasiar,


Antonio and Esguerra, JJ., concur.
[G.R. No. 127296. January 22, 1998.]

EDUBIGIS GORDULA, CELSO V. FERNANDEZ, JR., CELSO A.


FERNANDEZ, NORA ELLEN ESTRELLADO, DEVELOPMENT
BANK OF THE PHILIPPINES, J.F. FESTEJO AND CO., INC.
AND REGISTER OF DEEDS OF LAGUNA, petitioners, vs. THE
HONORABLE COURT OF APPEALS and REPUBLIC OF THE
PHILIPPINES (represented by the National Power
Corporation), respondents.

Fernandez, Panote, Quimbo & Guades for petitioners.

The Solicitor General for respondents.

SYNOPSIS

The land in question is located in Talaongan, Cavinti, Laguna and within the
Caliraya-Lumot River Forest Reserve pursuant to Proclamation No. 573 as
issued by President Ferdinand E. Marcos. However, three years after such
proclamation of petitioner Edubigis Gordula filed an application for free patent
over the land with the Bureau of Lands, which was granted. Subsequently, a
Certificate of Title covering the land in question was issued by the
Register of Deeds in the name of Gordula. The other petitioners acquired rights
over the land in question by purchase. On July 16, 1987, former President
Corazon Aquino issued Executive Order No. 224 vesting in the Napocor
complete jurisdiction, control and regulation over the "Caliraya-Lumot
Watershed Reservation as covered by Proclamation No. 573. The Napocor then
stopped the construction of the road and fences being undertaken by private
petitioner Estrellado, as well as ordered Estrellado to remove all the
improvements made therein. For failure of petitioner Estrellado to remove the
improvements, the NAPOCOR then filed a complaint for Annulment of the Free
Patent and Cancellation of Titles and Reversion. After the trial court ruled in
favor of petitioners. But it was reversed by the Court of Appeals.

Hence, this petition.

In denying this petition the court ruled that it is well-settled that forest land is
incapable of registration, and its inclusion in a title is incapable of registration,
and its inclusion in a title nullifies that title. To be sure, the
defense of indefeasibility of a certificate of title issued pursuant to a free patent
does not lie against the state in an action for reversion ofthe land covered
thereby when such land is a part of a public forest or of a forest reservation,
the patent covering forest land being void ab initio. Nor can the mistake or
error ofits officials or agents in this regard be invoked against the government.

SYLLABUS

1. CIVIL LAW; PUBLIC LAND ACT; FOREST LANDS; OUTSIDE THE


COMMERCE OF MAN. — We start with the proposition that the sovereign
people, represented by their lawfully constituted government, have
untrammeled dominion over the forests on their native soil. Forest lands, being
the self-replenishing, versatile and all-important natural resource that they
are, need to be reserved and saved to promote the people's welfare. By their
very nature or by executive or statutory fiat, they are outside the
commerce of man, unsusceptible of private appropriation in any form, and
inconvertible into any character less than of inalienable public domain,
regardless of their actual state, for as long as the reservation subsists and is
not revoked by a subsequent valid declassification. It is well-settled that forest
land is incapable of registration, and its inclusion in a title nullifies that title.

2. ID.; ID.; NO PUBLIC LAND CAN BE ACQUIRE BY PRIVATE PERSONS


WITHOUT ANY GRANT FROM THE GOVERNMENT; CASE AT BAR. — No public
land can be acquired by private persons without any grant, express or implied
from the government; it is indispensable that there be a showing of a title from
the state. The facts show that petitionerGordula, did not acquire title to the
subject land prior to its reservation under Proclamation No. 573. He filed his
application for free patent only in January, 1973, more than three (3) years
after the issuance of Proclamation No. 573 in June, 1969. At that time, the
land, as part of the Caliraya-Lumot River Forest Reserve, was no longer open to
private ownership as it has been classified as public forest reserve for the
public good. TDSICH

3. ID.; ID.; ID.; CLAIMANT MUST SHOW BY CLEAR AND CONVINCING


EVIDENCE THAT PROPERTY WAS ACQUIRED BY ANY MEANS FOR
ACQUISITION OF PUBLIC LANDS. — InDirector of Lands v. Reyes, we held that
a settler claiming the protection of "private rights" to exclude his land from a
military or forest reservation must show ". . . by clear and convincing evidence
that the property in question was acquired by [any] . . . means for the
acquisition of public lands." In fine, one claiming "private rights" must prove
that he has complied with C.A. No. 141, as amended otherwise known as the
Public Land Act, which prescribes the substantive as well as the procedural
requirements for acquisition ofpublic lands. This requires at least thirty (30)
years of open, continuous, exclusive and notorious possession and
occupation of agricultural lands of the public domain under abona
fide claim of acquisition, immediately preceding the filing of the application for
free patent. The rationale for the 30-year period lies in the presumption that
the land applied for pertains to the State, and that the occupants and/or
possessors claim an interest therein only by virtue of their imperfect title or
continuous, open and notorious possession.

4. ID.; ID.; CONTINUOUS POSSESSION FOR LONG


PERIOD OF TIME OF PUBLIC AGRICULTURAL LAND NEVER CONFERS TITLE;
REASON. — Indeed, the possession of public agricultural land, however long
the period may have extended, never confers title thereto upon the possessor.
The reason, to reiterate our ruling, is because the statute oflimitations with
regard to public agricultural land does not operate against the State, unless
the occupant can prove possession and occupation of the same under
claim ofownership for the required number of years to constitute a grant from
the State.

5. ID.; ID.; ID.; POSSESSION FOR 25 YEARS OF THE LAND PRIOR TO ITS
RESERVATION SHORT OF 30-YEAR ACQUISITIVE PRESCRIPTION. — In the
case at bar, petitioners have failed to comply with the mandatory 30-year
period of possession. Their 25-year possession of the land prior to its
reservation as part of the Caliraya-Lumot River Forest Reserve cannot be
considered compliance with C.A. No. 141, as amended.

6. ID.; TORRENS SYSTEM OF LAND REGISTRATION;


DEFENSE OF DEFEASIBILITY OF TITLE DOES NOT LIE AGAINST THE STATE.
— The defense of indefeasibility of a certificate oftitle issued pursuant to a free
patent does not lie against the state in an action for reversion of the land
covered thereby when such land is a part of a public forest or of a forest
reservation, the patent covering forest land being void ab initio.

7. POLITICAL LAW; STATE; MISTAKES OR ERROR OF ITS OFFICIAL CANNOT


BE INVOKED AGAINST IT. — Nor can the mistake or error of its officials or
agents in this regard be invoked against the government.

8. CIVIL LAW; PUBLIC LAND LAW; FOREST LAND; CONVERSION OF FOREST


RESERVE INTO PUBLIC ALIENABLE LAND REQUIRES A CATEGORICAL
ACT OF DECLASSIFICATION BY THE PRESIDENT. — The conversion of a
forest reserve into public alienable land, requires no less than a categorical
act of declassification by the President, upon the recommendation of the proper
department head who has the authority to classify the lands of the public
domain into alienable or disposable, timber and mineral lands. There is none
such in this case.

DECISION
PUNO, J p:

Before us is a petition to affirm the Decision of the Regional Trial Court,


Branch 27, Sta. Cruz, Laguna, which was reversed by the
respondent Court of Appeals in its Decision 1dated June 20, 1995 in C.A.-
G.R. CV No. 45466. Petitioners' Motion for Reconsideration was denied by
respondent court on November 15, 1996. 2

The facts show that on June 26, 1969, former President Ferdinand E. Marcos
issued Proclamation No. 573 3 withdrawing from sale and settlement and
setting aside as permanent forest reserves, subject to private rights, certain
parcels of the public domain denominated as follows:

Parcel No. 1 — Magat River Forest Reserve cdtai

Parcel No. 2 — Chico River Forest Reserve

Parcel No. 3 — Abulug River Forest Reserve

Parcel No. 4 — Penaranda River Forest Reserve

Parcel No. 5 — Angat River-Bustos Dam Forest Reserve

Parcel No. 6 — Ambayawan River Forest Reserve

Parcel No. 7 — Umiray River Forest Reserve

Parcel No. 8 — Kaliwa River Forest Reserve

Parcel No. 9 — Caliraya-Lumot River Forest Reserve

Parcel No. 10 — Barit River-Lake Buhi Forest Reserve

Parcel No. 11 — Jalaur River Forest Reserve

They were primarily for use as watershed area. Their development was to be
undertaken by the Bureau of Forestry, with the cooperation of, among other
government agencies, the National Power Corporation (Napocor).
Located in Talaongan, Cavinti, Laguna with an area of Twenty Nine Thousand
Seven Hundred Seven (29,707) square meters, and bearing the following
boundaries:

North — National Power Corporation

South — Road and Baldomero Halili

West — National Power Corporation

East — National Power Corporation

the parcel of land subject of the case at bar is, by petitioners' explicit
admission, 4 within Parcel No. 9, the Caliraya-Lumot River Forest Reserve.

More than three years after the land was segregated as part of the Caliraya-
Lumot River Forest Reserve, or on January 9, 1973, petitioner
Edubigis Gordula, a native of Cavinti Laguna, filed with the Bureau of Lands,
an Application 5 for a Free Patent over the land. Manuel Fernandez and several
others also filed free patent applications covering other parcels of land in the
area.

On February 5, 1973, petitioner Gordula declared the land for taxation


purposes in his name as shown in Tax Declaration No. 0429.

The Regional Director of the Bureau of Lands referred the free patent
applications of petitioner Gordula, Fernandez, and several others to Mr.
Ravanal Ravanzo, then the GeneralManager of the Napocor. Mr. Ravanzo
responded through the following letter:

"October 24, 1973

The Director

Regional Lands Office No. IV

757 Gen. Solano St.


San Miguel, Manila

Sir:

This refers to the Free Patent Application[s] of Manuel Fernandez,


et al., of Barrio Talaongan, Cavinti, Laguna, which were referred
to this Office for clearance it having been found that they are
within the Caliraya-Lumot Watershed Reservation under
Proclamation No. 573 dated June 26, 1969.

Investigation conducted by this Office reveals that applicants


have sufficient ground to establish 'priority rights' over the areas
claimed and that agricultural improvements introduced thereon
are not detrimental to the watershed.

In view thereof, this Office interpose[s] no objection to the


application by the applicants contained in your letter dated
October 2, 1973.

Very truly yours,

(Sgd.) R.R. RAVANZO

General Manager"

On December 10, 1973, petitioner Gordula had the land surveyed; Survey
Plan No. F(IV-5) 949-D under his name was approved by the Regional
Director of the Bureau of Lands. Thereafter, Mr. Amundo Munda, a Land
Inspector of the Bureau of Lands, conducted the requisite investigations.

On January 17, 1974, petitioner Gordula's Application for Free Patent was
approved. Free Patent No. 693 was issued in his name.

On January 30, 1974, the Register of Deeds of Laguna issued, on the


basis of Free Patent No. 693, Original Certificate of Title No. P-1405 in the
name of petitioner Gordula.
He declared the land anew for taxation purposes under Tax Declaration No.
6498. He paid its real estate taxes from 1975 to 1979.

In the meantime, respondent Republic, through the Napocor, contracted the


Certeza Surveying Company to survey the area constituting the Caliraya-Lumot
River Forest Reserve. The survey plans were approved by the Regional
Director of the Bureau of Lands on October 27, 1975.

The said survey plans, as well as the Cadastral Map of Talaongan and the
Geological Plan of the Caliraya-Lumot River Forest Reserve, show that
petitioner Gordula's land is located in the saddle area of the watershed
recreation for the hydro-electric reservoir.

On January 22, 1979, petitioner Gordula sold the land to petitioner Celso V.
Fernandez, Jr. for six thousand pesos (P6,000.00). The
Register of Deeds of Laguna cancelled Original Certificate of Title No. P-1405
and issued, in lieu thereof, Transfer Certificate of Title No. T-85445 in the
name of petitioner Fernandez, Jr. The latter declared the land for taxation
purposes in his name.

On March 12, 1979, Fernandez, Jr. executed a Deed of Absolute Sale over the
land in favor of petitioner Celso A. Fernandez for six thousand five hundred
pesos (P6,500.00). Transfer Certificate of Title No. T-85445 was cancelled and
Transfer Certificate of Title No. 85594 was issued on March 21, 1979 in the
name of petitioner Fernandez.

As approved by the Bureau of Lands in Psd-Plan 04-014230, petitioner


Fernandez subdivided the land into nine (9) lots. On August 16, 1985, the
Register of Deeds of Laguna issued Transfer Certificates of Title Nos. 102492 to
102500 in his name covering the nine (9) subdivision lots.

On August 29, 1985, he sold the lots to petitioner Nora Ellen Estrellado for
twenty one thousand pesos (P21,000.00) Transfer Certificates of Title Nos.
102492 to 102500 were cancelled, and in lieu thereof, Transfer
Certificates of Title Nos. T-103404 to T-103412 were issued to petitioner
Estrellado.

On October 17, 1986, petitioner Estrellado mortgaged to petitioner


Development Bank of the Philippines (DBP) four 6 (4) of the (9) lots. Another
lot, covered by Transfer Certificate of Title No. 103408, was sold to petitioner
J.F. Festejo Company, Inc. to whom was issued, in lieu of the former, Transfer
Certificate of Title No. 106495.

On July 16, 1987, former President Corazon Aquino issued Executive Order
(E.O.) No. 224 7 vesting in the Napocor "complete jurisdiction, control and
regulation" over the "Caliraya-Lumot Watershed Reservation as covered by
Proclamation No. 573".

On July 26, 1987, Mr. Antonio Aquino, Jr., the Civil Security Officer of the
Cavinti reservoir complex, sent a Memorandum to the President of the Napocor
informing him of the fences and roads being constructed in the saddle area,
more particularly, in the lots sold by petitioner Fernandez to petitioner
Estrellado.

On July 28, 1987, Mr. A. Coronado, the Manager of the Cavinti reservoir
complex, asked petitioner Fernandez to remove all the improvements made in
the Estrellado lots. In reply, petitioner Fernandez claimed that the roads being
constructed would not adversely affect the reservoir area in case of heavy floods
because the Estrellado lots were elevated at a height of around fifty (50) feet.

In view of petitioner Fernandez's refusal, the Napocor assigned two (2) security
guards over the lot. The guards ordered the construction workers to leave their
posts and barred their return without permission from the Napocor.

On October 18, 1987, petitioner Fernandez, as attorney-in-fact and


counsel of petitioner Estrellado, wrote to the President of the Napocor
threatening to file a multi-million damage suit if the guards were not removed
within fifteen (15) days.
On November 18, 1987, respondent Republic, through the Napocor, filed
against petitioners a Complaint for Annulment of Free Patent and
Cancellation of Titles and Reversion with Writ of Preliminary Injunction in the
RTC of Sta. Cruz, Laguna. 8 On January 29, 1988, the trial court issued a
writ of preliminary injunction upon a bond of one hundred thousand pesos
(P100,000.00).

On December 28, 1993, the trial court rendered judgment in


favor of petitioners. The dispositive portion of its decision states:

"WHEREFORE, judgment is hereby rendered in favor of the


defendants and against plaintiff:

(1) Dismissing plaintiff's complaint and dissolving the


writ of preliminary injunction issued in this case;

(2) Ordering National Power Corporation to pay defendant Celso


A. Fernandez P300,000.00 as actual damages and P30,000.00 as
attorney's fees; and

With costs against the plaintiff.

SO ORDERED." 9

Respondent Republic, through the Napocor, elevated the case to the


respondent Court of Appeals.

On June 20, 1996, the respondent Court of Appeals ruled against petitioners.
It held, inter alia, viz:

"The kernel and primal issue to be resolved by the Court is


whether or not Free Patent No. IV-5(693) and Original
Certificate of Title No. P-1405 and all derivative titles thereafter
issued to the Appellees . . . are null and void. The Appellant avers
that the parcel of land covered by the aforesaid Free Patent
issued to Gordula is a portion of the vast tract ofland reserved by
former President Marcos as permanent forest under
Proclamation No. 573 dated June 26, 196[9] . . . and hence, non-
disposable and inalienable, pursuant to Section 88 in relation to
Section 83 of Commonwealth Act [No.] 141, as amended. In
contrast, the Court a quo dismissed Appellant's complaint, in the
light of the exclusionary clause in Proclamation No. 573 . . . that
the setting up of the permanent forest reserves over the Caliraya-
Lumot Watershed area was 'subject to private rights' if there be
any and the letter-clearance of the
then General Manager of [Napocor] . . . dated October 24, 1973,
interposing no objection to the application for a free
patent of Manuel Fernandez, at al.

xxx xxx xxx

We are convinced, beyond cavil, that the parcel of land


subject of the Free Patent issued to Gordula on January 17, 1974
and covered by Original Certificate of Title No. P-1405 issued On
January 30, 1974 . . . as the two (2) parcels of land purportedly
purchased by the [Republic] from Perez and Glorioso in 1941,
were public disposable and alienable lands before the issuance,
by the former President, of Proclamation No. 573, on June 26,
196[9]. . . the property was, however, later reserved, under
Proclamation No. 573, as a permanent forest, on June 26, 196[9].
Since then, the property became non-disposable and inalienable
public land. . .

xxx xxx xxx

At the time Gordula filed his application for a Free Patent, on


January 9, 1973, the parcel of land . . . was already reserved as a
permanent forest under Proclamation No. 573. Since the property
was already a forest reservation as of June 26, 196[9], the same
could no longer be disposed of or alienated in favor of private
individuals . . .

xxx xxx xxx

We do not agree with Appellees' and the Court a quo's pose


that Gordula's property was exempt from the
application of Proclamation No. 573 because, by express
provision thereof, the reservation was 'subject to private rights, if
there be any' . . .

Appellees failed to adduce proof that, as of June 26,


196[9], Gordula had acquired ownership or title to the aforesaid
property either by deed or by any other mode of acquisition from
the State by operation of law for that matter such as for instance,
alienable public land held by a possessor personally, or through
his predecessors-in-interest, openly, continuously and exclusively
for the prescribed period of thirty (30) years, is converted into
private property by mere lapse of period ipso jure . . . In the
present recourse,Gordula, as of 196[9], had been in
possession of the property or only [twenty-five (25) years] years
since 1944 when he commenced, as can be gleaned from his
application . . . for a free patent, possession of the property. The
period of Gordula's occupancy after 196[9] should not be tacked
to the period from 1944 because by then the property was not
susceptible of occupancy, disposition, conveyance or alienation. .
.

xxx xxx xxx

The Appellees cannot find refuge in the letter of the


then General Manager of [Napocor], Ravanal Ravanzo, on October
24, 1973 . . .
In the first place, Ravanzo made no explicit and unequivocal
statement, in said letter, that Gordula had priority rights to the
property. What he merely declared was that 'applicants have
sufficient ground to establish priority rights over the areas
claimed . . .' Even if it may be conceded, for the nonce, that
indeed, Ravanzo declared that Gordulahad priority rights over the
property claimed by him, such a declaration is irrefragably
erroneous. Munda and the Director of Lands erred in
recommending the approval ofGordula's application in the same
manner that the then Secretary of Agriculture and Natural
Resources erred in issuing the patent to Gordula. But then, well-
settled is the doctrine, enunciated by the Supreme Court, in a
catena of cases, that the State cannot be bound and estopped by
the errors or mistakes of its agents or officials . . .

The General Manager of the Appellant is not vested with


authority to allow the occupancy or acquisition, by private
individuals, of such properties, whether still needed by the
Appellant or not, reserved by the President of the Philippines for
permanent forests. Only the President or [the] Congress, by
statutory fiat, can revert the property to the disposable or
alienable portion of the public domain. LLjur

Anent Appellees' plea that they are buyers of the property in good
faith, they must harken to the Decision of the
Supreme Court in Republic of the Philippines vs. Court ofAppeals,
et al., 148 SCRA 480 that:

'. . . even assuming that the transferees are innocent


purchasers for value, their titles to said lands derived from
the titles of private respondents which were not validly
issued as they cover lands still a part of the public domain,
may be cancelled.' . .

We do not agree with Appellees' claim that Appellant's suit was


barred by prescription and by the purported
indefeasibility of their title. Prescription, basically, does not run
against the State. The right of the State for the
reversion of unlawfully acquired property is not barred by
prescription nor by the perceived indefeasibility of Appellees' title
for that matter. . ." 10

Thus states the dispositive portion of the decision of respondent


appellate court:

"IN THE LIGHT OF ALL THE FOREGOING, THE assailed Decision


is hereby REVERSED and SET ASIDE. Another Decision is
hereby rendered as follows:

1. Free patent No. IV-5-693 and Original


Certificate of Title No. P1405 issued under the
name of Edubigis Gordula and all derivative titles issued to
the Appellees are hereby declared null and void;

2. The parcel of land covered by said titles is hereby


declared reverted to the Government under the jurisdiction,
control and supervision of the [Napocor] under Executive
Order No. 224 of former President Corazon C. Aquino;

3. The Appellees and all those acting for and in their behalf
are hereby prohibited from intruding into and disturbing
the Appellant of its possession and dominion ofthe subject
property; [and]

4. Appellees' counterclaims are


DISMISSED. No pronouncement as to costs.
SO ORDERED. 11

Hence, this petition anchored on the following grounds:

"FIRST

RESPONDENT COURT OF APPEALS ERRED TANTAMOUNT TO


LACK OF JURISDICTION WHEN IT CONCLUDED THAT THE
SUBJECT LAND IS WITHIN THE LANDS BOUGHT BY THE NPC
EITHER FROM GERONIMO PEREZ ON MARCH 10, 1941
AND/OR FROM CELERINO GLORIOSO ON SEPTEMBER 26,
1941;

SECOND

ON [sic] THE LAST PARAGRAPH OF PAGE 19 UP TO PAGE 23,


LAST PAGE OF THE DECISION, THE
RESPONDENT COURT WENT BEYOND THE ISSUES OF THE
CASE WHICH RESULTED [IN THE] REVERSAL OF THE,
DECISION OF THE LOWER COURT . . .;

THIRD

THE FACTUAL FINDINGS AND CONCLUSION OF THE


TRIAL COURT ARE IN CONFLICT WITH THE FINDINGS OF THE
RESPONDENT COURT CONCERNING THE ISSUE OF WHETHER
OR NOT PETITIONER EDUBIGIS GORDULA HAD ACQUIRED
'PRIVATE RIGHTS' ON THE SUBJECT LAND, WHICH IS AN
EXCEPTION UNDER PROCLAMATION NO. 573. HENCE, THIS
CASE IS A QUESTION OF FACTS AND OF LAW. . .;

FOURTH

THERE IS NO QUESTION THAT THE SUBJECT LAND IS WITHIN


THE AREA OF PROCLAMATION NO. 573. HOWEVER THE
RESPONDENT [COURT] GRAVELY ERRED TANTAMOUNT TO
LACK OF JURISDICTION WHEN IT WENT TO THE
EXTENT OF DISCUSSING ON [sic] THE CIRCUMSTANCES AND
INVESTIGATION RELATIVE TO THE ISSUANCE OF THE TITLE
TO PETITIONER EDUBIGIS GORDULA AND AFTERWARD
DECLARED THAT GORDULA WHO HAS A TITLE ON THE
SUBJECT LAND HAS NOT ACQUIRED 'PRIVATE RIGHTS' ON
THE LAND DESPITE OF [sic] THE FACT THAT SAID
RESPONDENT COURT IS ALREADY PRECLUDED FROM
DISCUSSING THE FACTS RELATIVE ON [sic] THE
ISSUANCE OF THE TITLE BY AUTHORITY OF THE
PRESIDENT OF THE PHILIPPINES, MORE SO ITS FINDINGS
AND CONCLUSION IS [sic] AGAINST THE LAW, JUSTICE AND
EQUITY. THIS IS AGAINST THE RULING IN ESPINOSA VS.
MAKALINTAL, 79 PHIL. 134 and ORTUA VS. SINGSON
ENCARNACION, 5[9] PHIL. 440; and

FIFTH

THE RESPONDENT COURT GRAVELY ERRED IN CONCLUDING


THAT THEN GENERAL MANAGER RAVANZO OF NPC AND
UNDERSECRETARY OF AGRICULTURE BY AUTHORITY OF THE
PRESIDENT OF THE PHILIPPINES ERRED IN ISSUING THE
PATENT TO PETITIONER GORDULA. THIS IS AGAINST THE
RULING IN ESPINOSA VS. MAKALINTAL, 79 PHIL. 134 and
ORTUAVS. SINGSON ENCARNACION, 5[9] PHIL. 440." 12

We affirm the Court of Appeals.

We start with the proposition that the sovereign people, represented by their
lawfully constituted government, have untrammeled dominion over the forests
on their native soil. Forest lands, being the self-replenishing, versatile and all-
important natural resource that they are, need to be reserved and saved to
promote the people's welfare. By their very nature 13 or by executive or
statutory fiat, they are outside the commerce of man, unsusceptible of private
appropriation in any form, 14 and inconvertible into any character less
than of inalienable public domain, regardless of their actual state, for as long
as the reservation subsists and is not revoked by a subsequent valid
declassification. 15

"Once again, we reiterate the rule enunciated by this Court in


Director of Forestry vs. Muñoz and consistently adhered to in a
long line of cases the more recent of which is
Republic vs. Court of Appeals, that forest lands or forest reserves
are incapable of private appropriation, and possession thereof,
however long, cannot convert them into private properties. This
ruling is premised on the Regalian doctrine enshrined not only in
the 1935 and 1973 Constitution but also in the 1987
Constitution." 16

Petitioners do not contest the nature of the land in the case at bar. It is
admitted that it lies in the heart of the Caliraya-Lumot River Forest Reserve,
which Proclamation No. 573 classified as inalienable and indisposable. Its
control was vested in the NAPOCOR under E.O. No. 224.

Petitioners, however, contend that Proclamation No. 573 itself recognizes


private rights of landowners prior to the reservation. They claim to have
established their private rights to the subject land.

We do not agree. No public land can be acquired by private persons without


any grant, express or implied from the government; it is indispensable that
there be a showing of a title from the state. 17 The facts show that
petitioner Gordula, did not acquire title to the subject land prior to its
reservation under Proclamation No. 573. He filed his application for free patent
only in January, 1973, more than three (3) years after the
issuance of Proclamation No. 573 in June, 1969. At that time, the land, as
part of the Caliraya-Lumot River Forest Reserve, was no longer open to private
ownership as it has been classified as public forest reserve for the public good.

Nonetheless, petitioners insist that the term, "private rights", in


Proclamation No. 573, should not be interpreted as requiring a title. They opine
that it suffices if the claimant "had occupied and cultivated the property for so
many number of years, declared the land for taxation purposes, [paid] the
corresponding real estate taxes [which are] acceptedby the government, and
[his] occupancy and possession [is] continuous, open and unmolested and
recognized by the government". 18 Prescinding from this premise, petitioners
urge that the 25-year possession by petitioner Gordula from 1944 to 1969,
albeit five (5) years short of the 30-year possession required
under Commonwealth Act (C.A.) No. 141, as amended, is enough to vest upon
petitioner Gordula the "private rights" recognized and respected in
Proclamation No. 573.

The case law does not support this submission.


In Director of Lands v. Reyes, 19 we held that a settler claiming the
protection of "private rights" to exclude his land from a military or forest
reservation must show ". . . by clear and convincing evidence that the property
in question was acquired by [any] . . . means for the acquisition of public
lands".

In fine, one claiming "private rights" must prove that he has complied
with C.A. No. 141, as amended, otherwise known as the Public Land Act, which
prescribes the substantive as well as the procedural requirements for
acquisition of public lands. This law requires at least thirty (30) years of open,
continuous, exclusive and notorious possession and occupation of agricultural
lands of the public domain, under a bona fide claim of acquisition, immediately
preceding the filing of the application for free patent. The rationale for the 30-
year period lies in the presumption that the land applied for pertains to the
State, and that the occupants and/or possessors claim an interest therein only
by virtue oftheir imperfect title or continuous, open and notorious
possession. 20

Indeed, the possession of public agricultural land, however long the period may
have extended, never confers title thereto upon the possessor. 21 The reason,
to reiterate our ruling, is because the statute of limitations with regard to
public agricultural land does not operate against the State, unless the
occupant can prove possession and occupationof the same under
claim of ownership for the required number of years to constitute a grant from
the State. 22

In the case at bar, petitioners have failed to comply with the mandatory 30-
year period of possession. Their 25-year possession of the land prior to its
reservation as part of the Caliraya-Lumot River Forest Reserve cannot be
considered compliance with C.A. No. 141, as amended.
The Court has no authority to lower this requirement for it cannot amend the
law.

Next, petitioners contend that their "private rights" have been recognized by the
government itself. They point to (1) the letter dated October 24, 1973 of then
NAPOCOR GeneralManager, Ravanal Ravanzo, (2) the action of the
Bureau of Lands which after investigation, declared him qualified to acquire
the land; and (3) the Free Patent issued on January 17, 1974 by the
Undersecretary of Agriculture and Natural Resources, by authority of the
President of the Philippines. Petitioners urge that the findings and
conclusions of the aforementioned government agencies and/or officers are
conclusive and binding upon the courts, as held in the cases of Ortua v.
Singson Encarnacion 23 and Espinosa v. Makalintal. 24

The submissions are unconvincing.


In the first place, there is nothing in Espinosa v. Makalintal that is relevant to
petitioners' claims. On the other hand, our ruling in Ortua v. Singson
Encarnacion that "a decision rendered by the Director of Lands and approved
by the Secretary of Agriculture and Commerce, upon a question of fact is
conclusive and not subject to be reviewed by the courts," 25 was made subject
to the categorical caveat "in the absence of a showing that such decision was
rendered in consequence or fraud, imposition, or mistake". 2 6

Undoubtedly, then General Manager Ravanzo erred in holding that


petitioner Gordula "ha[d] sufficient ground to establish 'priority rights' over the
areas claimed". This error mothered the subsequent error of the
Bureau of Lands which culminated in the erroneous grant of a free patent on
January 17, 1974. The perpetration of these errors does not have the
effect of converting a forest reserve into public alienable land. It is well-settled
that forest land is incapable of registration, and its inclusion in a title nullifies
that title. 27To be sure, the defense of indefeasibility of a certificate of title
issued pursuant to a free patent does not lie against the state in an action for
reversion of the land covered thereby when such land is a part of a public
forest or of a forest reservation, the patent covering forest land being void ab
initio. 28 Nor can the mistake or error of its officials or agents in this regard be
invoked against the government. 29 Finally, the conversion of a forest reserve
into public alienable land, requires no less than a categorical
act ofdeclassification by the President, upon the recommendation of the proper
department head who has the authority to classify the lands of the public
domain into alienable or disposable, timber and mineral lands. 30 There is
none such in this case. LLphil

SO ORDERED.

Regalado, Mendoza and Martinez, JJ .,concur.

||| (Gordula v. Court of Appeals, G.R. No. 127296, [January 22, 1998], 348
PHIL 670-689)
MATEO CARIÑO, petitioner-appellant, vs.
THE INSULAR GOVERNMENT, respondent-appellee.

Coudert Brothers, for appellant.

Solicitor-General Araneta, for appellee.

SYLLABUS

1. REALTY; PUBLIC LANDS; TITLE. — Under the express provisions of


the law, a piece of land being of common origin presumptively belonged to
the State during its former sovereignty, and in order to perfect the legitimate
acquisition of such land by private persons it was necessary that the same
passed from the possession of the State by title of egression, title under
agreement or composicion or title by way of possessory proceedings
equivalent during a certain period to that of adjustment (composicion).

2. ID.; ID.; POSSESSORY INFORMATION; TITLE. — The title under


possessory proceedings, the only title presented herein, is not a proprietary
title authorized in substitution for that of agreement or adjustment by royal
decree of February 13, 1894, this being that last law or decree of the former
sovereignty applicable to the present subject-matter of common lands: First,
for the reason that the land referred to is not covered nor does it come
within any of the conditions required in article 19; second, because the
possessory proceedings authorized in said royal decree for the purpose of
acquiring proprietary title, equivalent to that of agreement or adjustment,
can only be brought or instituted within a period of one year, in accordance
with article 21, and the possessory information or proceedings presented in
this case was instituted on March 7, 1901, and registered on the 11th day
of the same month and year.

3. ID.; ID.; REVERSION. — After the expiration of the period of one


year allowed by such royal decree, the right of the cultivators and persons in
possession to obtain a free title thereto becomes canceled and lapses, and
the land and the full possession thereof reverts to the State or to the
community, as the case may be.

4. ID.; ID.; POSSESSION; TITLE. — The possessors not included or


mentioned in the said provisions of the royal decree can only acquire, by
time, the ownership and title to public alienable lands in accordance with
common law.

5. ID.; ID.; ID.; POSSESSORY INFORMATION. — In accordance with


common law, the possession as attested to and shown in a possessory
information could not go further to show right of ownership or title until
after the expiration of twenty years from the time of verification or registry of
the same in the Registry of Properties, as prescribed in article 393 of the
Mortgage Law and upon other conditions required by this law.

6. ID.; ID.; PRESCRIPTION. — Section 6 of Act No. 627 of the


Philippine Commission admits prescription, in accordance with the terms
and conditions prescribed inAct No. 190, covering the title for the obtaining
of the right of ownership of lands not exceeding an extension of 16 hectares,
but not when the land in question consists of an extension of 40 hectares,
as is the case with the petition presented herein, or of an extension of 28
hectares as referred to in the possessory information proceeding upon which
such petition has been based.

DECISION

ARELLANO, C.J p:

Mateo Cariño, the appellant herein, on the 23d of February, 1904,


filed his petition in the Court of Land Registration praying that there be
granted to him title to a parcel of land consisting of 40 hectares, 1 are, and
13 centares, and situated in the town of Baguio, Province of Benguet,
together with a house erected thereon and constructed of wood and roofed
with rimo, and bounded as follows: On the north, in lines running 1,048
metes and 20 decimeters with the lands of Sepa Cariño, H. Phelps
Whitmarsh, and Calsi; on the east, in lines running 991 meters and 50
decimeters with the land of Kuidno, Esteban Gonzales, and of the
Civil Government; on the south, in lines of 115 meters and 60 decimeters,
with the lands of Talaca; and on the west, in lines running 982 meters and
20 decimeters, with the lands of Sisco Cariño and Mayengmeng.

By order of the court the hearing of this petition, No. 561, and that of
Antonio Rebollo and Vicente Valpiedad filed under No. 834, were heard
together for the reason that the latter petition claimed a small portion of
land included in the parcel set out in the former petition.

The Insular Government opposed the granting of these petitions,


alleging that the whole parcel of land is public property of
the Government and that the same was never acquired in any manner or
through any title of egresion from the State.

After trial, and the hearing of documentary and oral proof, the court of
Land Registration rendered its judgment in these terms:

"Therefore the court finds that Cariño and his predecessors


have not possessed exclusively and adversely any part of the said
property prior to the date on which Cariñoconstructed the house
now there — that is to say, for the years 1897 and 1898,
and Cariño held possession for some years afterwards of but a
part of the property to which he claims title. Both petitions are
dismissed and the property in question is adjudged to be public
land. (Bill of exceptions, p. 15.)

The conclusions arrived at the set forth in definite terms in the


decision of the court below are the following:
"From the testimony given by Cariño as well as from that of
several of the witnesses for the Government it is deduced, that in
or about the year 1884 Cariño erected and utilized as a domicile
a house on the property situated to the north of that property
now in question, property which, according to the plan attached
to expediente No. 561, appears to be property belonging to
Donaldson Sim; that during the year 1893 Cariño sold said
house to one Cristobal Ramos, who in turn sold the same to
Donaldson Sim, moving to and living on the adjoining property,
which appears on the plan aforesaid to be the property of H.
Phelps Whitmarsh, a place where the father and the grandfather
of his wife, that is to say, Ortega and Minse, had lived . . .

"In or about the years 1898 Cariño abandoned the property


of Whitmarsh and located on the property described in the plan
attached to expediente No. 561, having constructed a house
thereon in which he now lives, and which house is situated in the
center of the property, as is indicated on the plan; and since
which time he has undoubtedly occupied some portion of the
property now claimed by him." (Bill of exceptions, pp. 11 and 12.)

1. Therefore it is evident that this court can not decree the registration
of all of the superficial extension of the land described in the petition and as
appears on the plan filed herein, such extension containing 40 hectares, 1
are, and 13 centares, inasmuch as the documentary evidence accompanying
the petition is conclusive proof against the petitioners; this documentary
proof consists of a possessory information under date of March 7, 1901, and
registered on the 11th day of the same month and year; and, according to
such possessory information, the land therein described contains an
extension of only 28 hectares limited by "the country road to the barrio of
Pias," a road appearing on the plan now presented and cutting the land, as
might be said, in half, or running through its center from north to south, a
considerable extension of land remaining on the other side of the said road,
the west side, and which could not have been included in the possessory
information mentioned.

2. As has been shown during the trial of this case, this land, of which
mention is made in said possessory information, and upon which is situated
the house now actually occupied by the petitioner, all of which is set forth
as argument as to the possession in the judgment, is "used for pasture and
sowing," and belongs to the class called public lands.

3. Under the express provisions of law, a parcel of land, being of


common origin, presumptively belonged to the State during its sovereignty,
and, in order to perfect the legitimate acquisition of such land by private
persons, it was necessary that the possession of the same pass from the
State. And there is no evidence or proof of title ofegresion of this land from
the domain of the Spanish Government, nor is there any possessory
information equivalent to title by composicion or under agreement.

4. The possessory information filed herein is not the title to property


authorized in substitution for that of adjustment by the royal decree of
February 13, 1894, this being the last law or legal disposition of the former
sovereignty applicable to the present subject-matter of common lands: First,
for the reason that the land referred to herein is not covered nor does it
come within any one of the three conditions required by article 19 of the
said royal decree, to wit, that the land has been in an uninterrupted state of
cultivation during a period of six years last past; or that the same has been
possessed without interruption during a period of twelve years and has been
in a state of cultivation up to the date of the information and during the
three years immediately preceding such information; or that such land had
been possessed openly without interruption during a period of thirty or
more years, notwithstanding the land had not been cultivated; nor is it
necessary to refer to the testimony given by the two witnesses to the
possessory information for the following reason: Second, because the
possessory information authorized by said royal decree or last legal
disposition of the SpanishGovernment, as title or for the purpose of
acquiring actual proprietary right, equivalent to that of adjustment with the
Spanish Government and required and necessary at all times until the
publication of said royal decree was limited in time to one year, in
accordance with article 21, which is as follows: " A period of one year, not to
be extended, is allowed to verify the possessory informations which are
referred to in articles 19 and 20. After the expiration of this period of the
right of the cultivators and persons in possession to obtain gratuitous title
thereto lapses and the land together with full possession reverts to the state,
or, as the case may be, to the community, and the said possessors and
cultivators or their assigns would simply have rights under universal
or general title of average in the event that the land is sold within a period of
five years immediately following the cancellation. The possessors not
included under this chapter can only acquire by time the ownership and
title to unappropriated or royal lands in accordance with common law."

5. In accordance with the preceding provisions, the right that


remained to Cariño, if it be certain that he was the true possessor of the
land in question, was the right of average in case the Government or State
could have sold the same within the period of five years immediately
following for example, if the denouncement of purchase had been carried
out by Felipe Zafra or any other person, as appears from the record of the
trial of the case. Aside from this right, in such event, his possession as
attested in the possessory information herein could not, in accordance with
common law, go to show any right of ownership until after the expiration of
twenty years from the expiration of twenty years from the verification
and registry of the same in conformity with the provisions of article 393 of
the Mortgage Law and other conditions prescribe by this law.
6. The right of possession in accordance with common law — that is
to say, civil law — remains at all times subordinate to the Spanish
administrative law, inasmuch as it could only be of force when pertaining to
royal transferable or alienable lands, which condition and the determination
thereof is reversed to the government, which classified and designated the
royal alienable lands for the purpose of distinguishing them from those
lands strictly public, and from forestry lands which could at no time pass to
private ownership nor be acquired through time even after the said royal
decree of February 13, 1894.

7. The advent of the new sovereignty necessarily brought a new


method of dealing with lands and particularly as to the classification and
manner of transfer and acquisition of royal or common lands then
appropriated, which were thenceforth merely called public lands, the
alienation of which was reserved to the Government, in accordance with
section 12 and 13 of the act of Congress of July 1, 1902, 1 and in
conformity with other laws enacted under this act of Congress by the
Philippine Commission prescribing rules for the execution thereof, one of
which is Act No. 648, 2 herein mentioned by the petitioner, in connection
with Act No. 627, 3 which appears to be the law upon which the petition
herein is founded.

8. Section 6 of Act No. 627 admits prescription, in accordance with


the provisions contained in Act No. 190, as a basis for obtaining the right of
ownership. "The petitioners claims title under the period of prescription of
ten years established by that act, as well as by reason of his occupancy and
use thereof from time immemorial." (Allegation 1.) But said act admits such
prescription for the purpose of obtaining title and ownership to lands "not
exceeding more that sixteen hectares in extent." (Sec. 6 of said act.) The land
claimed by Cariño is 40 hectares in extent, if we take into consideration his
petition, or an extension of 28 hectares, according to the possessory
information, the only thing that can be considered. Therefore, it follows that
the judgment denying the petition herein and now appealed from was
strictly in accordance with the law invoked herein.

9. And of the 28 hectares of land as set out in the possessory


information, one part of same, according to the testimony of Cariño, belongs
to Vicente Valpiedad, the extent of which is not determined. From all of
which it follows that the precise extent has not been determined in the trial
of this case on which judgment might be based in the event that the
judgment and title be declared in favor of the petitioner, Mateo Cariño. And
we should not lose sight of the fact that, considering the intention of
Congress in granting ownership and title to 16 hectares, that
Mateo Cariño and his children have already exceeded such amount in
various acquirements of lands, all of which is shown in different cases
decided by the said Court of Land Registration, donations or gifts of land
that could only have been made efficacious as to the conveyance thereof
with the assistance of these new laws.

By reason of the findings set forth it is clearly seen that the court
below did not err:

"1. In finding that Mateo Cariño and those from whom he


claims his right had not possessed and claimed as owners the
lands in question since time immemorial;

"2. In finding that the land in question did not belong to


the petitioner, but that, on the contrary, it was the property of
the Government." (Allegation 21.)

Wherefore, the judgment appealed from is affirmed with the costs of


this instance against the appellant. After the expiration of twenty days from
the notification of this decision let judgment be entered in accordance
herewith, and ten days thereafter let the case be remanded to the court from
whence it came for proper action. So ordered.

Torres, Mapa, Willard, and Tracey, JJ., concur.


Johnson, J., reserves his vote.

||| (Cariño v. The Insular Government, G.R. No. L-2869, [March 25, 1907], 8
PHIL 150-157)
[G.R. No. 81564. April 26, 1990.]

ACTING REGISTRARS OF LAND TITLES AND DEEDS OF PASAY CI


TY, PASIG AND MAKATI, METRO
MANILA, petitioners, vs. THE REGIONAL TRIAL COURT, BRANCH
57, IN MAKATI, METRO MANILA PRESIDED OVER BY THE
HONORABLE JUDGE FRANCISCO X. VELEZ, AND THE INTESTATE
ESTATE OF THE LATE DELFIN CASAL, represented by DOMINGO
C. PALOMARES, ADMINISTRATOR, respondents.

[G.R. No. 90176. April 26, 1990.]

THE INTESTATE ESTATE OF THE LATE DELFIN CASAL,


represented by DOMINGO C. PALOMARES,
ADMINISTRATOR, petitioner, vs. HONORABLE CONRADO
VASQUEZ, JR., Presiding Judge, BRANCH 118, RTC, RICARDO P.
SANTIAGO, ET AL., * respondents.

Tañada, Vivo & Tan for the Intestate Estate of the Late Delfin Casal.

Antonio J. Dalangpan for himself and the heirs of Delfin Casal.

Pedro S. Ravelo for Gerardo Casal.

Filomeno Peralta, Jr. for Domingo C. Palomares.

DECISION

SARMIENTO, J p:

The petitioners ** charge His Honor, Judge Francisco


Velez, of the Regional Trial Court, Branch 57, Makati, Metro Manila, with grave
abuse of discretion in issuing an order authorizing the private respondent,
through Domingo Palomares, to perform acts of ownership over a 2,574-
hectare parcel of land known as Hacienda de Maricaban spread out in various
parts of Makati, Pasig, Taguig, Pasay City, and Parañaque. There
is no controversy as to the facts.

On November 5, 1985, the private respondent, Domingo Palomares, as


administrator of the heirs of Delfin Casal, commenced suit with
the Regional Trial Court, Branch 132,Makati, Metro Manila for declaratory
relief, quieting of title, cancellation of Transfer Certificate of Title No. 192, and
cancellation of entries upon Original Certificate of Title No. 291.

Palomares had earlier come to this Court (February 27, 1985) on a similar
petition, and in addition, to direct the Register of Deeds to issue a duplicate
owner's copy of Original Certificate of Title No. 291, embracing allegedly
Hacienda de Maricaban, in lieu of the (alleged) lost one. On September 9, 1985,
the Court denied the petition for lack of merit. (G.R. No. 69834).

On December 19, 1985, the petitioners filed their answer.

On June 2, 1986, the private respondent filed a motion to admit amended


complaint impleading the Republic of the Philippines and the
registers of deeds of Pasig, Makati, andPasay City as parties-respondents, and
alleging, among other things, that: (1) on October 1, 1906,
the Court of Land Registration (James Ostrand, Presiding Judge) confirmed the
title of Dolores Pascual Casal y Ochoa, a native of Madrid, Spain, over the
2,574-hectare parcel above-mentioned; (2) on October 17, 1906, the
Register of Deeds of Rizal issued OCT No. 291 in her name; (3) upon her death,
and successive deaths of her heirs, the property devolved on Gerardo,
Reynaldo, Lolita, and Erlinda, all surnamed Casal, great
grandchildren of Dolores; (4) no conveyances or dispositions of any kind have
been allegedly made upon the parcel; (5) TCT No. 192, which covers the same
landholding, is allegedly spurious and inexistent; (6) the State itself, by placing
27,213,255 square meters thereof under a military reservation (Fort McKinley,
now Fort Bonifacio), byProclamation No. 423, and fifty hectares thereof
pursuant to Proclamation No. 192, had been guilty of landgrabbing; (7) any
and all holders of any and all TCTs emanating therefrom or from TCT No. 192,
are null, void, and of no force and effect; and (8) as a consequence thereof, the
heirs of Dolores Casal suffered various damages and attorney's fees.

On June 26, 1986, the petitioners filed an answer, stating, among other things,
that: (1) the estate of Dolores Casal (or Delfin Casal, her grandchild) is not a
juridical person authorized by law to bring suit; (2) the
registers of deeds of Makati, Pasig, and Pasay City are not the real parties in
interest, but rather, the registered owners over which thecourt had not
acquired jurisdiction; (3) the non-joinder of the real parties in interest is fatal;
(4) OCT No. 291 has long been cancelled; (5) Judge Gregorio Pineda of the
then Courtof First Instance of Rizal, Branch XXI, Pasig, had earlier denied
prayers for the issuance of duplicate owner's copy of OCT No. 291 because
the land embraced therein had been validly delivered to the Government; (6)
the Supreme Court itself had denied the Casals' appeal; *** (7) as a
consequence, res judicata is a bar; (8) prescription has also set in; and (9) the
Casal's claims can not validly override the titles of innocent purchasers for
value.

On August 29, 1986, the respondent judge issued a temporary restraining


order, directing the petitioners to cease and desist from performing the acts
complained of.

In a subsequent memorandum, the petitioners alleged that Dolores Casal had


conveyed the property to the Government of the United States in 1906 and the
Manila Railroad Company on which Judge Ostrand, the Presiding
Judge of the Court of Land Registration, later Justice of this Court, had
stamped his imprimatur.
On October 12, 1987, the respondent court issued an order in the tenor, as
follows:

No other opposition having been registered, this Court hereby


resolves to grant the plaintiffs' prayer in the OMNIBUS MOTION
in order to safeguard the integrity of the landembraced in OCT
291, hereby authorizing for this purpose the plaintiff Domingo C.
Palomares:

1. To order such subdivision and or individual survey


or surveys within Parcel II, Parcel III and Parcel IV under
Survey Plan Psu-2031 by a licensed geodetic engineer or
engineers at plaintiffs' expense in order to facilitate and
simplify the efficient administration of the property
described in OCT 291; and

2. To sell, exchange, lease or otherwise dispose


(of any area or areas or portion or portions thereof, subject
to the approval of the Intestate Estate Court, to cover
expenses for the payment of taxes to which the property is
subject, as well as expenses of administration and for the
protection of the integrity of the said lands.

SO ORDERED. 1

Eleven days later, or on October 23, 1987 to be precise, it issued another order,
as follows:

Acting on the plaintiffs MOTION dated October 15, 1987 praying


for the issuance of a Writ of Execution implementing the
Order of this Court dated October 12, 1987 before the
expiration of the time to appeal, and after inquiring from the
plaintiff s counsel for their reason in seeking the same,
the Court hereby issues this clarificatory order affirming the
power of the plaintiff Domingo C . Palomares to execute and
perform the acts authorized in the said Order of October 12,
1987 without the need of a Writ of Execution, where norelief has
been sought therefrom by any party, said Order being
implementable at the instance of the said plaintiff Domingo C.
Palomares, anytime when the said Order becomes final 15 days
after the said plaintiff received copy of the same (see Section 39,
Chapter IV, B.P. Blg. 129). Plaintiff Domingo C. Palomares may
therefore take whatever steps he considers appropriate for the
implementation of the said Order without need of further Orders
or additional authority from this Court.

SO ORDERED. 2

The petitioners filed a notice of appeal; the respondent court, however, denied
it, 3 "it being directed against . . . an interlocutory order . . ." 4

Hence, this recourse.

The petitioners interpose the following questions:

A. Whether or not respondent Court can validly decide


before trial in favor of private respondent the ownership and
possession of the 25,743,514 square meters (of landknown as
"Hacienda de Maricaban", which is the main issue in this case;

B. Whether or not respondent Court can validly allow private


respondent to exercise and perform all acts of ownership and
possession over the said land before trial;

C. Whether or not respondent Court has acquired j jurisdiction to


hear and decide this action;

D. Whether of not respondent Court committed grave


abuse of discretion amounting to lack of jurisdiction in not
dismissing this action or allowing petitioners to appeal from the
orders in question. 5

In their comment, the private respondent averred, among other things, that: (1)
the respondent court, contrary to the petitioners' claim, did not decide the case
"before trial"; (2) OCT No. 291 had not been validly cancelled and that the
rubber stamp impression thereon, "CANCELLED" is a forgery; (3) the
act of Judge Pineda, in denying issuance of OCT No. 291, duplicate owner's
copy, can not be considered res judicata because that case involved
purportedly a mere petition for issuance of duplicate owner's copy; (4) non-
joinder ofproper parties is not a jurisdictional defect; (5) the TCTs issued
thereafter are a nullity because OCT No. 291 had not been shown to have been
duly cancelled; (6) OCT No. 291 has become imprescriptible; and (7) the private
respondent has a valid right of dominion over the property.

In the meantime, the private respondent came to this Court on certiorari


(G.R. No. 90176) alleging that on December 15, 1987, in connection with Sp.
Proc. No. Pq-2993 of theRegional Trial Court, Branch 118, Pasay City, entitled
"In the matter of the Intestate Estate of the Late Fortunato Santiago and
Mariano Pantanilla, Crisanta P. Santiago, et al., Petitioners," Judge Conrado
Vasquez, Jr. issued an order disposing of certain parcels which the private
respondent claims as forming part and parcel of Hacienda de Maricaban.

On June 20, 1988, the respondent judge in G.R. No. 81564 filed his own
comment, asserting, among other things, that: (1 ) what he had sought to bar,
by virtue of injunction, was incursions and forcible entries of trespassers and
squatters; (2) the petitioners can not rightly claim that he had prematurely
adjudicated the case, because there was allegedlyno decision to begin with; (3)
that he issued the writ of preliminary injunction in order only to maintain
the status quo ante bellum, that is, to re-place the private respondent, which
had been allegedly in prior possession, in possession; (4) he did not allegedly
authorize unbridled "acts of ownership" to be exercised on the property; (5) all
rights ofdominion given thereon were subject to the approval of the intestate
estate court; (6) he denied the notice of appeal because the order dated October
12, 1987, was interlocutory in nature from which no appeal lies; (7) as to
jurisdiction, the various motions filed by petitioners, allegedly accepting
the court's jurisdiction, have clothed the courtwith jurisdiction, and that
besides, the jurisdictional question was never raised except now.

On July 7, 1988, the petitioners filed a reply traversing the respondent judge's
allegations.

On August 26, 1988, the respondent judge filed a supplemental comment. He


reiterated that the writ of injunction was directed only on such spaces not
occupied by the Government (Fort Bonifacio, Libingan ng mga Bayani, Ninoy
Aquino International Airport, Nayong Pilipino, Population Commission,
National Science and Development Board, and National Housing Authority).

Meanwhile, Atty. Antonio J. Dalangpan, for and on behalf purportedly of the


"Heirs of Delfn Casal" and the private respondent, Domingo Palomares, filed a
"Comment Opposition in Intervention", dated December 23, 1988 asking for the
outright dismissal of the petition.

On December 14, 1989, the private respondent filed a manifestation, stating,


among other things, that assuming OCT No. 291 had been cancelled, there was
still basis for the respondent judge to prevent landgrabbers from entering into
vacant portions of the estate embraced thereby.

The Court finds the issues, quintessentially, to be:

(1) Is OCT No. 291 still valid and subsisting?

(2) Did the respondent judge, in issuing the orders, dated October
12 and October 23, 1987, commit a grave
abuse of discretion equivalent to lack or
excess ofjurisdiction?
I.

Is OCT No. 291 still valid and subsisting?

The Court takes judicial notice of the fact that the hectarage embraced by
TCT No. 192 (OCT No. 291) consists of Government property. Three things
persuade the Court: (1) the decrees of Proclamations Nos. 192 and 435; (2) the
incontrovertible fact that OCT No. 291 has been duly cancelled; and (3) the
decision of the Court of Appeals in AC-G.R. CV No. 00293, affirming the
decision of Hon. Gregorio Pineda, Judge of the then Court of First
Instance of Rizal, Branch XXI, in LRC (GLRO) Rec. No. 2484, Case No. R-1467
thereof, entitled "In Re: Issuance of Owner's Duplicate of Certificate of Title No.
291," as well as our own Resolution, in G.R. No. 69834, entitled "Domingo
Palomares, et al., v. Intermediate Appellate Court".

(a)

Proclamation No. 192 ("RESERVING FOR THE VETERANS CENTER SITE


PURPOSES CERTAIN PARCEL OF LAND OF THE PUBLIC DOMAIN SITUATED
IN THE PROVINCE OF RIZAL, ISLAND OF LUZON") and Proclamation No.
423 ("RESERVING FOR MILITARY PURPOSES CERTAIN PARCELS OF THE
PUBLIC DOMAIN SITUATED IN THE MUNICIPALITY OF PASIG, TAGUIG, AND
PARAÑAQUE, PROVINCE OF RIZAL, AND PASAY CITY") have the
character of official assertions of ownership, and the presumption is that they
have been issued by right of sovereignty and in the exercise of the State's
dominical authority. We take not only judicial notice thereof 6 but accept the
same as a valid asseveration of regalian right over property.

With respect to the premises occupied by the Libingan ng mga Bayani, Ninoy
Aquino International Airport, Nayong Pilipino, the Population Commission,
National Science and Development Board, and the National Housing Authority,
we do not have the slightest doubt that they stand on Government property by
sheer presumption that, unless otherwise shown, what the Government
occupies is what the Government owns. LibLex
While there is no presumption that property is Government property until
otherwise shown, because the law recognizes private ownership, thus:

Art. 425. Property of private ownership, besides the patrimonial


property of the State, provinces, cities, and municipalities,
consists of all property belonging to private persons, either
individually or collectively. 7

we find hard evidence on record that: (1) the property covered by OCT No.
291 had been conveyed to the United States of America; (2) it had been later
ceded to the Republicof the Philippines, and (3) as a consequence, OCT No.
291 was cancelled upon final order of Judge Ostrand.

Be that as it may, the private respondent in G.R. No. 81564 is pressed hard to
establish the fact that portions of the property, especially the open spaces
referred to in the lowercourt's writ of injunction and the private respondent's
manifestation of December 14, 1989, and which open spaces it claims to be
outside Maricaban, are indeed outside Maricaban (or OCT 291). With respect,
however, to parts thereof on which Fort Bonifacio, Libingan ng mga Bayani,
Ninoy Aquino International Airport, Nayong Pilipino, Population Commission,
National Science and Development Board, and National Housing Authority sit,
the hands of the private respondent are tied.

Claims that Judge Ostrand's decree was a counterfeit is not only self-serving, it
finds no support from the records. The presumptions is "that official duty has
been regularly performed," 8 and the burden is on the private respondent to
prove irregular performance. The barren insistence that Judge Ostrand's order
was a forgery is not sufficient to overthrow the presumption. To begin with, the
act of forgery has been seasonably disputed by the petitioners. Secondly,
the Acting Registrar of Deeds of Pasig, who supposedly certified to the fake
character of Judge Ostrand's order, has himself joined the other petitioners in
opposing the reconveyance sought.

(b)
The decision in AC-G.R. No. 00293, dismissing the private respondent's
petition for the issuance of a new owner's copy of OCT No. 291, a dismissal
affirmed by this Court in G.R.No. 69834, also militates against the
return of the property to the heirs of Delfin Casal. The Appellate Court's
judgment, a judgment sustained by this Court, operates as, at the very least,
the law of the case between the parties, that OCT No. 291 has been cancelled
and the land covered has been conveyed and ceded to the National
Government. The fact that AC-G.R. CV No. 00293 dealt with a petition for
issuance of lost owner's duplicate copy is no argument because be that as it
may, the private respondent can not rightfully say that the heirs of Delfin Casal
still have title to the land. If it can not secure a new owner's copy, it can mean
that they have lost title thereto.

(c)

The principle of res judicata is also a bar to the instant proceedings. It should
be noted that in G.R. No. 69834, Mr. Domingo Palomares prayed:

WHEREFORE, premises considered it is most respectfully prayed


to the most Honorable Supreme Court, that in the name of law,
justice and fair play, to prevent and frustrate "land-grabbing' by
the government, decision be rendered: cdrep

FIRST, That a thorough review of the aforementioned


resolution of the Intermediate Appellate Court be made;

SECOND, That after due consideration, the


resolution subject of review be set aside based on the
aforestated assignment of error;

THIRD, That the Order of the Lower Court dated Jan.


19, 1977 be affirmed as the lawful and valid order;
FOURTH, To erase all doubts by declaring OCT No.
291 as continuously and existing validly against the whole
world;

FIFTH, Clearing OCT No. 291 of all adverse claims,


since the herein petitioners are the true and legally
declared heirs; and

SIXTH, Ordering the Register of Deeds of Pasig, Rizal


to issue the Owner's Duplicate Copy of OCT No. 291.

Petitioner-Appellant further prays for other just and equitable reliefs. ****

When we therefore denied that petition, we, in effect, held that reconstitution
(of lost duplicate owner's copy) was not possible because the mother title
(OCT No. 291 ) had been duly cancelled. And when we therefore declared
OCT No. 291 to have been cancelled, we perished all doubts as to the
invalidity of Mr. Palomares' pretenses of title to Maricaban. Our judgment was
conclusive not only as to Mr. Palomares, but also as to the existing
status of the property. As we have held:

The lower Court correctly ruled that the present action is barred
by the final judgment rendered in the previous case of Tuason &
Co. vs. Aguila, Civil Case No. Q-4275, of theCourt of First
Instance of Rizal. The reason is plain: if the herein appellants
really had a preferential right to a conveyance of the land from
J.M. Tuason & Co., or if the certificate of(Torrens) title held by
Tuason & Co. were truly void and ineffective, then these facts
should have been pleaded by these appellants in the previous
case (Q-4275), since such facts, if true, constituted a defense to
the claim of Tuason & Co. for recovery of possession. If
appellants failed to plead such defenses in that previous case,
they are barred from litigating the same in any subsequent
proceeding, for it is a well established rule that as between the
same parties and on the same subject and cause of action, a final
judgment is conclusive not only on matters directly adjudicated,
but also as to any other matter that could have been raised in
relation thereto. 9

II

Did the respondent judge, in issuing the order, dated October 12, 1987,
commit a grave abuse of discretion equivalent to lack of excess of jurisdiction
?

(a)

The Court has no doubt that Judge Velez is here guilty of grave
abuse of discretion tantamount to lack or excess of jurisdiction to warrant
certiorari. As above-stated, what he gave away, by virtue of reconveyance, was
property that inalienably belongs to the Government or its successors. Worse,
he gave away property without notice to the actual possessors, that is, the
present registered owner. It is beyond debate, as we have indicated, that
the land had been, since the cancellation of OCT No. 291, parcelled out to a
succession of buyers and owners. In the absence of notice, it
acquired no jurisdiction to decree redelivery or reconveyance. It is well-
established that owners of property over which reconveyance is asserted are
indispensable parties, without whom no relief is available and without whom
the court can render no valid judgment. 10

Furthermore, the present holders of the land in question are innocent


purchasers for value, or presumed to be so in the absence of contrary evidence,
against whom reconveyance does not lie. 11

(b)

The respondent judge can not conceal his faults behind arguments that he did
not intend to convey the premises, but rather, to secure, allegedly, vacant
portions thereof from interlopers. First, this is not stated in his order. Second,
that order is clear and unequivocal that Domingo Palomares has the right "[t]o
sell, exchange, lease or otherwise disposeof any area or areas or portion or
portions thereof . . ." 12 Third and last, the security of the property is the
lookout of the claimants, and not the court's. In case the premises the
respondent judge's injunctive writ have been directed belong to others, let them
air their plaints.

(c)

The Court is also agreed that the challenged order was issued
with no benefit of trial or hearing. The private respondent can not validly rely
on AC-G.R. No. 00293 as the "trial or hearing' to justify the issuance of its said
order, in the first place, because it is a different proceeding. But above all, the
private respondent itself says that AC-G.R. CV No. 00293 can not be made a
basis for denying reconveyance because "the.. petition was merely for the
issuance of a new owner's duplicate copy . . . 13 Accordingly, it can not invoke
that case and yet, repudiate its effects. It is the height of contradiction.

(d)

It was also grave error for the lower court to deny the Solicitor General's
notice of appeal. The Government had all the right to appeal because: (1) the
order of October 12, 1987 was in the nature of a final judgment, as "final
judgment" is known in law (however it is captioned), that is to say, one that
"finally disposes of the pending action so that nothing more can be done with it
in the trial court;" 14 (2) it did not merely maintain the status quo, but allowed
Mr. Domingo Palomares to transact on the property by near-right ofdominion
over it.

Judge Velez had therefore no reason, indeed, excuse, to deny the Government's
notice of appeal. What is plain is the fact that Judge Velez was hell-bent, so to
speak, in blocking the Government's efforts to defend what rightfully belongs to
it.
What has obviously been lost on the parties, Judge Velez in particular, is the
established principle that injunction does not lie "to take property out of the
possession or control ofone party and place it into that of another." 15 In this
wise it has also been held:

xxx xxx xxx

It is a well established doctrine in this jurisdiction that an


injunction is not the proper remedy for the
recovery of possession of real estate and the improvements
thereon, as well as for the ejectments therefrom of the actual
occupants who claim to have title to or material interest therein.
The use of said remedy in such cases has invariably been
considered unjustified, in open violation of the legal presumption
that the bona fide possessor of a certain piece of land and
improvements thereon, holds the same under claimof ownership
and with a just title, and as an advanced concession of the
remedy to which the claimant might be entitled. (Citations
omitted) 16

xxx xxx xxx

Injunction, moreover, is an extraordinary remedy. It lies only in certain cases,


to wit: llcd

Sec. 3. Grounds for issuance of preliminary injunction. — A


preliminary injunction may be granted at any time after the
commencement of the action and before judgment when it is
established:

(a) That the plaintiffs is entitled to the relief demanded, and the
whole or part of such relief consists in restraining the
commission or continuance of the acts complained of, or in the
performance of an act or acts, either for a limited period or
perpetually;

(b) That the commission or continuance of some act


complained of during the litigation or the non-performance
thereof would probably work injustice to the plaintiff; or

(c) That the defendant is doing, threatens, or is about to do, or is


procuring or suffering to be done, some act probably in
violation of the plaintiff's rights respecting the subject of the
action, and tending to render judgment ineffectual. 17

xxx xxx xxx

The conspicuous and unusual zeal with which Judge Francisco Velez now
defends his acts 18 has not escaped us. His Honor should have borne in mind
that in proceedings under Rule 65 of the Rules, such as the present cases, the
judge is included only as a nominal party. Unless otherwise ordained by
this Court, he is not called upon to answer or comment on the petition, but
rather, the private respondent. It is indeed distressing to note that it is the very
judge who has taken the cudgels for the latter, in defending its interests, when
he, the judge, should have remained a neutral magistrate. Res ipsa
loquitor. 19 He must get his just deserts.

III

The Court thus closes the long-drawn tale of Hacienda de Maricaban. In this
connection, let trial judges be cautioned on the indiscriminate
disposition of our dwindling natural resources to private persons. Accordingly,
we grant G.R. No. 81564 and dismiss G.R. No. 90176, and so also, end what
has come down as nearly a century of uncertainty, doubt, and
conflict Maricaban has left in its trail. The Court has finally spoken. Let the
matter rest.

WHEREFORE:
1. The petition in G.R. No. 81564 is GRANTED:

(a) The Writ of Preliminary Injunction issued by our


Resolution, dated April 13, 1988, enjoining the respondent judge
from enforcing his: (i) order ofOctober 12, 1987 and (ii) the follow-
up order of October 23, 1987, is made permanent; and

(b) Original Certificate of Title No. 291 is declared duly


CANCELLED;

2. The petition in G.R. No. 90176 is DISMISSED; and

3. Judge Francisco Velez is ordered to SHOW CAUSE why he should not be


administratively dealt with for giving away, by virtue of reconveyance, property
that inalienably belongs to the Government, without notice to the registered
owner, and without benefit of trial or hearing; for blocking Government efforts
to defend what rightfully belongs to it; and for filing his comment of June 17,
1988 and supplemental comment of August 26, 1988 without express
leave of court.

Costs against the private respondent.

SO ORDERED.

Narvasa, Melencio-Herrera, Cruz, Paras, Feliciano, Gancayco, Padilla, Bidin,


Cortes, Griño-Aquino, Medialdea and Regalado, JJ., concur.

Fernan, C.J. and Gutierrez, Jr., J., are on leave.

||| (Acting Registrars of Land Titles and Deeds of Pasay City v. Regional Trial
Court, Branch 57, Makati, G.R. No. 81564, 90176, [April 26, 1990], 263 PHIL
568-584)

Você também pode gostar